面白い問題おしえて〜な 十三問目

このエントリーをはてなブックマークに追加
1132人目の素数さん
面白い問題、教えてください
2132人目の素数さん:2007/07/06(金) 09:02:00
3132人目の素数さん:2007/07/07(土) 03:30:30
虚数i面体のサイコロをπ回振ったときの期待値を求めなさい
4132人目の素数さん:2007/07/07(土) 04:26:17

('A` ) プウ
ノヽノ) =3'A`)ノ ヒャー>>3
  くく へヘノ
5132人目の素数さん:2007/07/07(土) 06:06:25
虚数iとπを公式にあてはめればいいだろう?
何も屁をかますことはないと思うが…
6132人目の素数さん:2007/07/07(土) 06:35:49
虚数面体ってなに?
明太子と関係ある?
7132人目の素数さん:2007/07/07(土) 07:11:30
誰も丁度に書き込もうとしなかったな
8132人目の素数さん:2007/07/07(土) 14:17:18
ゴルゴのような正確さでスルーにワラタ
9132人目の素数さん:2007/07/07(土) 18:51:39
A君は最初にR^2内の(0,1)にいる
そしてサイコロを振っていき、
・1or2が出ればx座標+1、y座標+1の地点へ
・3or4が出ればx座標+1の地点へ
・5or6が出ればx座標+1、y座標-1の地点へ
という風に移動する
この移動を繰り返しy座標が0の地点に着いたらストップする
線分(0,0)-(0,1)とA君が通ったルート、x軸で囲まれる面積の期待値は幾つ?
10132人目の素数さん:2007/07/07(土) 18:52:20
囲まれる面積→囲まれる領域の面積
11132人目の素数さん:2007/07/07(土) 22:11:12
発散するんじゃないのか、これ?
12132人目の素数さん:2007/07/07(土) 22:12:37
このスレを見ている人はこんなスレも見ています。(ver 0.20)
http://life8.2ch.net/test/read.cgi/kankon/1174579828/l50x [生活全般]
13132人目の素数さん:2007/07/08(日) 00:16:55
次は2008/08/08 08:08:08か。このスレなら射程距離だなw
14132人目の素数さん:2007/07/09(月) 16:24:43
縦、横の長さがa,bである長方形Xを有限個の正方形A1〜Anに分割出来るとき、
a/bは有理数である事を証明せよ
15132人目の素数さん:2007/07/09(月) 23:16:55
こりゃ難しそうだ。
16132人目の素数さん:2007/07/14(土) 23:39:53
自作問題。

(1)f:[0,∞) → Rは2回微分可能であるとし、sup[x≧0]|x^2f''(x)|<∞であるとする。
さらに、任意のε>0に対してΣ[k=1〜∞]f(εk)(−1)^kが存在するとする。このとき、
lim[ε↓0]Σ[k=1〜∞]f(εk)(−1)^k=−f(0)/2 が成り立つことを示せ。

(2)g(x)=Σ[k=1〜∞](−1)^k/√(x^2+k^2) (x∈R)とおく。右辺の級数は任意のx∈Rで
収束することを示し、lim[x→±∞]xg(x)=−1/2となることを示せ(ある大学院入試の過去問より)。

(1)について…与えられた級数に形式的にε=0を入れると、f(0)(−1+1−1+…)となる。
適当な総和法では−1+1−1+…=−1/2となるので、、f(0)(−1+1−1+…)=−f(0)/2
となり、右辺に一致する。……この現象は偶然ではない。今、任意のε>0に対して
Σ[k=1〜∞]f(εk)(−1)^kの存在が保障されているので、アーベル総和法による
シグマ(これをA-Σと書く)でも同じ値になる。すなわち、
Σ[k=1〜∞]f(εk)(−1)^k=A-Σ[k=1〜∞]f(εk)(−1)^k
となる。sup[x≧0]|x^2f''(x)|<∞という条件は、この総和法の下でlimとA-Σの
順序交換が可能であることを保障する条件になっていて、
lim[ε↓0]Σ[k=1〜∞]f(εk)(−1)^k=lim[ε↓0]A-Σ[k=1〜∞]f(εk)(−1)^k
=f(0)A-Σ[k=1〜∞](−1)^k=f(0)(−1/2)
となる。(こんなことしなくても、(1)は解けます)。
17132人目の素数さん:2007/07/15(日) 01:14:01
外接円の半径Rを〜まで読んだ
18132人目の素数さん:2007/07/15(日) 02:28:46
>>17
つ眼科
19132人目の素数さん:2007/07/18(水) 01:09:46
プロ野球の終盤になると安打を打つと打率が二厘上がり凡打だと一厘下がりますが
このことが始まるのは何打数からでしょうか?
20132人目の素数さん:2007/07/18(水) 02:48:48
脳みそ沸いてるの?
21132人目の素数さん:2007/07/18(水) 08:55:57
野球見ている奴って頭悪そうだな・・・
22132人目の素数さん:2007/07/18(水) 09:04:16
おいおい、そういうことは答えてから言え
23132人目の素数さん:2007/07/18(水) 15:28:56
これは流石に、答える前に言っても構わんだろう…
24132人目の素数さん:2007/07/18(水) 15:33:26
打率10割の人は、なん打数目だろうともうそれ以上上がることはないな。
打率0の人は、なん打数目だろうともうそれ以上下がることはないな。

それ以上下がるって変な表現だな。
25132人目の素数さん:2007/07/18(水) 15:43:27
たぶん、あくまで推測の話だが、
普通、打率は1/10000の位で四捨五入して出す。
なので>>19の言っていることには一定の解答を与えることは出来る。

しかし面白くはない。質問スレに持っていくべきだろう。
26転載:2007/07/21(土) 19:18:19
三角形ABCの
Aを中心に半径Max(b,c)、
Bを中心に半径Max(a,c)
Cを中心に半径Max(a,b)
の3つの円を描くとき、それらの合併集合の面積を求めよ。

http://science6.2ch.net/test/read.cgi/math/1183317476/499
27132人目の素数さん:2007/07/22(日) 01:40:48
平面格子上をA=(0,0)をスタート、B=(n,m)をゴールとして通る経路を考える
線分ABに触れず(点A,Bは除く)に格子の辺のみを通ってゴールに至る経路で最短な奴の個数は幾つ?
28132人目の素数さん:2007/07/22(日) 01:55:52
>>27
カタラン数の一般化ですな?
念のため訊いておくが、お主解答は用意出来て御座るか?
29132人目の素数さん:2007/07/22(日) 06:36:00
>>26
正三角形の場合だけやってみた。
a^2*(11pi+3√3)/6

三角形ABCのAを中心に半径r(A).. として
r(A)=a, r(B)=b, r(C)=c
r(A)=c, r(B)=a, r(C)=b
etc とした方がきれいな結果になるような気がしないでもない。
30132人目の素数さん:2007/07/22(日) 13:41:20
>>28
すいませぬ。n>mという条件付で
×線分ABに触れず(点A,Bは除く)に格子の辺のみを通って
○直線y=xに触れず(点Aは除く)に格子の辺のみを通って
だった

27だと階乗使うだけじゃ表せねぇ
31132人目の素数さん:2007/07/22(日) 14:29:14
>>30
あーしてこーしてひっくり返すと
((n-m)/n)*C[n+m-1,m]
32132人目の素数さん:2007/07/22(日) 16:34:56
>26

a≧b≧c としても一般性を失わない。
A,B,C を中心とする円を α,β,γ とすれば、
 (α∪β∪γ) = α + β + γ - (α∩β) - (β∩γ) - (γ∩α) + (α∩β∩γ),
 S(α) = πb^2,
 S(β) = S(γ) = πa^2,
 S(α∩β) = S(α∩γ) = f(a,b),
 S(β∩γ) = f(a,a),

【補題】
 0<r≦R とする。半径Rの円をC, その周上の点を中心とする半径rの円をcとすると,
 共通部分の面積 S(C∩c) は,
  f(R,r) = (π/2)r^2 + (2R^2 -r^2)・arcsin(r/2R) -r√{R^2 -(r/2)^2},
  f(R,R) = {(2π/3) - (√3)/2}R^2,

残った S(α∩β∩γ) をどうするかという問題。
33132人目の素数さん:2007/07/22(日) 17:31:42
>>32
この問題ってa=BC, b=CA, c=ABってことでいいんだよね?

円と円の交点(一番外側にあるやつ)と、三角形の頂点を線分で結んでいくと、
4つの三角形と3つの扇形が出来る。
こっちのほうが楽でない?

34132人目の素数さん:2007/07/22(日) 20:05:48
必要なのってヘロンの公式だけで、わからんスレにあったやつだし、普通に高1の数学じゃね?
結果出してないから何ともいえんけど、結果は綺麗にならない悪寒がする。
35132人目の素数さん:2007/07/22(日) 20:20:37
>>34
やってから言えよ、ジジイ!
36132人目の素数さん:2007/07/22(日) 20:36:57
>32

 S(α∩β) = S(α∩γ) = f(b,a),
【補題】は 0<r≦2R のとき。
r≧2R のときは πR^2 だった. スマソ.
37132人目の素数さん:2007/07/26(木) 02:45:04
>26

>32,34 より、S(α∩β∩γ) を求めればよい。
α周,β周とγ周の交点をD,Eとおく。△BCEは正3角形。

まづ B ≧ π/6 のときを考える。
B ≧ π/3 -B = ∠ABE,
b = AC ≧ AE, Eはα内にある。
CAの延長とγ周の交点をA'とおくと、
α∩β∩γは2直線BC,CA'により3分割される。
α∩β∩γ = (筍形BCE) + (筍形A'CD) - (扇形A'CB),
S(α∩β∩γ) = (1/2)f(a,a) + (1/2)f(b,a) - (1/2)(a^2)C.
38132人目の素数さん:2007/07/26(木) 03:46:02
なぜこの問題はマルチのみならず、マルチ進行が許されるのですか?
教えてください。夏だからですか?
39132人目の素数さん:2007/07/26(木) 19:30:00
deg(f)=2.
deg(g)=2.
f(g(x))=x^4+x^3+x^2+x+1.
40132人目の素数さん:2007/07/26(木) 23:12:11
>>39
解あるの?
41132人目の素数さん:2007/07/27(金) 19:12:45
>37 の続き

B≧π/6 のときは
S(α∩β∩γ) = {(π/3) -(√3)/4 -C/2 +θ/2}a^2 + {(π/2)-θ}b^2 -(a/2)√{b^2 -(a/2)^2},
S(αUβUγ) = {(5/3)π +(√3)/4 -B -C/2 -θ/2}a^2 + {(π/2)-A+θ}b^2 +2Δ + (a/2)√{b^2 -(a/2)^2},
θ = arccos(a/2b), ΔはABCの面積。

>32, >36
S(α∩β) = (a^2)B + (b^2)A -2Δ,
だな。
42132人目の素数さん:2007/07/28(土) 23:28:08
>41 の続き

 B≦π/6 のときは α周とβ周の交点をC,E'とする。E'はCとEの間にある。
 α∩β∩γ = (α∩γ) - (弓形CE') = (α∩γ) - {(扇形CAE') +2Δ -(扇形CBE')},
S(α∩β∩γ) = S(α∩γ) - {(b^2)(B+C) +2Δ -(a^2)B}.

ハァハァ
43名無しさん@そうだ選挙に行こう:2007/07/29(日) 12:08:59
>42 の続き

 CE'は2つの円周に挟まれてるから「三日月形CE'」と言うべきだな。
44名無しさん@そうだ選挙に行こう:2007/07/29(日) 15:27:51
>42 の続き

B≦π/6 のときは
 S(α∩β∩γ) = (B+θ)a^2 +(A-2θ)b^2 -2Δ -a√{b^2 -(a/2)^2},
 S(αUβUγ) = {(4/3)π +(√3)/2}a^2 = 5.05481560…a^2
45132人目の素数さん:2007/07/30(月) 03:31:25
>44 の続き

B≦π/6 のときは
 α ⊂ (βUγ)
 α∩β∩γ = β∩γ
だな…
46132人目の素数さん:2007/07/31(火) 01:27:57
>44 の続き

まちがえた…orz
B≦π/6 のときは
 αUβUγ = βUγ
だった…
47132人目の素数さん:2007/08/01(水) 20:33:38
3乗して下3桁が777になるような整数は存在するか?
48132人目の素数さん:2007/08/01(水) 20:38:27
753^3=426957777
49132人目の素数さん:2007/08/01(水) 21:39:40
N,mを正の整数とし、nをNの桁数とする。
f(N,m)の値を以下の値で定義する。
「N^mを下からn桁ずつ区切っていき、それらの総和をf(N,m)とする。
n桁ずつ区切ったときに最上位の数字が0の部分は桁数がn未満の値として扱う」

(1)任意のnに対してf(N,2)=Nを満たすNが存在することを示せ。
(2)任意のnに対してf(N,2)=f(N,3)=Nを満たすNは存在するか?
50132人目の素数さん:2007/08/01(水) 22:08:38
>>49
問題の意味を俺が理解できているか確認したいんだけど、例えば
f(101,3)=1+30+301=332
ってことでよい?
5149:2007/08/01(水) 22:17:32
>>50
それでOKです。
理解が早くて助かります。
5249:2007/08/01(水) 22:23:57
すいません、一つ追加です
(1)は「二つ以上存在」にしてください。10^k-1以外にも存在することを示す感じで。
5350:2007/08/01(水) 22:29:12
>>49
ありがとう。チャレンジしてみるよ。
54132人目の素数さん:2007/08/04(土) 02:16:42
n!を10進法で表記したとき、それを1の位から見ていき、初めて0でない数が現れた
ところで、その数をf(n)と書く。たとえば
4!=24なのでf(4)=4
5!=120なのでf(5)=2
10!=3628800なのでf(10)=8
となる。自然数nの5進方表示n=納i=0〜u](5^i)aiを与えたとき、
納i=1〜u]i*ai≡t (mod 4)となる自然数tを1つ選べば、
{au!a(u−1)!…a1!a0!}2^t の1桁目の数字がf(n)になっていることを示せ。
55132人目の素数さん:2007/08/04(土) 02:30:50
例:
n=21のとき、5進法でn=納i=0〜u](5^i)ai , u=1 , a0=1 , a1=4
と表せる。納i=1〜1]i*ai≡t (mod 4)となるtはt≡1*a1=4≡0 (mod 4)なので、
t=4が選べる。このとき{au!a(u−1)!…a1!a0!}2^t=4!1!2^4=24*16 なので、
この数の1の位は4となり、f(21)=4となる。
直接求めると、21!=51090942171709440000であるから、f(21)=4である。
56132人目の素数さん:2007/08/05(日) 00:50:58
>>49の(1)をやってみた(出来てないけど)

p=10^nとして
   N^2=ap+b, f(N,2)=a+b=N
の2式から
   a=N(N-1)/(p-1), b=N(p-N)/(p-1)
となる。a, bが整数であるためには、
   N(N-1)≡0 (mod p-1)…(*)
を満たす必要がある(p≡1 (mod p-1)だからbから来る式はいらない)。
任意のnに対して、(*)を満たす10^{n-1} ≦ N < 10^nなるNがあれば万事OK。

p-1以外のNということで、n≡±1 (mod 3)の時は、I=(p-1)/9, d=1,2,...,8として
   N = dI for n≡1 (mod 3)
   N = dI+1 for n≡-1 (mod 3)
がある。ただし、dはnに応じて適当に選ぶ必要がある。だけどn≡0 (mod 3)はこのゾロ目形は通用しない。無念。

詳しくないので(*)の一般的な解法があるのか知らぬ。
5756:2007/08/05(日) 00:56:13
後半の(mod 3)は全部(mod 9)だった。
5856:2007/08/05(日) 01:05:05
簡単に修正しようとして失敗した。

I=(p-1)/9として、n≡1,2,4,5,7,8 (mod 9)の時はd=1,2,...,8を適当に選んで
   nd≡1 (mod 9)と出来る場合 N = dI
   nd≡-1 (mod 9)と出来る場合 N = dI+1
とすればOK。
だけどn≡0,3,6 (mod 9)のときはゾロ目形は通用しない。
59132人目の素数さん:2007/08/05(日) 01:37:36
>>49
(1) N=99・・・9 (n桁)=(10^n)-1は条件を満たす。
実際,N^2=10^(2n)-2*10^n+1において,
下n桁は1,その後のn-1桁は99・・・8 (=10^n-2)なので
それらの和は(10^n)-1=N
60132人目の素数さん:2007/08/05(日) 01:41:23
>>49
(2) 存在しない。
f(2,N)=Nより,N=(10^n)-1と書かれることが必要。
ところがn=1のとき,N=9だが,N^3=729となり,f(3,N)=18≠N
61132人目の素数さん:2007/08/05(日) 02:05:42
>>59
(1)に限っては、>>52で99・・・9 「以外でいつでもあるか」と言ってるよ。

>>60
>f(2,N)=Nより,N=(10^n)-1と書かれることが必要。
その必要は無いよ。
それと(2)では(1)と違って、その形であっても存在すればOKと出題者は言いたいのだと思う。

俺の計算機がしょぼいので、ここまでしか確認できてないけど、
f(N,2)=N
1, 9, 45, 55, 99, 297, 703, 999, 2223, 2728, 4950, 5050, 7272, 7777, 9999, 17344, 22222…
f(N,3)=N
1, 8, 10, 45, 297…

ちなみに俺は>>56だけど、n≡0,3,6 (mod 9)のとき、(1)を満たすNが無いと言いたいのではなくて、どのような形になるかわかってないと言いたかった。
(もちろん俺は出題者じゃないからね。誤解無いように。)
62132人目の素数さん:2007/08/05(日) 04:37:36
単位円がある。Pの後の「_n」は添え字を表すものとする。

OP_1がx軸の正の向きとなす角がθ(0<θ<π/4)となるような点P_1をとる。
∠OP_1P_2=θとなるような点P_2をとる。
∠P_1P_2P_3=θとなるような点P_3をとる。
∠P_2P_3P_4=θとなるような点P_4をとる。

・・・以降同様に、∠P_(n-1)P_(n)P_(n+1)=θとなるような点P_(n+1)を順次とる。

ただし∠P_(n-1)P_(n)P_(n+1)は線分P_(n-1)P_(n)について、∠P_(n-2)P_(n-1)P_(n)と同じ側にある(同位角)とする。

P_(n)の座標をθを用いて表せ。

問題を思いついただけで答えがわからん。考えたけど面倒でやめたから後は任せた。
63132人目の素数さん:2007/08/05(日) 04:43:37
>>62
>問題を思いついただけで答えがわからん。考えたけど面倒でやめたから後は任せた。

ふざけんなwww
他所にそれ相応のスレがある。
64132人目の素数さん:2007/08/05(日) 05:51:28
>>63
え、このスレって答えも用意してないとダメなの?

正直スマンカッタ。問題ごと忘れてくれ。
65132人目の素数さん:2007/08/05(日) 06:24:58
>>64
いや、答えがわからなくてもいいとは思うが、
>考えたけど面倒でやめたから後は任せた。
というところに突っ込みたくなった。それでは面白いかどうかわからないでしょ?
66132人目の素数さん:2007/08/05(日) 06:32:47
>>65
別の問題を考えていた時に、ふと思いついた図形から考えてみた。グラフィカルに面白いと思ったから出してみただけ。そういう意味じゃないって?

いずれにしても答えはわからなかった。軟弱な俺を許して。
67132人目の素数さん:2007/08/05(日) 07:28:50
>>>62
P_3の取り方に(一般性を失う)2通りあるでしょ。そこら辺ちゃんと考えてるの?
6866:2007/08/05(日) 07:31:27
ていうかP_(n)が円周上の点だと明示してなかったorz

今何とか考えてみたらこうなった。

P_(2n)=((-1)^ncos(2n+1)θ,sin(2n+1)θ)
P_(2n+1)=(cos(2n+1)θ,sin(2n+1)θ) ただしn=0の時P_(0)は定義しない。
6966:2007/08/05(日) 07:35:03
>>67
意味わからない。どういうこと?
70132人目の素数さん:2007/08/05(日) 09:40:55
知将
71132人目の素数さん:2007/08/05(日) 09:58:27
>>56
>   N(N-1)≡0 (mod p-1)…(*)
N^2 ≡N (mod p-1)
N と p-1 の最大公約数を q とすると
N ≡1 (mod (p-1)/q)
N ≡0 (mod q)
の二本の式が出てきて、中国剰余定理からp-1未満の
整数解の存在は保証される。
そして、そのNがf(2,N)=Nを本当に満たしているって言うのは
不等号ではさんでゴニョゴニョスればいいはず。
というか、ここまでは前スレの終わりに出てた問題と一緒でしょう。

(2)は繰上りとかがうまく処理できなくて難しいね。
7266:2007/08/05(日) 15:55:37
>>70
多分俺に言ってるんだと思うけど、罵るのはいいからさあ・・・教えて欲しいのよ、こっちは。

まあ、よく考えたら大して面白い問題でもなさそうだったからスルーでもかまわないが。
73132人目の素数さん:2007/08/05(日) 22:33:36
>>72
簡単 かつ つまらない かつ 解答も考えずに書き込んだお前は屑だ!
100年ロムって、そのままs…
74132人目の素数さん:2007/08/05(日) 22:37:21
100年と言わず、半万年ろm(ry
75132人目の素数さん:2007/08/06(月) 01:14:47
半径rの球面上にn個の点(a1,a2,...,an)を配置するとき、
ある1点akと他の点間の各距離の最小値をdkとする。
n=5のとき、d1+d2+d3+d4+d5の値が最大となる配置はどのような配置か?

長年疑問だったのでお尋ねします。
76132人目の素数さん:2007/08/06(月) 01:38:55
mathnoriの問題か?
77132人目の素数さん:2007/08/06(月) 08:57:51
>>62の反省を生かして別問題を作ってみた。答えも用意してあるが、後に公開するそれが間違ってたら指摘を求む。

原点をOとし、0<θ<π/2とする。Pの後の「_n」は添え字を表すものとする。

線分OP_1がx軸の正の向きとなす角がθとなるような点P_1をとる。
次に線分P_1P_2が線分OP_1の延長の正の向きとなす角が2θとなるような点P_2をとる。
次に線分P_2P_3が線分P_1P_2の延長の正の向きとなす角が3θとなるような点P_3をとる。

・・・以下同様に、線分P_(n)P_(n+1)が線分P_(n-1)P_nの延長の正の向きとなす角がnθとなるような点P_(n+1)を順次とっていく。

(1)OP_(n)の座標を、θを用いて表せ。

(2)P_4がy軸上にあるようなθの値を求めよ。

(3)θが(2)で一義的に定まる時、P_4のy座標を求めよ。
定まらない時は、最も小さなθに対応する点P_4を点A、最も大きなθに対応する点P_4を点Bとし、線分ABの長さを求めよ。

なお、既知の角度が求められない場合は三角比の表を用いるなどして良い(注:ここだけ美しくなくて残念)。
78132人目の素数さん:2007/08/06(月) 09:03:55
>>77
また一つ書き忘れたおバカな俺。「線分P_(n)P_(n+1)の長さはいずれも1とする。」これがなきゃ解けねーよ。
79132人目の素数さん:2007/08/06(月) 11:12:22
A[n+2]=(4n+2)A[n+1]+A[n]
B[n+2]=(4n+2)B[n+1]+B[n]
A[1]=1、A[2]=3
B[1]=1、B[2]=1
を満たすA[n],B[n]においてA[n]/B[n]の極限値を求めよ。
8075:2007/08/06(月) 13:01:20
>>76
問題背景は金属錯体化学です。球面縛りはこれが由来です。

中心に金属原子を持ち、その周りに幾つかの原子団(配位子)
が結合(配位)したものを金属錯体と言います。

金属原子に配位したそれぞれの配位子はお互いの立体反発を小さくするような
空間配置をします。

4つの配位子がある場合、正四面体の頂点(平面正方形となる場合もあります)
6つ場合、正八面体の頂点となるような配置になります。

5つの場合は、中心金属を含む平面内に正3角形の頂点をなすように3つ、
残りの2つはその平面と垂直になり、中心金属を通る直線上にとります(三方両錐型)。

数学的に考えて、三方両錐型が一番反発の少ない構造なのかを
知りたくて出題させて頂きました。

数学素人の問題ですがお願いします。
類似の問題がありましたら教えてください。
81132人目の素数さん:2007/08/06(月) 13:15:22
>>78
糞して寝ろ! (゚Д゚)≡゚д゚)、カァー ペッ!!
82132人目の素数さん:2007/08/06(月) 17:30:01
(゚Д゚)
83132人目の素数さん:2007/08/06(月) 22:16:32
>>77
とりあえず2分くらい考えただけだが
(2)π/9,π/7,π/4,π/3,3π/7
(3)2sin3π/7-√3

あってるかどうかとか解説とか、そういうレスはなくていいです
84132人目の素数さん:2007/08/06(月) 22:16:48
>>80
立体反発というのがどういうものか知らないんだけど、
問題>>75の「距離の和」が関係してるの?

むしろそっちのほうが気になってしまうやつがここにいる。
85132人目の素数さん:2007/08/06(月) 22:58:42
距離の二乗の和?
86132人目の素数さん:2007/08/06(月) 23:15:59
>>83
解説は不要とのことなので、感想だけ。

2分でよく答えをはじき出せたねえ・・・。問題として面白いかとか、自分もちゃんと解けるかどうかとかを考えてたら2時間以上かかったよ。
87132人目の素数さん:2007/08/06(月) 23:56:44
>80
 それだったら、静電エネルギーを最小にするんぢゃね?
 Σ[1≦i<j≦n] 1/d(i,j) → min.
8883:2007/08/07(火) 00:04:41
>>86
あ、すいません。解説っていうのは回答に対する考察のことで、問題の解説はほしかったり。
ほとんど当て推量で、
10-1=6+3=9
10-3=6+1=7
10-6=3+1=4
の最右辺の値を分母でcosに持っていけば大丈夫だろうくらいしか考えてませんでした。
ただ一般化はしにくいかも知れませんね。東工大でこのままありそうな問題って感じで。
あと(3)はなんか意味のある問題だったり?
8987:2007/08/07(火) 00:34:28
>80
 3方両錐型のとき
  Σ[i<j] 1/d(i,j) = 6/(a√2) + 3/(a√3) + 1/(2a) = 6.4746915…/a. (たぶん最小)
9075:2007/08/07(火) 00:42:56
>>84,85,87
レスありがとうございます。

イメージとして個々の点の持つ「縄張り」ができるだけ均等になるのは
どのような常態かが知りたくて>>75のような問題文になってしまいました。

この「縄張り」の定義があいまいな為、混乱を招いてしまったと思います。


球面ではなくて円周にした場合、点が幾つであろうと等間隔に
点を円周上に並べれば、各点間の反発が均等になります。

これが球面になるとどうなってしまうのか?
とくに対象性の悪い数の場合はどうか?
が知りたくて出題しました。
9175:2007/08/07(火) 00:57:39
>>89
解答ありがとうございます。
各点間の距離の逆数の和が三方両錐型の場合に最小に
なることの証明は難しいのでしょうか?
92132人目の素数さん:2007/08/07(火) 01:05:30
>>89
6.4746915
をぐぐったら Distributing n Charges on a Sphere
http://tracer.lcc.uma.es/problems/thomson/thomson.html
なんてページが出てきた。
9375:2007/08/07(火) 01:09:35
連投で申し訳ないです。
各点に立体角を割り振るようなうまい定義
ができていないのがダメですね。
9475:2007/08/07(火) 01:14:31
>>92
面白いページですね。
5つのときはやはり三方両錐型ですね。
95132人目の素数さん:2007/08/08(水) 00:24:02
>>88
勘違いすみませぬ。

(1)P_(n)の座標はベクトルを用いて↑OP_(n)=↑OP_(n-1)+↑P_(n-1)P_(n)と表せる。これを用いれば
↑OP_2=↑OP_1+↑P_1P_2=(cosθ,sinθ)+(cos(θ+2θ),sin(θ+2θ)=(cosθ+cos3θ,sinθ+sin3θ)
↑OP_3=↑OP_2+↑P_2P_3=(cosθ+cos3θ,sinθ+sin3θ)+(cos(θ+2θ+3θ),sin(θ+2θ+3θ)
=(cosθ+cos3θ+6θ,sinθ+sin3θ+6θ)
以下同様にすれば
↑OP_(n)=↑OP_(n-1)+↑P_(n-1)P_(n)=(納k=1,n]cos(k(k+1)θ/2),納k=1,n]sin(k(k+1)θ/2))
・・・確かに成り立つかどうかの吟味って必要かな?

(2)P_4がy軸上にある、つまりx座標が0であるから、cosθ+cos3θ+6θ+cos10θ=0。
変形すると-4sin2θsin(9θ/2)cos(7θ/2)=0であり、0<θ<π/2よりsin2θ≠0なので
sin(9θ/2)cos(7θ/2)=0。これを満たすθは0<θ<π/2において、θ=π/9,π/7,3π/7,4π/9 の4つ。

(3)(2)で求めたようにθは一つだけではないから、最小のθに対応する点P_4つまり点Aのy座標は
となる。これはsin(π/7)+sin(3π/7)+sin(6π/7)+sin(10π/7)=2sin(2π/7)である。

最大のθに対応する点P_4つまり点Bのy座標はsin(2π/9)+sin(6π/9)+sin(12π/9)+sin(20π/9)=2sin(2π/9)である。
したがって、線分ABの長さ=2sin(2π/7)-2sin(2π/9)=2sin(π/63)≒0.997。

>あと(3)はなんか意味のある問題だったり?
別にあまり意味は無い。本当はθは一つに定まると思ってた(このへんが浅はかだなあ)から、
それに対応する点の座標を求めるだけのつもりだった。しかし一つには定まらないことに気づいてから、
「だったら複数の点の座標を求め、それが作る多角形についても問題にしよう。」と考えた。

さらに言えば、「どうせθは一つには決まらないんだから、P_4がx軸上にある場合も問題にしてやれ。」
との考えにいたった。それぞれで題意に沿う最小および最大のθに対応する4点を定めて、それが作る台形の
面積を求めるつもりだった。でも自分が大変なのでやめた。時間と気力があったらやってみてね。
96132人目の素数さん:2007/08/08(水) 03:47:31
2^186+1/65 が整数であることを証明せよ。
97132人目の素数さん:2007/08/08(水) 08:34:43
(2^186+1)/65?
98132人目の素数さん:2007/08/08(水) 17:44:15
以下65を法とする
2^6≡64≡-1
2^186≡(2^6)^31≡(-1)^31≡-1
2^186+1≡-1+1≡0
99132人目の素数さん:2007/08/09(木) 11:11:31
自作問題。

(1)f:(-1,1)→Rは次の条件を満たすとする。
・fは(-1,1)上でC^1級である
・|f'(x)|<1 (-1<x<1)が成り立つ
・f(0)=0である
このとき、任意のa∈(-1,1)に対してlim[n→∞]f^n(a)=0が成り立つことを示せ。
ただし、f^n(a)=f(f(…f(a))) (fをn回合成した関数)とする。

(2)f:(-1,1)→Rは次の条件を満たすとする。
・fは(-1,1)上で微分可能である(しかしC^1級とは限らない)
・|f'(x)|<1 (-1<x<1)が成り立つ
・f(0)=0である
このとき、任意のa∈(-1,1)に対してlim[n→∞]f^n(a)=0が成り立つことを示せ。
ただし、f^n(a)=f(f(…f(a))) (fをn回合成した関数)とする。
10089:2007/08/11(土) 01:17:39
>92

ddクス

nが小さいところでは
n=2, 直径,   f(2) = 1/2, a=2,
n=3, 正3角形, f(3) = √3, a = √3,
n=4, 正4面体, f(4) = (3/2)√6, a = √(8/3),
n=5, 三方両錐, f(5) = (1/2) + 3√2 + √3, a(ax)=√2, a(eq)=√3, 
n=6, 正8面体, f(6) = (3/2) + 6√2, a=√2,
n=7, 五方両錐, f(7) = (1/2) + 5√2 + 5√{(5+√5)/10} + 5√{(5-√5)/10}, a(ax)=√2, a(eq)=√{(5-√5)/2},
n=8, 捩れ正方形柱, f(8) < 2 + 6√3 + 3√6, (square anti-prism),
 a(top) = a(bot) = 1.171247738380718…, a(side) = 1.28769352633104…,
n=12, 歪20面体, f(12) < -12 +15√5 +15√{(5+√5)/2},
n=20, 歪12面体, f(20) < 5 +30√3 +15√6 +15√15,
かな。

n=8,12,20 では正多面体からずれている。ヤーン・テラー効果?
10189:2007/08/11(土) 01:51:01
>80

平面正方形では, f(4) < 1+2√2, a=√2,

でつが実在しまつね。

軌道函数どうしの重なり積分が≒0 となる必要があるので、結合角にも制約があり…
静電エネルギーだけで決まる訳ぢゃね…
102132人目の素数さん:2007/08/24(金) 16:36:07
ほしゅ
103132人目の素数さん:2007/08/25(土) 18:24:43
ひまでしたら解いてみてください
6桁の自然数ABCDEFは3桁の自然数ABC*DEFで割り切れる。
6桁の自然数ABCDEFをいくつか見つけてください。
104132人目の素数さん:2007/08/25(土) 19:27:36
ポテンシャル問題でしょ、普通に解けば?ラグランジェとかで?
105132人目の素数さん:2007/08/25(土) 19:32:34
V=eiej/dij
dij=d(ri-rj)
d(ri)=d(rj)
G=V-sjd(rj)
106132人目の素数さん:2007/08/25(土) 22:15:04
>>103
143143 = 143*1001 = 143*143*7
167334 = 167*1002 = 167*167*6

この問題には深い意味ありますか?ただいま検討中。
107132人目の素数さん:2007/08/25(土) 22:34:39
>>104
さすがに (5-1)*2=8 の変数を単純計算では、取り付く島もなかろう。
せめて対称性を分析してからでないと。
108132人目の素数さん:2007/08/26(日) 12:03:50
次の数式は何故そうなるのかわかる方どなたかご教授ください。

鉱山を営むとする鉱業権は、次により評価する。

(1) 操業している鉱山の鉱業権の場合
a×(1/(s+(r/((1+r)^n−1))))−E
(1+r)の後はn乗です。

a 鉱山が毎年実現しうる純収益 
s 報酬利率 9パーセントから15パーセントまでの間において適正に定めた率
r 蓄積利率
n 可採年数 
E 今後投下されるべき起業費の現在価額


(2) 未着手のまま据置期間のある場合の鉱山の鉱業権の場合
(1/(1+r)^m)×a×(1/(s+(r/((1+r)^n−1))))−E
(1+r)の後m乗、  (1+r)のあとn乗

m 据置期間
a、s、r、n及びE (1)に定めるとおりとする。

(3) 開坑後予定収益を生ずるまでに期間のある場合における鉱業権の場合
a×(((1+r)^n−1)/(r+s{(1+r)^n+m−1}))−E

(1+r)のあとn乗、(1+r)のあと n+m乗

m 補償時から予定収益を生ずるまでの期間
a、s、r、n及びE (1)に定めるとおりとする。

どなたかわかる人がいればご教授ください。比較級数の和の公式のようにも思えるし、複利計算の式にも思えるし・・・悩み中です。
109132人目の素数さん:2007/08/26(日) 12:51:28
>>108
マルチ
110132人目の素数さん:2007/09/19(水) 23:53:54
ほしゅ
111132人目の素数さん:2007/09/23(日) 17:10:38
転載。
http://science6.2ch.net/test/read.cgi/math/1190430000/22

半径r の円の中で一回に距離1だけ(好きな方向に)逃げたり
追いかけたりすることが出来る鬼ごっこをするとします。 
最初に、鬼は中央に子は円周にいるとして先に子が逃げるとします。
さて、半径rがある程度大きくなると永遠に逃げ回ることが
可能になるのでしょうか?それとも絶対に捕まるのでしょうか?

また、円以外の閉領域で上の鬼ごっこをするときに
必ず捕まる条件みたいなのは計算可能でしょうか?
112132人目の素数さん:2007/09/23(日) 23:55:07
>111
 鬼は子の方向に追いかけるとする。距離RがR-1になる。
 子が鬼の反対側に逃げた場合には距離がRに戻る。しかし θだけ逸れると
 R - √{R^2 -2(R-1)(1-cosθ)} ≧ (R-1)(1-cosθ)/R だけ近づく。
113132人目の素数さん:2007/09/24(月) 02:14:06
鬼が子の方向に必ず1進めるわけではないので
かならずそれが適応できるわけではない。
114132人目の素数さん:2007/09/24(月) 02:16:23
ああ、ごめん。ルールを勘違いしてた。 交互に逃げたり追ったりするんだね
115132人目の素数さん:2007/09/24(月) 10:16:51
そもそも永遠に逃げられるパターンが思いつけない。
116132人目の素数さん:2007/09/24(月) 11:11:50
(1) 正方形を合同でない二つの相似な図形に分割せよ
(2) 正三角形を合同でない二つの相似な図形に分割せよ
(3) 円を合同でない二つの相似な図形に分割せよ
117132人目の素数さん:2007/09/24(月) 13:18:23
>>115
任意の凸領域(任意の二点を結ぶ線分がその領域内を通るもの)は毎回、鬼が子の方向に進んでいけば距離が小さくなっていく。
よってずっと逃げ回るためには境界が凹領域のところ(つまり、穴というか進入禁止領域)があることが必要。
それでは、どれだけの大きさの穴があれば逃げ回れるのでしょうか?

球面とか、トーラスのように境界がない曲面も逃げ回ることが出来る大きさの最小値がありそう。
118132人目の素数さん:2007/09/24(月) 13:23:22
>>116(1)これでどうだ! 文句あるか!

■■■■■■■■■■■■■■■■
■■■■■■■■■■■■■■■■
■■■■■■■■■■■■■■■■
■■■■■■■■■■■■■■■■
■■■■■■■■■■■■■■■■
■■■■■■■■■■■■■■■■
■■■■■■■■■■■■■■■■
■■■■■■■■■■■■■■■■
■■■■■■■■■■■■□□□□
■■■■■■■■■■■■□□□□
■■■■■■■■■■/□□□□□
■■■■■■■■■■□□□□□□
■■■■■■■■□□□□□□□□
■■■■■■■■□□□□□□□□
■■■■■■■■□□□□□□□□
■■■■■■■■□□□□□□□□

(/の部分は、フラクタル)
119132人目の素数さん:2007/09/24(月) 14:02:17
>>118
フラクタルはダメです
曲線でわけても結構です
120132人目の素数さん:2007/09/24(月) 14:04:07
>>118
相似にならねえじゃん
121132人目の素数さん:2007/09/24(月) 17:48:20
>>116
(1)と(3)は思いついたが(2)が思いつかん。
122132人目の素数さん:2007/09/24(月) 17:49:06
>>120
なるだろ。
123132人目の素数さん:2007/09/24(月) 17:49:24
>>121
解答頼む
124132人目の素数さん:2007/09/24(月) 17:50:34
あ、(2)もできた。
125132人目の素数さん:2007/09/24(月) 17:54:22
結局フラクタルな図形以外でできるのか?
126121,124:2007/09/24(月) 18:19:14
自分が考えたのもどれもフラクタルな図形です。
そうでないのは出来るんだろうか?
127132人目の素数さん:2007/09/24(月) 18:23:40
>>122
ならねえよ
128132人目の素数さん:2007/09/24(月) 18:24:54
>>122
デカい方が角が2つ多いだろ。
129132人目の素数さん:2007/09/24(月) 18:29:20
誰か>>79教えて〜
130132人目の素数さん:2007/09/24(月) 18:32:05
>>128
まず「フラクタル」について調べてから言え。
131132人目の素数さん:2007/09/24(月) 18:32:43
>>127
フラクタルの意味はわかった上で、ならないと言ってるのか?
132132人目の素数さん:2007/09/24(月) 18:46:34
フラクタルはダメって言われてるのにフラクタルにこだわる奴ら
133132人目の素数さん:2007/09/24(月) 18:51:03
条件からはずれたところで揉めるなよ、おまいら。

しかし、フラクタルがダメとなると、4つの角のうち2つずつ引き受けねばならなくなってしまいそうだけどなあ。
でも、そうすると相似に出来ねえし。可能なのか?
134132人目の素数さん:2007/09/24(月) 19:47:11
フラクタル無しは厳しいな。
でも円はフラクタルありでも厳しいな
>>121さん、もし良かったら教えて
135132人目の素数さん:2007/09/24(月) 21:05:47
点でしか接していなくてもひとつの図形としていいなら
円もできたんだが、それでもいいだろうか?

もちろんフラクタル図形。
136132人目の素数さん:2007/09/24(月) 21:20:00
(0-1)+(2-3)+(4-5)+....
(1-2)+(3-4)+(5-6)+....
137132人目の素数さん:2007/09/24(月) 21:54:06
>>135
もしかして三日月がたくさんくっついたような形?
138132人目の素数さん:2007/09/24(月) 22:06:36
>>137
全然違う
139132人目の素数さん:2007/09/24(月) 22:17:49
>>137
135ですが、そうです。
140132人目の素数さん:2007/09/24(月) 22:19:35
>>119
それ以前にフラクタル不使用の解答ってあるの?
ない場合、できない証明をすれば正解かな?
挑戦してみよう。
141132人目の素数さん:2007/09/24(月) 22:33:18
円の場合について考えたんだが
小さいほうの図形が円の外周を含むとしたらそれは連続した曲線としては含めず
また1点でしか含めないんではないかと思う。
連続した曲線として含んでも、2点以上含んでも、大きいほうの図形が構成できない。

つまり、小さいほうの図形の外周は、一点を除いて円の内部になければならない。
そしてその外周は、大きいほうと相似なのだから円形でなくてはならない。

てことは、>>137で言うような点で接するような図形を考えない限りは
ふたつの非合同な相似形には分割できない。

小さいほうが大きいほうの内部に含まれるような図形は自己相似形なので
フラクタルを禁止したら、この分割は出来ないということになる。

ぜんぜん厳密じゃないけど、どう?
142132人目の素数さん:2007/09/25(火) 06:54:04
切ってから組みなおすのではなく、最初からブッツリと二つにしないとけないのだろうか?
幾つかに切り離していいなら、例えば(1)なら
辺の長さが√5の正方形を5つに切り離して、辺の長さが1と2の正方形を作るという話はよくあるが…
143132人目の素数さん:2007/09/25(火) 11:29:54
それはそれで考え進めていいと思う。
144132人目の素数さん:2007/09/25(火) 13:11:20
切り離して組み合わせていいなら、正方形と三角形は簡単なんじゃないか?
145132人目の素数さん:2007/09/25(火) 21:04:25
>>136
これ問題? 「振動する」でいいんじゃない?
その他細かい条件があるのかは知らないけど
146132人目の素数さん:2007/09/25(火) 21:16:49
有限個に切り離して、組みなおしてもよいなら
(i) 長辺/短辺 > 2 の長方形を作る(長辺/短辺 ≦ 2 になってしまったら、また半分に切って組み直す)。
(ii) 長辺/短辺 > 2 の長方形は、長辺の適切な場所で、長辺に垂直に切れば、合同でない相似な二つの長方形に分けられる。
円をこの話に帰結できるかは分からないが。

やっぱり自己相似を使わず、さらに組みなおすこともなく、ということだろう。たぶん
147132人目の素数さん:2007/09/25(火) 21:34:36
んなややこしく考えなくても、組み直していいなら5*5に分けて3*3と4*4にするとかでいいじゃん。
三角形も25分割して16と9で出来るな。
148132人目の素数さん:2007/09/25(火) 21:41:07
…まあ…正多角形なら全部これで片付くということで、目をつむってくれや
149132人目の素数さん:2007/09/25(火) 21:49:33
いくら分割してもよく、組みなおしていいなら楽勝だろう・・・。
150132人目の素数さん:2007/09/25(火) 23:17:20
正方形を中心を通らずに合同な図形に二分割って出来る?
151132人目の素数さん:2007/09/26(水) 00:02:35
出来ぬ
152132人目の素数さん:2007/09/26(水) 10:12:44
>>117
穴があいていなくとも、たとえば半径3くらいの円板でも、
「子は円周を一定方向に回り続け、鬼はそれを馬鹿正直に追跡する」
というアルゴリズムでは、鬼の軌道は円周に漸近してくだけで
追いつけない気がする。つまり、鬼と子の距離は単調減少するが
0には収束しないという状況が起こりうるのではないか。

もちろん、鬼に先回りなどの知能を搭載すれば話は変わってくるけど。
153132人目の素数さん:2007/09/26(水) 10:54:36
1,2,3,...,L[mm]の長さの
L種類の棒を縦に並べて
きっちりL[mm]の長さにするには
何通りの場合があるか?
同じ長さの棒は何度でも使え、
区別もしないとする。
154132人目の素数さん:2007/09/26(水) 16:46:27
>>153
2^(L-1)
155132人目の素数さん:2007/09/26(水) 16:47:57
>>153

L種類の棒で作られるL[mm]の長さが何通りあるかを f(L)で表す。

L=1のとき、 明らかに1mmの棒が一本の1通りである。

L=n (ただしn>1) の場合について考える。
一番上になる棒の長さが1であるものは f(n-1)の上に長さ1の棒を重ねたものと等しい
一番上になる棒の長さが2であるものは f(n-2)の上に長さ2の棒を重ねたものと等しい
一番上になる棒の長さが3であるものは f(n-3)の上に長さ3の棒を重ねたものと等しい

一番上になる棒の長さがL-1であるものは f(n-(L-1))の上に長さ(L-1)の棒を重ねたものと等しい
一番上になる棒の長さがLであるものは1通り
なので
f(L) = Σ_[k=1.Ln-1]{f(k)} + 1 = 2^n-1

この式は L=1のときにも f(L) = 2^n-1 =2^1-1 = 1 なので 当てはまる。
156132人目の素数さん:2007/09/26(水) 16:50:50
下2行訂正

f(L) = Σ_[k=1.Ln-1]{f(k)} + 1 = 2^(L-1)

この式は L=1のときにも f(L) = 2^(L-1) = 2^(1-1) = 1 なので 当てはまる。
157153:2007/09/26(水) 21:19:08
解答

問題の場合の数は
L[mm]の棒の1,2,...,L-1[mm]の箇所に印をつけ
それぞれを切断するか否かの場合の数に等しいので
2^(L-1)
158132人目の素数さん:2007/09/26(水) 22:07:45
同じ部品からなる場合は重複と考える場合はどうだろうか?

( 1+2+1で高さ4のものと 1+1+2で高さ4のものは同じとみなす)
159132人目の素数さん:2007/09/27(木) 02:32:41
>152
 子が外周を回るとき、鬼は子より内側の円周を回るので…
160132人目の素数さん:2007/09/27(木) 10:10:56
円周はまわらんのでは。
161132人目の素数さん:2007/09/27(木) 10:35:28
漸近的に円周に近づくだけで、円周に到達しないということ?
領域の円Aの円周上に中心を取って、一回の移動分の半径の円Bを書く
BとAの円の交点とAの中心を結んだ線が円Bの内側にあれば、鬼は円周に到達可能
円Bの接線と一致するなら、到達不能、か?・・・円Aじゃなくなりそうだけど。
162132人目の素数さん:2007/09/27(木) 22:09:10
↓これ解けばよさげかな

子の座標(X,Y)
X=Rcos(ωt), Y=Rsin(ωt)

鬼の座標(x,y)
r=√((X-x)^2+(Y-y)^2)として
dx/dt =(X-x)/r, dy/dt=(Y-y)/r

R,ωは定数
X,Y,x,y,rは時刻tの関数

t->∞でr->0を示す
163132人目の素数さん:2007/09/27(木) 22:46:01
円である限り追いつかれる?
164159:2007/09/28(金) 03:05:52
>160-163
 鬼は子より内側を回るので…  でした。
165162:2007/09/28(金) 03:57:49
ってマズった

子と鬼が
距離1ずつ交互に逃げるのだったね

ということは
俺が書いたのは
一ステップあたりに
子と鬼が進める距離を無限小にとった場合
もしくは
領域となる円の半径を無限にとった場合に相当する・・・のか?
166132人目の素数さん:2007/09/28(金) 04:22:58
鬼の番のときに子との距離が1以下だったら捕まるということでいいのかな?
167132人目の素数さん:2007/09/28(金) 08:14:52
動くことが出来る領域をK、子を中心とした半径1の円内をM、鬼を中心とした半径1の円内をNとすると
子はMとNの共通部分以外の領域とKの共通部分L=K∪M ∪(MxorN)を動かないと捕まる。

逃げることが出来なくなるのはLが空集合になることを証明すればよい。。。
168132人目の素数さん:2007/09/28(金) 10:11:25
鬼ごっこの問題は日本数学コンクールのヤツかな.

http://www10.plala.or.jp/mathcontest/2006s3.htm
169132人目の素数さん:2007/09/28(金) 11:55:38
問、
マッチ棒85本を使用して正8角形をつくると何個できるか(1本のマッチを隣り合う複数の正8角形の1辺としてもよい )

この問題の答えを出す数式を教えてください
170132人目の素数さん:2007/09/28(金) 12:04:50
野暮な質問だけど
正八角形の一辺はかならずマッチ棒一個の長さで作らなきゃ駄目?
正八角形には重なりがあってもよい?
これが問題の趣旨なら答えなくてもいいけど
171132人目の素数さん:2007/09/28(金) 12:05:35
ここ質問スレだっけ?
172132人目の素数さん:2007/09/28(金) 12:12:31
>>167
何か解く指針になるような表現になってる?
言い換えにすぎない印象なんだけども・・・如何に。
173132人目の素数さん:2007/09/28(金) 12:16:50
>>170

1辺は同じマッチ棒の長さで、昔のサッカーボールの6角形のように辺と辺で繋げてく感じなんですが…。


>>171
すんません。ここ質問スレじゃないんですね。
174132人目の素数さん:2007/09/28(金) 12:17:25
>>169
立体は?
175132人目の素数さん:2007/09/28(金) 13:08:34
平面です
176132人目の素数さん:2007/09/28(金) 13:55:48
二本のマッチの尻と尻を合わせて正8角形がひとつできる。
とりあえずそれだけで42個
177132人目の素数さん:2007/09/28(金) 21:11:54
A , B⊂Nに対して、A+B:={a+b|a∈A , b∈B}∪A∪B と定義する。
また、Aの元の個数を|A|で表すことにする。

(1)|A∩{1,2,…,n}|+|B∩{1,2,…,n}|≧nならば、n∈A+Bとなることを示せ。

(2)自然数列{xn}はliminf[n→∞]n/xn>1/2を満たすとする。
X={xn|n∈N}とおくとき、X+Xに含まれない自然数は有限個であることを示せ。
(十分大きな自然数は高々2個のxnの和で表せる、ということ)
178132人目の素数さん:2007/09/30(日) 15:05:31
(1)
n∈A,もしくはn∈Bの時n∈A+Bは定義より明らかなので
AもBもnを含まない場合を考える

この時
|A∩{1,2,…,n}|+|B∩{1,2,…,n}|
=|A∩{1,2,…,n-1}|+|B∩{1,2,…,n-1}|≧nが成り立っている

以下、背理法でn∈A+Bを示す
あるA,Bが存在して、n∈A+Bではないとする

|A∩{1,2,…,n-1}|={a1,a2,...,ak}=kとすると
B∩{1,2,…,n-1}は{n-ak,...,n-a1}を含まない
(もし含むとするとn∈A+Bではないことに反する)
なのでB∩{1,2,…,n-1}は{1,2,…,n-1}から{n-ak,...,n-a1}を除いた元しか持ち得ず
これはn-1-k個以下である
しかしこれは
|B∩{1,2,…,n-1}|≧n-|A∩{1,2,…,n-1}|=n-k
なので矛盾する

従ってn∈A+B

無駄があるかも
179132人目の素数さん:2007/10/02(火) 18:26:01
正n角形を一筆書きして出来る図形のパターンをp[n] 通りとする。
ただし、回転や鏡影を施して重なるものは同じパターンとします。
例 p[3]=1, p[4]=2

(1) p[5],p[6] を求めよ。
(2) p[n] を求めよ。
180132人目の素数さん:2007/10/02(火) 19:07:12
>>179
>正n角形を一筆書きして出来る図形

すべての正n角形の頂点を通る一筆書き(頂点同士を直線で結ぶ)して出来る図形
星型etc
181132人目の素数さん:2007/10/05(金) 04:38:29
通信網の問題:
互いに離れたところにいくつかの通信基地がある。
これらの基地の間には通信ケーブルが張り巡らされており、
どの二つの基地もちょうど一本のケーブルで結ばれている。
ところがこのケーブルは一方通行でしか情報を送れない。
つまり、二つの基地の間で、どちらかの基地は他方へ情報を送信できるが、逆方向へは直接送信はできない
このような通信基地たちとケーブルによって構成された通信網を考える。
さて、Aを通信基地のひとつとする。
もし以下が成り立つならば、このようなAを通信網の要と呼ぶ

「任意の基地B(A自身は除く)に対して@またはAが成り立つ
 @)AとBの間のケーブルはAが送信側でBが受信側である
   (これをA→Bと書くことにする)
 A)ある基地CがあってA→C→Bである 」

つまり、Aが要であるとはAは自分以外のどの基地へも高々2ステップで情報を送信できる事を意味する。

問題:
どんな通信網も必ず少なくとも一つ要を持つことを示せ
182132人目の素数さん:2007/10/05(金) 07:07:44
同じ問題を出してもしょうがないでしょう。
183132人目の素数さん:2007/10/07(日) 04:03:50
>>181
N個の基地からなる通信網に要Aがあると仮定する。
Aから1ステップで到達できる基地をB={B1,B2,‥,Bm}とし、
残り全部をC={C1,C2,‥,Cn}とする。
仮定より、Cの基地は全て、あるBiから1ステップで到達できる。

ここに新たに基地Xを追加したとき、
・A→XならAが要。
・あるBiに対しBi→Xなら、A→Bi→Xとなるため、やはりAが要。
・X→A、かつ全てのBiに対しX→Biのときは、任意のCjに対し
 あるBkがあってX→Bk→Cjとなるため、Xが要になる。

よって、N+1個のときも要がある。
184132人目の素数さん:2007/10/07(日) 04:10:06
>>117
 直径1以上の円形の穴。円周上も立入り禁止。
 鬼が近付いて来たら、円の中心Oについて対称な点に逃げる。
185132人目の素数さん:2007/10/07(日) 12:23:15
ABCDに正の整数を入れて等式を成立させて下さい

(A÷B)の3乗+(C÷D)の3乗=17
186132人目の素数さん:2007/10/07(日) 12:31:51
「の3乗」なんて書くやつの問題がおもしろい確率を答えよ。
187132人目の素数さん:2007/10/07(日) 12:45:39
「面白い」の定義を答えよ
188132人目の素数さん:2007/10/07(日) 13:30:21
顔の表面が明るい無彩色
189132人目の素数さん:2007/10/08(月) 00:36:40
〔問題〕
 a = logφ = log((1+√5)/2) ≒ 0.481211825 とおく。
次の双曲線函数
(1) y = cosh(ax),
(2) y = sinh(ax),
(3) y = 2cosh(ax),
(4) y = 2sinh(ax),
(5) y = (2/√5)cosh(ax),
(6) y = (2/√5)sinh(ax),
が通る格子点をもとめよ。
なお、(1)の格子点は(3)の格子点、(2)の格子点は(4)の格子点.
190132人目の素数さん:2007/10/08(月) 02:00:14
>>183
俺も解いたけど、解き方が違ったので書いてみる。

背理法で証明ので、要がないと仮定する。
1ステップで到達できる基地の数がもっとも多い基地のうちの一つをAとする。
Aから1ステップで到達できる基地をB={B1,B2,‥,Bm}とし、
残り全部をC={C1,C2,‥,Cn}とする。
要がないという仮定から、Ci -> Bj (任意のi,j) という経路があることと、
Ci -> A (任意のi) を言って、Aの定義に矛盾することを示してOK。
191132人目の素数さん:2007/10/08(月) 03:04:56
>>190
下から2行目だけど、
「要がないという仮定から、あるCiが存在し、任意のBjに対しCi -> Bjとなることと 」
じゃない?
192132人目の素数さん:2007/10/08(月) 08:45:36
>>191
証明のためだけなら、あるCiについて述べればそれで十分だけど
どのCiをとってもそうなっているのだから別にめくじらたてるほどのことでもない。
193132人目の素数さん:2007/10/08(月) 10:14:22
なってねぇだろ
194132人目の素数さん:2007/10/08(月) 11:12:30
ああ、Bのどの基地からも1ステップでいけない残り全部がCだ。
要がないと仮定したのでCは空でない。
195132人目の素数さん:2007/10/08(月) 16:33:19
結局、1ステップで到達できる基地の数が最も多い基地が
自動的に要になるってことか。
196132人目の素数さん:2007/10/08(月) 20:06:07
ABCDEの5人に◯×試験をしたら下の様な回答が帰ってきました
これより各問の正解が◯×どちらであったかを推測して下さい

※abcdefghij
A:OxxOxOxOxx 8点
B:xxOOOxxOOx 7点
C:xxxOxOxxOO 6点
D:OOOOOxOxOx 5点
E:xOOxxOOOxO 4点
197132人目の素数さん:2007/10/08(月) 20:32:45
oxooxoxoox
198132人目の素数さん:2007/10/08(月) 22:35:00
OxOOxOxOOx
199132人目の素数さん:2007/10/09(火) 08:44:43
長さ2009cmの紐があります.
コレを二人が交互に切るというゲームをします.
ひもは一本に付き一箇所だけ, cm単位でしか切れません.
# つまり, (5,2004)に切るのはokで(5.5,2003.5)に切るのはNG
また紐を好きなだけ何本でも重ねて切ることもできます.
# (5,2004) -> ((2,3),(1000,1004)) の様な切りかた
先に切れなくなった方が負けで, パスは出来ません

先手必勝でしょうか後手必勝でしょうか

p.s.
問題書きながら気になったんだけど
> ひもはcm単位でしか切れません.
という条件を緩めて"長さ1cm以下の紐を作っては良けません"にしても面白いかも
200132人目の素数さん:2007/10/09(火) 12:33:53
直径1cm、重さ1g、密度一様の球が毎秒一回転の速さで回っている時の運動エネルギーはいくらか?
201132人目の素数さん:2007/10/09(火) 15:29:33
0
202132人目の素数さん:2007/10/09(火) 15:32:05
回るってのは自転するのか楕円軌道を描いているのかどっちよ?
203132人目の素数さん:2007/10/10(水) 12:37:47
>>200宿題にしか見えない・・・
204132人目の素数さん:2007/10/11(木) 04:27:21
>200

KE = (1/2)Iω^2,
I = (2/5)ma^2
a: 球の半径[m],
m: 球の質量[kg],
I: 球の慣性モ−メント(中心を通る軸まわりの) [kg・m^2],
ω: 回転速度[rad/s],
KE: 運動エネルギー[J]
205132人目の素数さん:2007/10/11(木) 22:34:25
>200
 ω = 2πf,
 f: 回転数 [Hz]
206132人目の素数さん:2007/10/13(土) 04:19:51
>199

定義:
 nを自然数とする。長さnのひもで始めたゲームが
 先手必勝のときnを先手必勝型と呼ぶ。
 そうでないとき後手必勝型と呼ぶ

場に二本のひもが存在し、それぞれが後手必勝型である場合を考える。
この局面はその時点での手番の負けである。
なぜならあなたは二つあるひもを別個に扱い、それぞれに対して必勝法を行えるからだ。
例えば二本の紐をA,Bとする。
ここで相手がAだけに何か操作を行い、Bに何もしなかったとしよう。
この場合、あなたもBには何もしない。
「元Aだった部分」だけに注目して必勝法を一手進めるのだ。
この事はひもが三本以上のときも成り立つ。
即ち、後手必勝型のひものみで構成された局面はその時点での手番の負けである。

同様に、先手必勝型のひも(と長さ1のひも)のみの局面はそのときの手番の勝ち

さて、ここで具体的な値に対して、先手必勝型か後手必勝型か考えてみると
1は後手必勝で、2は先手必勝である。
また、2n−1までの全ての奇数が後手必勝と仮定すると、2n+1は後手必勝である。
∵奇数を二数に分けると必ず偶数と奇数になる
 この偶数を奇数+奇数になるように分ければ2n−1以下の奇数三つができる
 後手必勝のひものみの状態で先手に手番が渡ったので2n+1は後手必勝型

従って全ての奇数は後手必勝であり、偶数は先手必勝。とくに2009は後手必勝
207132人目の素数さん:2007/10/13(土) 19:25:03
(1)
ある商品を買うと6種類の内1種類がランダムでおまけとしてついてきます
このおまけを6種類全部集めるには平均いくつの商品を買えば良いでしょうか?

(2)
ある商品を買うとa種類の内b種類がランダムでおまけとしてついてきます
このおまけをa種類全部集めるには平均いくつの商品を買えば良いでしょうか?
208132人目の素数さん:2007/10/13(土) 22:27:36
オタク的には箱買いすればいい
209132人目の素数さん:2007/10/14(日) 00:24:03
(1)
すでにk(0≦k≦5)種類もっているとき、新たに一個買って
k+1種類になる確率は 1-k/6
よってk種類からk+1種類にするのに平均で買う個数は
Σ[n=1から∞]n・(k/6)^(n-1)・(1-k/6)
=1/(1-k/6)
従って
6種類そろえるまで買う個数の期待値
=Σ[k=0から5](k種類からk+1種類にするまでの期待値)
=1 + 6/5 + 6/4 + 6/3 + 6/2 + 6
=147/10

(2)
同様に考えると、おまけが1種類つく場合のa種類そろえるまでの期待値は
a(1 + 1/2 + 1/3 + ・・・ + 1/a)
商品一個につきb種類のおまけがつく場合は、
「1種類のおまけがつく商品を常にb個セットで買う」と同じことだから
(a(1 + 1/2 + 1/3 + ・・・ + 1/a)) / b
210132人目の素数さん:2007/10/14(日) 07:35:18
>>206
正解.

"長さ1cm以下の紐を作っては良けません",
"長さ1cm未満の紐を作っては良けません"
のケースも暇があったら考えてみてください^^
211132人目の素数さん:2007/10/15(月) 10:01:28
× 「良けません」

△ 「行けません」
○ 「いけません」
212132人目の素数さん:2007/10/15(月) 23:02:50
>209

「b種類のおまけ」と「b個のおまけ」を勘違いしてないか?
213132人目の素数さん:2007/10/16(火) 01:47:05
>189

チェビシェフの多項式より、
(1) (p,q) が格子点ならば (np, T_n(q) ) も格子点
(2) (p,q) が格子点ならば ((2n+1)p, q*U_(2n)(q) )
(3) (p,q) が格子点ならば (np, q*T_n(q/2) ) も格子点
(4) (p,q) が格子点ならば ((2n+1)p, 2*U_(2n)(q/2) ) も格子点
(5) (p,q) が格子点ならば (np, (2/√5)T_n((√5)q/2) ) も格子点
(6) (p,q) が格子点ならば ((2n+1)p, q*U_(2n)((√5)q/2) ) も格子点
214132人目の素数さん:2007/10/16(火) 03:30:30
>>212
なぜそう思う?
215132人目の素数さん:2007/10/16(火) 04:08:32
>>214
おまえの脳を読んだからさ!
216132人目の素数さん:2007/10/16(火) 04:15:13
いや俺は考えていないからそうは思っていないはずだ。
それとも深層心理まで読み取られてしまったのだろうか?
217132人目の素数さん:2007/10/16(火) 19:08:54
任意の実数aに対して

f(a,a^2,a^3)=f(a^2,a^3,a)=f(a^3,a,a^2)=f(a,a^3,a^2)=f(a^3,a^2,a)=f(a^2,a^3,a)=0

を満たすようなx,y,zの多項式f(x,y,z)を求めよ。
218132人目の素数さん:2007/10/16(火) 23:17:13
>>210

1未満を作ってはいけない場合:
まず、>>206で定義した先手必勝型、後手必勝型に加え、「作ると負けになる長さ」を
禁止型と呼ぶことにする。禁止型は広い意味では後手必勝型に含まれると考えられそうだが
これらを区別した方が後の議論に都合がいいので。
>>206の議論は切断位置を実数にした場合にも自然に拡張され
「場に存在するのが後手必勝型のみの局面はその手番の負け」、さらに
「場に少なくとも一本先手必勝型が存在すれば勝ち」が言える。
後者は整数ルールのときから言えたことであり、ここから
「全ての先手必勝型xに対して、少なくとも一つあるyが存在し、yとx-yはともに後手必勝型」
が言える。
さて、半開区間(1,2]の要素はそれ以上切断できない。(切断しようとすると禁止型を生じる)
よってx∈(1,2]のとき、xは後手必勝型
またこの区間の要素の和で表せる(2,4]の要素は全て先手必勝型
ここで、ε>0に対して4+εを先手必勝型とする。
先手必勝型の性質から4+εはある後手必勝型の和で表せるはずだが、既知の後手必勝型は最大でも2なので
可能なのはなにか未知の後手必勝型4+δが存在して4+ε→(4+δ,ε-δ)と分解されるときだけ
ここでε-δ>1であり、とくにε>1
まとめると「4+εが先手必勝型ならばεは1より大きい」
対偶を取って「ε≦1ならば4+εは後手必勝型」
よって半開区間(4,5]の要素は全て後手必勝型
こうしてまた新たに後手必勝型が見付かったので、それらの和で表せる(5,7]や(8,10]は先手必勝型
同様の議論を続けていくと、結局k=0,1,2,3,・・・に対して
(3k+1,3k+2]は後手必勝型であり、(3k+2,3k+4]は先手必勝型
とくに2009=3・669+2は後手必勝型
1以下を作ってはいけない場合もほとんど同様にして半開区間の向きだけ変わり、
2009は今度は先手必勝型になる
219132人目の素数さん:2007/10/17(水) 04:04:21
>217
f(x,y,z) = (xy-z)(yz-x)(zx-y)g(x,y,z),
f(x,y,z) = (xy-z^2)(yz-x^2)(zx-y^2)h(x,y,z),
など。
220132人目の素数さん:2007/10/17(水) 20:01:48
(x-yz)(y-xz)(y-x^2)
221132人目の素数さん:2007/10/18(木) 19:15:11
f(x,y,z) = 0 で十分。
222132人目の素数さん:2007/10/19(金) 11:09:34
>>215
人の脳を(ry
223132人目の素数さん:2007/10/19(金) 12:09:56
媒介変数tを用いて表される二曲線(i),(ii)がある。

(i)x=e^(-t)cost,y=e^(-t)sint
(ii)x=e^(-t)cos2t,y=e^(-t)sin2t

この二曲線で囲まれる部分の面積を求めよ。
224132人目の素数さん:2007/10/19(金) 18:14:30
>>223
t≧0 でいいんかな。1/2 になったけど。
225224:2007/10/19(金) 23:35:07
1/4だた
226132人目の素数さん:2007/10/21(日) 13:27:37
>223
 囲まれる部分はたくさんあるが・・・
 (i)のtと(ii)のtは
227132人目の素数さん:2007/10/21(日) 20:38:30
自然数列{an}(n∈N)は狭義単調増加であって、さらに、あるC>0とあるt≧1に対して
an<Cn^t (n=1,2,3,…)を満たしているとする。自然数mに対して、mの素因数の
最大値をp(m)とおくとき、limsup[n→∞]p(an)=∞ となることを示せ。
228132人目の素数さん:2007/10/23(火) 05:31:39
>>227
limsup[n→∞]p(an)<∞と仮定する。
いずれかのanの素因数であるような素数を小さい順にp1, …,pkとする。
g∈N, g≧pkとするとき、g以下であるような(p1)^(i1)*…*(pk)^(ik) (i1,…ikは0以上の整数)の形の数の個数は
[log(g)/log(p1)]*…*[log(g)/log(pk)]≦[log(g)/log(p1)]^k以下。右辺をrとおく。
以上よりg≦ar<Cr^t≦C[(log(g)/log(p1))^(kt)であるが、この不等式は十分gが大きいとき成立しない。
229132人目の素数さん:2007/10/23(火) 18:49:05
>>207
>(2)
>ある商品を買うとa種類の内b種類がランダムでおまけとしてついてきます
>このおまけをa種類全部集めるには平均いくつの商品を買えば良いでしょうか?

平均 E(a,b) 個の商品を買えば良いとする。
a=b のときは、E(a,b)=1.
以下では、a>b≧1 のときを考える
このおまけを k(≧1) 個買ったとき、a種類のおまけが全部そろっている確率を p(k)
とすると、包除原理より、
p(k)=((C(a,b))^(-k))*Σ[m=0,a-b]C(a,m)*((-1)^m)*(C(a-m,b))^k.
( C(n,m)=n!/(n!*(n-m)!) ) 
よって、
E(a,b)=Σ[k=2,∞] k*(p(k)-p(k-1))
=Σ[m=1,a-b]((-1)^(m+1)*(a!*(2*a!^2*(a-b)!*(a-b-m)!^2*(a-m)!-a!*(a-b)!^2*(a-b-m)!*(a-m)!^2)/(m!*(a-m)!*(a!^3*(a-b-m)!^3-a!^2*(a-b)!*(a-b-m)!^2*(a-m)!))))

計算例:
E(5,4)=9/4,
E(50,40)=(185670706054169305015849546598666174081288628096161)/(55278965274533097503396347784510411985680743479276)
=3.35879…,
E(500,400)=4.72880….
230132人目の素数さん:2007/10/25(木) 16:13:32
P_1 = {1,1}
P_2 = {1,2,1}
P_3 = {1,3,2,3,1}
P_4 = {1,4,3,5,2,5,3,4,1}
....
P_nはP_(n-1)に隣り合う2数の和の値をその間に付け加えるようにして生成していきます。

(1) P_n の要素の数、最大値、要素の和を求めてください。
(2) 互いに素な自然数の順列(a,b)はあるP_kで隣り合う二数に一回だけなることを示してください。
(3) P_(n) にはk (1≦k≦.n)はいくつあるか?
231132人目の素数さん:2007/10/28(日) 06:12:35
age
232132人目の素数さん:2007/10/28(日) 21:20:30
233132人目の素数さん:2007/10/29(月) 20:55:45
>>230
(1)
[解] 要素数をA(n)とすれば、A(n+1)=A(n)+A(n)-1よりA(n)=(2^n)+1。
[解] 総和をS(n)とすれば、S(n+1)=S(n)+2(S(n)-2)+2よりS(n)=(3^n)+1。
[予想] 最大数は1,2から始まるフィボナッチ。

(2)
[略証] aとbが互いに素のとき、写像 (a,b)├→ if a>b then (a-b,b) else (a,b-a)
を考えると、互除法の原理から、これを繰り返し適用すれば必ず有限ステップNで
(1,1)に到達し、その経路は一通りである。すなわち、これを(1,1)から逆に
辿ることにより、唯一の(a,b)が問題文の手続きに従って生成されることがわかる。
よってP_Nは順序対(a,b)を含み、それが唯一である。

(3)
[予想] P_(n) に k (1≦k≦.n) はφ(k)個含まれる。ただしφ(k)は、
1≦i≦kであって、kと互いに素になるようなiの個数。
234132人目の素数さん:2007/10/30(火) 01:06:02
>>233
> (1)
> [予想] 最大数は1,2から始まるフィボナッチ。
最大数をM(n)とするとき、P_n (n≧2)に順列(M(n-1),M(n))および(M(n),M(n-1))が含まれていることを示せばいい。
n=2のときは明らかだから、2以上のnについてそうだと仮定する。
まず、生成規則より、P_nの偶数番要素の最大値はM(n)、奇数番要素の最大値はM(n-1)なので、
M(n+1)≦M(n)+M(n-1)であることに注意する。
P_n内の順列(M(n-1),M(n))および(M(n),M(n-1))からP_(n+1)内の順列
(M(n-1),M(n)+M(n-1),M(n))および(M(n),M(n)+M(n-1),M(n-1))が生成される。
先の注意より、M(n)+M(n-1)がP_nの最大値M(n+1)に等しいことがわかるから、
P_(n+1)には順列(M(n+1),M(n))および(M(n),M(n+1))が含まれていることになり、n+1のときも成立する。
以上によって、数学的帰納法から2以上のすべてのnについて成立する。
M(n+1)=M(n)+M(n-1), M(1)=1, M(2)=2から一般項を求めると
M(n)=((α^(n+1))-(β^(n+1)))/√(5), ただしα=(1+√(5))/2, β=(1-√(5))/2。

> (3)
> [予想] P_(n) に k (1≦k≦.n) はφ(k)個含まれる。ただしφ(k)は、
> 1≦i≦kであって、kと互いに素になるようなiの個数。
n≧2として、P_(n-1)からP_nを生成するときに(a,b)からk=a+bが生成されているとすると
(2)における議論で(a,k)あるいは(k,b)から(1,1)に至るステップ数Nはn-1以下。
よって、2≦k≦nであれば、k=a+bとなるような互いに素な2自然数の順列(a,b)はすべて
P_1,...P_(n-1)のいずれかに含まれていることになる。
このような順列(a,b)は、k以下であってkと互いに素な自然数aと一対一に対応する。
よって、2≦k≦nならば、P_nに含まれるkの個数は、
φ(n)=nΠ(1-(1/p)) (積はnのすべての素因数にわたる)。
ただし、1はP_nに2個(≠φ(1))含まれる。
235132人目の素数さん:2007/11/06(火) 12:09:07
H := { (x,y)∈R^2 ; |xy|≦1 } とする。
H に含まれる三角形の面積の最大値はいくつか。
236132人目の素数さん:2007/11/08(木) 09:49:15
スレ違い承知で。

15秒程度で答えて下さい。
7の8乗はだいたいいくつですか?
紙やペン、電卓などの道具は使わず暗算で答えて下さい。
237132人目の素数さん:2007/11/08(木) 10:00:49
>>236

7*7=49≒50
50*50=2500
25*25=625より
2500*2500=6250000

7*7=49
49*49=2401≒2400
24*24=576より
2400*2400=5760000

二桁の数の二乗がある程度サッと出てくるという前提で。
238132人目の素数さん:2007/11/08(木) 10:03:58
15秒でレスできねえよ
239132人目の素数さん:2007/11/08(木) 11:08:53
>>236は面接で頭の回転の速さを見るための問題と予想。主に外資系。
この手の問題は15秒じゃなくて10秒で答えろって言われるな。
240132人目の素数さん:2007/11/08(木) 21:47:09
このような数学の問題がたくさんのっている書籍ではなにがおすすめでしょうか?自分はまだ高校生で頭もよいほうではないのでなるべく簡単な問題がのっているものがよいです。よろしくお願いします。
241132人目の素数さん:2007/11/08(木) 23:15:09
56764801が10秒以内に出てきた俺は・・・
242132人目の素数さん:2007/11/08(木) 23:16:03
打ち損じた
5764801だ
243132人目の素数さん:2007/11/09(金) 00:13:11
>>242
どうやって考えたのよ
244132人目の素数さん:2007/11/09(金) 00:20:40
>>243
49^2=2401だから7^8=2401^2=5764801
下2桁が01だから計算が楽々
245132人目の素数さん:2007/11/09(金) 00:21:23
「暗算の鬼」参上
246132人目の素数さん:2007/11/09(金) 00:46:26
面接官( `∀´):15秒程度で答えて下さい。7^8は大体いくつですか?暗算で答えて下さい。

>>244 ( ´∀`):7^8=5764801 下2桁が01だから計算が楽々。

面接官( ・д・):

面接官( ・д・ ):
247132人目の素数さん:2007/11/09(金) 00:51:37
面接官( `∀´):15秒程度で答えて下さい。7^8は大体いくつですか?暗算で答えて下さい。

>>244 ( ´∀`):56764801。言い間違えた。5764801だ。下2桁が01だから計算が楽々。

面接官:
      ___    ━┓
    / ―\   ┏┛
  /ノ  (●)\  ・
. | (●)   ⌒)\
. |   (__ノ ̄  |
  \        /
    \     _ノ
    /´     `\
     |       |
     |       |

           ___   ━┓
         / ―  \  ┏┛
        /  (●)  \ヽ ・
       /   (⌒  (●) /
       /      ̄ヽ__) /
.    /´     ___/
    |        \
    |        |
248132人目の素数さん:2007/11/09(金) 00:57:51
むしろ
7*7=49≒50
50^4=6250kくらい

と思ったらかなり数字ちがっててびびった
ある程度は小さくなるだろうけど6000k下回ることはあるまいと思ってたら…

俺数学的センスないなw
249132人目の素数さん:2007/11/09(金) 01:14:46
>>248
お前は俺か!
250132人目の素数さん:2007/11/09(金) 01:34:49
1桁の自然数の常用対数くらい頭に入っているけどな。
log_10(7)≒0.85 の8倍は 6.8 なので
7^8=10^0.8×100万≒600万 (3秒で終了)

もちろん log_10(7)≒0.85 自体は 7^2 ≒ 50 から出せるが、
log_10(7)=0.84509804… (梯子を配れよ)が頭に入ってるしなあ。
251132人目の素数さん:2007/11/09(金) 08:08:37
インド人なら簡単なのかな?
252132人目の素数さん:2007/11/09(金) 08:12:21
49≒50の誤差は何乗かすれば一気に膨らむわけか
253132人目の素数さん:2007/11/09(金) 10:14:18
4乗すれば相対誤差は約4倍。
(εが微小なら (1+ε)^n ≒ 1+nε .
ただしnが大きくなり過ぎるとε^2なども効いてくるが)

本来の49は50の2パーセント減だから4乗すると約8パーセント減となる。
625万から8パーセント(=50万)引くと575万。
254132人目の素数さん:2007/11/09(金) 10:34:10
この手の面接は正解に辿り着く為のプロセスを相手にきちんと説明できるかも問われる気がする。
255132人目の素数さん:2007/11/09(金) 10:39:16
判定予想

×わからない
×数百万くらい
△6250k弱
△6000kくらい
○5800kくらい

ピタリ賞は暗算のプロかカンニングだな
256132人目の素数さん:2007/11/09(金) 10:42:21
判定予想 訂正

×わからない
×数百万くらい
△625k弱
△600kくらい
○580kくらい

ピタリ賞は暗算のプロかカンニングだな
257132人目の素数さん:2007/11/09(金) 11:18:40
k=1000 でっせ
258132人目の素数さん:2007/11/09(金) 13:09:35
2^(1/3) の近似値を暗算で計算せよ。
259132人目の素数さん:2007/11/09(金) 15:21:41
まず1よりデカくてルート2より小さいのはすぐわかる。
1.1^3 を考えると
1.1^2=1.21 でこれをさらに 1.1 倍しても
2 には全然足りないのも何となく分かる。
同様に 1.2^3 も全然足りない。
1.3^3 は 1.3^2=1.69 なのでこれをさらに
1.3 倍すると明らかにオーバー。
なので 1.2 よりデカくて 1.3 より小さい。
これ以上は暗算は大変なので適当に真ん中とって 1.25 。
260132人目の素数さん:2007/11/09(金) 18:36:44
暗算シリーズ
261132人目の素数さん:2007/11/09(金) 20:12:53
20までの三乗の値は覚えておくべきなのかな
262258:2007/11/09(金) 23:45:44
数学板なんだから数学的な工夫をしてくれー
263132人目の素数さん:2007/11/10(土) 00:07:44
125^3=5^9=1953125だから1.25よりちょっと大きい
126^3=2744×9×9×9=24696×9×9=222264×9=2000376だから
1.26より微妙に小さい
誤差が0.019%ほどだから1.26×0.99994=1.2599244よりちょっと大きいくらい
264132人目の素数さん:2007/11/10(土) 00:08:31
>258
 29^3 - 2*23^3 = 24389 - 2*12167 = 55,
より
 2^(1/3) ≒ 29/23 =1.260・・・
265132人目の素数さん:2007/11/10(土) 00:24:15
>263
 63^3 - 2*50^3 = 250047 -2*125000 = 47,
より
 2^(1/3) ≒ 63/50 = 1.26
266258:2007/11/10(土) 00:53:11
出題者ですが、皆様の暗算力に付いて行けませんw
もっとショボい暗算力でも計算できるのですが…
267Sir I.Newton:2007/11/10(土) 01:05:04
>258

 a_1 = 5/4,
 a_(n+1) = a_n - {(a_n)^3 -2}/{3(a_n)^2} = (2/3){a_n +1/(a_n)^2},
により定義された有理数列 {a_n} は 2^(1/3) に収束するぽ。
 a_2 = 63/50,
 a_3 = 375047/297675,
 ・・・・・・
268132人目の素数さん:2007/11/10(土) 01:15:38
>>267
ニュートン法を暗算でやれというわけだな
269258:2007/11/10(土) 02:05:20
Taylor展開による解答を御用意しております。
270132人目の素数さん:2007/11/10(土) 02:24:15
………
271132人目の素数さん:2007/11/10(土) 04:56:22
>269
それじゃぁ仕方ねぇなぁ・・・
 ln(2^(1/3)) = (1/3)ln(2) = 0.6931・・・/3 = 0.2310・・・
 2^(1/3) = exp(0.2310・・・) ≒ 1 + 0.231 + (0.231^2)/2 + (0.231^3)/6 + (0.231^4)/24 = 1.25985・・・
272132人目の素数さん:2007/11/10(土) 08:22:59
【当スレローカルルール】
高2以上の知識はNGとする
273258:2007/11/10(土) 08:46:33
時間が10秒とか15秒程度なら次のようになる。

2^10/10^3 = 1024/1000 = 1.024 の両辺の1/3乗を計算すると、

左辺 = (2*2^9/10^3)^(1/3) = (8/10)*2^(1/3)
右辺 ≒ 1+(1/3)*0.024 (1次近似) であるから

(8/10)*2^(1/3) ≒ 1+0.008

である。両辺に 10/8 = 1.25 を掛けると

2^(1/3) ≒ 1.25 + 0.01 = 1.26
274258:2007/11/10(土) 08:49:38
時間が1分あれば、2次近似も暗算で何とかなる。

(1+ε)^(1/3)
≒ 1 + (1/3)ε + (1/2)(1/3)(-2/3)ε^2
≒ 1 + ε/3 - (ε/3)^2

だから これに ε = 0.024 を代入すると

1.024^(1/3) ≒ 1 + 0.008 - 0.000064

である。これに 10/8 = 1.25 を掛ければ 2^(1/3) の近似値になり

2^(1/3)
≒ 1.25 + 0.01 - 0.00008
≒ 1.25992
275Sir I.Newton:2007/11/11(日) 08:12:06
>>258

 a_1 = 5/4,
 a_(n+1) = a_n -{(a_n)^2 - 2/a_n}/{2a_n +2/(a_n)^2} = (1/2){a_n + 3/[(a_n)^2 +1/a_n]},
の方が収束が早いらしいお。 >>267
 a_2 = 1 + 131/504 ≒ 1.2599206・・・

参考書
 一松 信, 「数値計算」 至文堂 近代数学新書 (1963)
    第2章, 第3節, §38, (2)立方根, 例1., p.151
276132人目の素数さん:2007/11/11(日) 11:53:01
>>269
Taylor展開と聞いてやってみますた
(1+x)^(1/3)=1+(1/3)x-(1/9)x^2+…
にx=1を代入すると 2^(1/3) でつ。
1000次まで計算すると 2^(1/3) > 1.259908747…
1001次まで計算すると 2^(1/3) < 1.259933336…
よって 2^(1/3) ≒ 1.2599
暗算にはかなり苦労しますた。
277132人目の素数さん:2007/11/11(日) 20:09:49
>>267>>275 は純粋な有理数近似の話で、十進表示するか否かに無関係な議論だな。
おかげで最後の十進表示の所で暗算が苦しくなるが…
>>273-274 は十進表示で計算することを考慮した曲芸だな。1024-1000 が3でも8でも
割り切れるのは幸運という他ない。ファインマンさんとソロバン男の話を思い出したぜ。
278132人目の素数さん:2007/11/11(日) 22:11:10
ひろし君のクラスの中から、4人の委員を選ぶことになりました。
クラスの全員がそれぞれ、自分を含めたクラス全員の中から4人の名前を選んで1枚の投票用紙に書きました。
ひろし君がすべての投票用紙を集めて調べたところ、面白いことに気づきました。
2枚の投票用紙をどのように取り出してみても、どちらの投票用紙にも共通して書かれている名前が必ず1人だけ見つかるのです。
このクラスの人数は何人ですか?
ただし、1枚の投票用紙に同じ名前を2人以上書いた人はいませんでした。
279132人目の素数さん:2007/11/11(日) 22:11:26
こんな簡単な質問で悪いんだけど・・・

1,2,3,4,5,6,7の番号がついたカード7枚がある。
7枚から4枚を取り出す場合、何通りあるか?

これってどういう風に考えればいいんだっけ、忘れちゃった・・・_| ̄|○
誰か答えを教えてください。。。
280132人目の素数さん:2007/11/11(日) 22:12:05
>>279
マルチ

終了
281132人目の素数さん:2007/11/11(日) 22:18:02
>>278
8人いたら、二つの投票用紙に全く別の4人の名が書かれている可能性があるから7人てこと?
それじゃ、簡単すぎるか…
投票用紙には4人の名前を書くんだよね?
282132人目の素数さん:2007/11/11(日) 22:21:36
>>281
7人ではないです。

> 投票用紙には4人の名前を書くんだよね?
そうです。
283132人目の素数さん:2007/11/12(月) 01:34:05
>>278
1+3+9=13
284132人目の素数さん:2007/11/12(月) 08:46:45
>>283
むずい。解説きぼん。
285132人目の素数さん:2007/11/12(月) 09:51:49
有限射影平面になるから。
286132人目の素数さん:2007/11/12(月) 10:04:41
【当スレローカルルール】
高2以上の知識はNGとする
287132人目の素数さん:2007/11/12(月) 10:17:02
>>272 >>286 氏ね
288132人目の素数さん:2007/11/12(月) 10:27:24
>>285
F_3 上の射影平面が題意を全部満たすのはわかった。
他に無いってのは、やっぱ射影平面の公理を覚えていないとダメ?
289132人目の素数さん:2007/11/12(月) 10:40:06
てっきり中学入試の問題かと思って一生懸命
順列とか組み合わせでがんばっていたのに
290132人目の素数さん:2007/11/12(月) 10:46:27
そういう解法があっても構わない。>>289
291132人目の素数さん:2007/11/12(月) 12:43:02
生徒は全部で n 人いるとする。

1) どの人を選んでも、その人に投票しなかった人がいる。
ひろし君に m 人が投票したとすると、その m 枚の用紙には
異なる 3m+1 人の名前が書いてある。

2) どの人を選んでも、その人に投票した人は高々 4 人。
ひろし君に 5 人以上投票したとすると、ひろし君に投票しなかった人の票と
その 5 票とに共通する名前が 5 人以上必要。

3) どの人を選んでも、その人に投票した人はちょうど 4 人。
投票用紙に記された名前ののべ総数は 2) より 4n 人以下。
一方、一枚の用紙に 4 人づつ書かれているので総数は ちょうど 4n 人。

4) ひろし君が投票した人の名前を全部の投票用紙から消す。
ひろし君以外の人の投票用紙からはちょうど 1 人の名前が消されるので、
残った名前の総数は 3(n-1) 人。これに n-4 人の名前が 4 回づつ書かれて
いるので、3(n-1)=4(n-4). したがって、n=13.
292132人目の素数さん:2007/11/12(月) 13:06:05
ヒルベルトが「机と椅子とビアマグでも幾何学は構築できる」と言った意味が
よくわかったよ >>285>>291
293278:2007/11/12(月) 23:18:05
>>285
難しくてよくわかりません・・・

1人が4つの名前を書きますから、平均すると1つの名前が4回。
まず、全員の名前が4回ずつ出なければならないことを示します。
全員が4票ずつの得票ではないと仮定すると5票以上得票した人
がいることになります。
ここではAという人が5票獲得したとします。
すると、Aの名前が書いてある5枚は
ABCD
AEFG
AHIJ
AKLM
ANOP
のようになります。
上記の5枚以外の紙を1つ取り出すと、その紙にはBCDのだれかの名前が
記されています。また同様に、この紙にはEFGのどれか、HIJのどれか、
KLMのどれか、NOPのどれかが記されていることになり、最低5人の名前
が記されることになり、1枚の紙には4人の名前が記されることに矛盾します。
一般にAがn票獲得したとき、そのn枚以外の紙には、最低n人の名前が記さ
れることになります。
各票には4人の名前が記されているはずだから、5票以上獲得する人はいない
ことになります。
以上で、全員の名前が4回ずつ出なければならないことが示されました。
294278:2007/11/12(月) 23:18:40
つづき

全員の名前が4回ずつ出てくる場合、
1枚の紙にABCDと書いてあったとすると、
この紙のほかにAの名前が書いてある紙が3枚、B,C,Dの名前が書いてあ
る紙もそれぞれ3枚ずつあります。
しかも、これらは全てABCDのうちのどれか1つしか書かれていないはずなの
で重複しません。
また、ABCDのどれも含まない紙は存在しません。
よって、全員の名前が4回ずつ出てくるとすると、そのときの紙の枚数は13枚
でなければならない、すなわちクラスの人数は13人でなければなりません。

以上のことから、解が存在する場合は13人以外はありえません。あとは13人で
題意を満たす組合せがあることを示せば終わりです。
295132人目の素数さん:2007/11/13(火) 01:07:10
>>293 の真ん中あたりの「上記の5枚以外の紙を1つ取り出すと」
とありますが、そのような紙が少なくとも1枚は存在することも
一応述べておかねばなりません。もちろん、他の紙が無ければ、紙の
枚数とクラスの人数が合いませんのでスグに他の紙の存在がわかります。
296292:2007/11/13(火) 02:10:01
>>292 で書いた事のココロを描いてみる。

[言葉の言い換え]
◎「紙」と呼ぶ代りに「直線」と呼ぶ。これは単なる名称であって、
通常のユークリッド幾何の直線をイメージしてはいけない。以下の
記述もこれと同様である。
◎「人名」と呼ぶ代りに「点」と呼ぶ。
◎「紙Lに人名Aが記されている」と言う代りに「直線Lが点Aを通る」
あるいは「直線L上に点Aがある」と言う。

[言葉の更なる定義]
◎直線Lと直線Mが共に点Aを通るとき、「直線Lと直線Mは点Aで交わる」
あるいは「点Aは直線Lと直線Mの共有点である」と言う。

[問題文の言い換え]
次の条件をみたす「点の集合」と「直線の集合」があるとき、
点の総数を求めよ。
(a) 少なくとも1点が存在する。(←ひろしくん)
(b) 点の総数と直線の総数は等しい。
(c) どの直線上にも、点がちょうど4つある。
(d) どの2直線も、ちょうど1点で交わる。
297292:2007/11/13(火) 02:11:00
[291氏の解答の翻訳]
1) どの点を選んでも、その点を通らない直線が存在する。
(証) 仮に「すべての直線が通過する点A」が存在したら、(d)より
(点の総数)=3*(直線の総数)+1 となって(b)に反する。

2) どの点を選んでも、その点を通過する直線は4本以下である。
(証) 点Aを5本以上の直線が通過したら、これらの直線と 1)で示した
「点Aを通らない直線」との交点が5点以上あることになり、(c)に反する。

3) どの点を選んでも、その点を通過する直線はちょうど4本。
(証) 直線が通過する点ののべ総数は、2)より 4*(点の総数)以下である。
一方(c)(d)よりそれは 4*(点の総数)に等しい。よって各点を通過する直線は
ちょうど4本でなければならない。

4) 点の個数は13個以外にはありえない。
(証) これは291氏のものよりも 278氏の >>294 の方がわかりやすい。
(a)(b)より少なくとも1本直線がある。それをLとすると、L上の各点と
交わる直線が3本ずつあるので、Lと交わる直線の総数は 3*4=12本である。
(d)より、この12本以外の直線はL以外には存在しないので、直線の本数は
13本でなければならない。(b)より点の個数は13以外にはありえない。
298292:2007/11/13(火) 02:12:37
>>294 が最後で触れた、十分性の確認をします。(モデルの構成)

[点゛]
空間の点 (x,y,z) のうち x,y,z の値が 0,1,-1 のいずれかであるものは
3^3 = 27 個ある。このうち原点を除くと26個である。この26個を
(x,y,z)≡(-x,-y,-z) で2つずつ同一視したものを「点゛」と呼ぶと、点゛は
全部で 26/2 = 13 個ある。これで問題文の条件(の翻訳)(a)は満たされる。

[直゛線゛]
a,b,c は 0,1,-1 のいずれかとし、(a,b,c)≠(0,0,0) とする。すると方程式
ax+by+cz=0 上には普通の点(x,y,z)が8個、「点゛」は4個ある。この点゛集合を
「直゛線゛」と呼ぶと、(a,b,c)と(-a,-b,-c)は同じ直゛線゛を定めるので
やはり13個ある。これで問題文の条件(の翻訳)(b)(c)は満たされる。

あとは条件(d)だが、2つの「直゛線゛」の「共有点゛」とは、通常の3次元空間の
言葉では 2平面 ax+by+cz=0 と a'x+b'y+c'z=0 の交線上にある点のうち、最初に
述べた同一視で「点゛」に落ちるものである。絵を描けばわかるが、どの2つの
「直゛線゛」も、ちょうど1つの「共有点゛」を持つことが確かめられる。
(もっとキチンと言えんものか)
299132人目の素数さん:2007/11/13(火) 03:13:39
>>294
> 解が存在する場合は13人以外はありえません。あとは13人で
題意を満たす組合せがあることを示せば終わりです。

>>288 のF_3 上の射影平面というのがこれの例。
>>298 は同じものをわかりやすく説明しようとして、ちょっと失敗している。
((1,1,1) 上に (1,1,1) がないとまずい)

一般の場合、この例を作れるかどうかが難問で、問題文の 4 人を 7 人や
11 人にすると題意を満たす組合せは存在しないのだそうだ。
300292:2007/11/13(火) 10:36:51
>>299
> >>298 は同じものをわかりやすく説明しようとして、ちょっと失敗している。
> ((1,1,1) 上に (1,1,1) がないとまずい)

じま゛っ゛だ。
x+y+z=0 上に (x,y,z) は6個、点゛は3個しかないね。
このスレの住人なら合同式は大丈夫だろうから、直゛線゛の定義を
ax+by+cz≡0 (mod3) に変更しておくよ。

> 問題文の 4人を 7人や11人にすると題意を満たす組合せは存在しないのだそうだ。

13人が未解決問題とか聞いたことあるな。
301132人目の素数さん:2007/11/14(水) 22:25:32
昔の話で悪いですが


>>266
しょぼい暗算で出来る2^(1/3)のだしかたはどうやるのですか?
302258:2007/11/14(水) 23:05:24
303132人目の素数さん:2007/11/18(日) 16:59:39
2^(1/3)
これを音階で考える。
2^(1/3) = 2^(4/12) であるから、これは ド−ミ の長3度である。
俺の経験上、これは1.25より僅かに大きい。


なんて感覚的な答じゃだめかな…
304258:2007/11/18(日) 23:11:49
素敵な答をありがとう! もっと理屈をこねると次のようになるね。

ミは平均律のミよりも僅かに下げないとハモらない。
ハモるのが 5/4 = 1.25 (純正律)だから 2^(1/3) は 1.25 よりちょっと大きい。

音階も研究したピタゴラスなら次のように言うだろう。

私の音階理論によれば、それは (9/8)^2 = 1.265625 が正しい。
無理数? そんなものは存在しない。(そんな事を言う奴は海に沈めてやる)
305132人目の素数さん:2007/11/19(月) 05:29:18
豆知識;
ちなみに実際の音楽で用いられる平均律は
2の12乗根を基にした半音というわけではないので
平均律のミはドの2^(1/3)倍の周波数ではない
306132人目の素数さん:2007/11/19(月) 05:50:49
>>305
ちょっと誤解を招く表現だな。
平均律で調律されるとされる楽器は実際にはそれとは異なる音程に調律されると言うべきだろう。
307132人目の素数さん:2007/11/19(月) 06:17:19
【米国】フランスの「人間計算機」、200桁の数字の13乗根の暗算で72.438秒の世界記録[071116]
http://news21.2ch.net/test/read.cgi/news5plus/1195394463/
308132人目の素数さん:2007/11/21(水) 15:39:19
何を言ってるかわかりませんが、平均律はちゃんと周波数比で等分しますよ。
一般に西洋音楽で使われる十二平均律は12等分します。
ただ、実際にどんな音律を採用するかの段階で
必ずしも普段耳にするものが(十二)平均律でない、という事であって。
平均律は平均律です。
309132人目の素数さん:2007/11/21(水) 20:11:22
平均律で調律されている楽器を探すほうが難しいくらいだったのだが
電子楽器の台頭でそうでもなくなったな
310132人目の素数さん:2007/11/22(木) 01:08:48
さて問題は「2^(1/3)を如何に計算するか」なのだ。今話題の手段を用いて 2^(1/3) ≒ 1.26 と
結論するには、

2^(1/3) ≒ 1.26 が純正律の 5/4 = 1.25 よりも 0.01 だけ大きい

という事を、この精度で感覚的にワカル事が必要だ。比率としては
1.26/1.25 = 1.008 だが、これは近似的には半音程 2^(1/12) ≒ 1.059463 の
8/59.463 ≒ 0.1345 倍の音程だ。厳密には対数を用いて
log_{2^(1/12)}( 2^(1/3)/1.25 ) ≒ 0.136862861351651825556166846127317889622… 倍
だがもちろんこれは冗談として、結局

「純正律のミ」と「理論的な12平均律のミ」の音程の違いが、半音の1/10強である

と言える感覚の持ち主なら、2^(1/3) ≒ 1.26 を感覚的に計算したと言える。
カラオケで半音は平気でズレる私には、とても無理なことである。
311132人目の素数さん:2007/11/22(木) 01:21:13
1/8 = 0.125 < 0.135 < 1.42857… = 1/7 だから

「純正律のミ」と「理論的な12平均律のミ」の音程の違いが、半音の 1/8 以上 1/7 以下である

と言える人なら完璧。
312132人目の素数さん:2007/11/22(木) 13:08:25
>258

√2 > 1.4142 より
 (3 -√2)^3 = 45 - 29√2 < 45 -29*1.4142 = 45 - 41.0118 < 2^2,
3乗根をとると
 3 -√2 < 2^(2/3),
したがって
 √(3-√2) < 2^(1/3) < 2/(3-√2) = (2/7)(3+√2) < (2/7)*4.4143,
 1.2592 < 2^(1/3) < 1.2613
313132人目の素数さん:2007/11/22(木) 13:15:06
>258

 29*29*2 - 41*41 = 1682 - 1681 = 1,
 29√2 > 41,
314132人目の素数さん:2007/11/22(木) 13:31:06
>>313
途中送信みたいだが、2^(1/3) ではなく 2^(1/2) を計算しているように見える。
315132人目の素数さん:2007/11/22(木) 13:34:13
もしも、x^3-2y^3=±1 の整数解で大きい値のがあれば 2^(1/3)≒ x/y となるけど
どうなのでしょうか? ペル方程式の3次元バージョンですけど。
316132人目の素数さん:2007/11/22(木) 13:36:56
ペル方程式みたいに、解の系列を生成する漸化式があればよいわけだ。
317132人目の素数さん:2007/11/22(木) 13:42:58
単に有理数近似で収束が速いものなら >>275 (2次収束)があるし、
暗算可能なシンプリシティーを追求したものなら >>274 がある。
新たな解法には何らかの美が要求されるぜ。(音階の話は美しいな)
ギャクでもいいけど。
318132人目の素数さん:2007/11/22(木) 13:45:01
×ギャクでもいいけど。
○ギャグでもいいけど。
319132人目の素数さん:2007/11/22(木) 20:15:27
>317
ギャクも真なり…
320132人目の素数さん:2007/11/23(金) 07:23:46
音楽家を含め絶対音感の持ち主とは何度も話をした事があるが
1/7を持ち出したひとは初めてだ。(1/8や1/4はよく聞く。)

おそれく半音の半分の半分とか、さらにその半分というのが
感覚的にわかりやすいのだろう。

1/8と1/7の違いくらいデリケートな話になると、セント(半音の1/100)単位か
A音の周波数(なぜか2hz単位の偶数のことが多い)で話をしていた。
321132人目の素数さん:2007/11/23(金) 10:30:31
1/7を持ち出したひとは絶対音感の持ち主ではなく、ただの数学野郎だ。
1/7と言えばガムランのスレンドロ音階はオクターブを7等分するな。

セントみたいに細かいと、「音楽に必要な音程」というよりも「調律用語」か
「民族音楽学の記録用の単位」になってしまうが、「全音の1/9」くらいなら
中東の古典音楽では必須の微分音程だ。

で、2^(1/3) を暗算で計算したいのだ。
322132人目の素数さん:2007/11/23(金) 12:50:50
>>315
x^3-2*y^3=±1 の大きな整数解は見つからないねえ。
仕方が無いので (x,y)=(5,4) が乗ってる曲線 x^3-2*y^3 = -3 で解を生成する
漸化式を立ててみたが、整数解ではなく有理数解を生成する漸化式だったので、
絶対値が大きくなってくれず、2^(1/3)の近似計算には失敗したorz
323132人目の素数さん:2007/11/23(金) 16:21:32
>>320
1ヘルツの違いはA音あたりだと聞き分けるの難しいんだよホント
2ヘルツ違うとよくわかる
324132人目の素数さん:2007/12/04(火) 18:39:31
保守
325132人目の素数さん:2007/12/07(金) 21:27:28
あげ
326132人目の素数さん:2007/12/07(金) 21:50:22
よく小学生に出す問題

3、3、7、7
この4つの数字を使って24を作りなさい
ただし使っていいのは四則演算のみ、数字をくっつけるのは無しです(33など)
327132人目の素数さん:2007/12/07(金) 22:41:58
消費税率100%のとき
税込み100円のものをかいました

さて消費税はいくら?

328132人目の素数さん:2007/12/07(金) 22:47:53
(3+3/7)*7=24
3/3+7*7=50円
329132人目の素数さん:2007/12/08(土) 00:52:43
地方消費税を考慮する必要はありますか?
330132人目の素数さん:2007/12/08(土) 02:09:04
ab+c=a(b+c/a)
331132人目の素数さん:2007/12/08(土) 15:07:47
3^n-2 (nは自然数)に含まれる素因子の逆数の和が発散することを示せ。
注:3^n-2 の素因数分解にあらわれるすべての素数って意味ね
332132人目の素数さん:2007/12/09(日) 01:27:46
p:3^n-2の素因数として表れない素数とする
このとき
任意のnに対して
3^n ≠ 2 (mod p)
一方、p=3のときを例外として、明らかに
3^n ≠ 0 (mod p)
従って、任意の自然数nに対し
3^n = 1 (mod p)
つまりpは3^n-1を常に割り切る
とくにn=1のときを考え、pは2を割り切る
2を割り切る素数、それは2しかない
∴pは3^n-2の形の整数の素因数に現れない ⇒ p=2またはp=3

逆に3^n-2が2でも3でも割れないのは明らか
よって
(3^n-2の素因数全体の逆数和)
=(2と3を除く全ての素数の逆数和)

素数全体の逆数和は発散するのでもとめる級数も発散する
333132人目の素数さん:2007/12/09(日) 02:36:02
>>332
7行目がわかりません。
なんで「従って」?
その時点ではpは3より大きいかもしれないんだから
3^n ≠ 2 (mod p) かつ3^n ≠ 0 (mod p) だからって
3^n ≠ 1 (mod p) とはならないんじゃない?
334333:2007/12/09(日) 02:41:31
間違った。
誤)3^n ≠ 1 (mod p) とはならないんじゃない?
正)3^n = 1 (mod p) とはならないんじゃない?
335132人目の素数さん:2007/12/09(日) 04:00:56
3^n ≠ 0 , 2 (mod p) から言えることは
3^n = 1,3,4,5,6,…,3^n−1 (mod p)だけだよな。
336132人目の素数さん:2007/12/09(日) 04:24:57
>>332
13 は 3^n-2 の素因数にならない
337132人目の素数さん:2007/12/09(日) 16:44:29
332です
どうも頭の中でmod pの話だったのがいつのまにかmod 3にすりかわって
ヘンな勘違いをしてしまったようです
なんでこんなことにも気づかないまま書き込んでしまったのか
我ながら理解に苦しみます
338132人目の素数さん:2007/12/11(火) 07:22:29
>>335
いえません
339132人目の素数さん:2007/12/11(火) 07:50:06
3^n ≠ 0 , 2 (mod p) から言えることは
3^n = 0,1,2,3,4,5,6,…,3^n−1 (mod p)だけだよな。
340132人目の素数さん:2007/12/11(火) 07:56:37
3^n ≠ 0 , 2 (mod p) から言えることは
3^n = 3^n (mod p)だけだよな。
341132人目の素数さん:2007/12/11(火) 23:30:23
自分で未解決の問題でもここに書いて良いのか?
342132人目の素数さん:2007/12/13(木) 00:30:28
面白ければ。
343132人目の素数さん:2007/12/13(木) 00:33:43
面白くないと承知しねーぞ。
344132人目の素数さん:2007/12/13(木) 00:38:02
では一つ。
(a, b) から R への C^1 級関数で、
有理数を有理数に、無理数を無理数に移す関数は1次関数に限るか?
345132人目の素数さん:2007/12/13(木) 09:21:47
f(x)=1/x
346132人目の素数さん:2007/12/13(木) 10:46:06
そんな簡単なのがあったか!
ではもう一つ。
(a, b) から R への C^∞ 級関数で、
代数的(実)数を代数的数に、超越数を超越数に移す関数は代数関数に限るか
347132人目の素数さん:2007/12/13(木) 10:56:25
>>346
は複雑すぎるようだから、
>>344
で、単調増加とするとどうだろうか?
348132人目の素数さん:2007/12/13(木) 12:06:10
349132人目の素数さん:2007/12/13(木) 14:30:08
>>347
f(x)=1-(1/x) on (0,1)

定義域をR全体にすると難しいかも。
350132人目の素数さん:2007/12/13(木) 18:33:10
>>345
を聞いて>>349に気が付かないとは俺も頭悪い。
そろそろ数学やめようか。
351132人目の素数さん:2007/12/13(木) 19:32:32
>そろそろ数学やめようか。

面白い問題だな!
352132人目の素数さん:2007/12/13(木) 22:33:05
0から9までの数字が書いてあるカードが一枚ずつあり、一列に並べて10桁の数を作った。
その10桁の数は、一番大きい位からn番目までのn桁の数を取り出してできる数はnで割り切れるという

この10桁の数は何でしょう?
353132人目の素数さん:2007/12/13(木) 22:39:22
どなたかエクセルで3次方程式の確実な解法教えて下さい。
354132人目の素数さん:2007/12/14(金) 01:14:17
>352
3816547290
355132人目の素数さん:2007/12/20(木) 03:18:50
>>354
おお、本当にそうなってる、うめぇw
356132人目の素数さん:2007/12/22(土) 00:31:38
(1/1√1)+(1/2√2)+(1/3√3)+(1/4√4)+……+(1/n√n)+……
はいくつに収束するか?
357132人目の素数さん:2007/12/22(土) 03:51:21
>356

(与式) = ζ(3/2) = 2.6123753486 8548834334 8567567924 0716305708 0065240006 3407573328 2488149277 6768827286 0996243868 1263119523 8…
358132人目の素数さん:2007/12/22(土) 17:19:35
\Gamma (1/2), \Xi (s) 見たいに「函数等式」って無かったのか!
359132人目の素数さん:2007/12/25(火) 22:25:17
>>354
今さらながら、どうやったの?
360132人目の素数さん:2007/12/25(火) 22:32:36
>>354
超うめぇw
361132人目の素数さん:2007/12/25(火) 22:35:41
>>353
エクセルじゃないけど、三次方程式の解の公式(カルダノの公式)↓


http://www2.biglobe.ne.jp/~ytajima/cardanos_formula.html
362132人目の素数さん:2007/12/25(火) 22:52:11
>>359

>>354ではないけど、倍数判定をしていけばよろし。
順番としては最後に7の倍数判定かな
順に絞っていくと、6通りぐらいに絞れて、最後に7の条件を満たす候補を探す。
363132人目の素数さん:2007/12/26(水) 05:08:17
OA=OB=OC=1として四面体OABCがある。頂点Oの立体角をπとするとき、この四面体の体積Vの範囲。
364132人目の素数さん:2007/12/26(水) 12:11:41
>>315
ペルの3次は
x^3+ay^3+(a^2)z^3-6xyz=1
だよ
aは立方数じゃないからな
365132人目の素数さん:2007/12/29(土) 14:20:50
動点Pが(0,0)から(1,1)まで移動する。
途中、Pの座標値(x,y)のうちどちらか一つが必ず有理数になるように移動すると考えたとき、
Pの移動距離の最小値を求めよ。
366132人目の素数さん:2007/12/29(土) 16:25:27
2
367132人目の素数さん:2007/12/29(土) 20:47:53
>>364
具体的な一般解(全ての解)の例を挙げてくれ
a = 2, 3 位で。
368132人目の素数さん:2007/12/29(土) 21:06:20
xy平面上に曲線y=log x と直線y=m(x-2)がある。ただしm>0。
この二つのグラフで囲まれる面積の最小を求めよ。
369132人目の素数さん:2007/12/31(月) 15:54:09
>>368
最小は2交点が x=2±t, log(2+t)+log(2-t)=0 となるとき。t^2=3 なので
m=(log(2+√3))/(√3) のときに最小。面積も大してキレイにならない。
議論の精密化をしたところで面白い話は出てこない。つまんないなー。

ここは「面白い問題」を楽しむ所だよ。
370132人目の素数さん:2007/12/31(月) 22:22:18
http://www6.atwiki.jp/omoshiro2ch/
を見てみると、代数の問題が少ないようだがどうしてだろう。
371132人目の素数さん:2007/12/31(月) 23:31:38
自作問題投下。

無限に広がる平面があって、同じ大きさの正方形のマス目に区切られているとする(碁盤の目のように)。
これらのマスのうち有限個のマスを黒で塗りつぶす。n個のマスを塗りつぶしたとする。
塗りつぶしたマス目に、1からnまでの自然数を1つずつ書き込む(順番はどうでもよい)。
各項が0と1のみから成る、n次の正方行列Aを次のように定める。

・Aの(i,j)成分=(iが書いてある黒マスとjが書いてある黒マスが隣接しているとき) 1 , (それ以外のとき) 0

ただし、i=jのときは、iが書いてある黒マスとjが書いてある黒マスは隣接していると
定義しておく(従って、Aの対角成分は全て1である)。このAについて、次が成り立つ
ことを示せ。

・A^(n^2)の成分が全て正ならば、塗りつぶしたn個の黒マスから成る図形は連結である。
・A^(n^2)の成分のうち0のものがあるならば、塗りつぶしたn個の黒マスから成る図形は連結でない。
372132人目の素数さん:2007/12/31(月) 23:32:27
転載ですがどうぞ

1)10cmの直線ABを引く、

2)直線ABのAから90度の角度で、直線AC(3cm)を引く

3)直線ABのBから88度の角度で、直線BD(3cm)を引く

4)点Cと点Dを結ぶ直線を直線CDとする。これで台形(のような)四角形ができる

5)直線ABの中点E1から垂線を引く

6)同様に、直線CDの中点E2から垂線を引く

7)E1の垂線とE2の垂線の交わる点をFとする

8)Fから、点ABCDに線を引く

9)三角形AFCと三角形BFDを考える。この2つは異なる形状となるのが見て取れるだろう

10)しかし、三角形は3辺の長さが同じなら合同のはずである。

10-1)三角形AFCの辺AFと、三角形BFDの辺BFは、ABの中点から伸ばした垂線の任意の1点からの長さなので、同一

10-2)同様に辺CFと辺DFも長さは同じ、辺ACと辺BDは初期条件が3cmでこれも同じ


問題:この話のどこかに間違いがある、それを見つけろ
373132人目の素数さん:2007/12/31(月) 23:32:45
たとえば、n=4として、以下のように4マスを黒で塗りつぶしたとする。

□□□□□
□■■■□
□□□■□
□□□□□

次に、1から4までの番号を適当に書き込む。例えば

□□□□□
□CAB□
□□□@□
□□□□□

と書き込む。このとき、行列Aは

1010
0111
1110
0101

となる。A^4を計算すれば、A^4の成分は全て正だと分かるので、A^(n^2)=A^16もまた、成分が全て正だと
分かり、よって、塗りつぶしたn個の黒マスから成る図形は連結である(実際、連結になっている)。
374132人目の素数さん:2008/01/01(火) 00:02:22
>>371
グラフ理論で良く知られた事実と思うが。
375132人目の素数さん:2008/01/01(火) 00:05:38
>>372
問題:この話のどこかに「間違い」がある、それを見つけろ
×××××××××××××↑ここに「間違い」があるじゃん
376132人目の素数さん:2008/01/01(火) 00:13:13
>>732
これも良く知られた話だ。
F が非常に遠くにあって、三角形BDFが裏返っているから。
377132人目の素数さん:2008/01/01(火) 01:27:12
>>374
そうなのか。グラフ理論のどんな定理?
378132人目の素数さん:2008/01/01(火) 08:56:24
>>377
定理という程の名前はなかったと思う。
379132人目の素数さん:2008/01/01(火) 10:42:59
正二十面体の一つの頂点を出発して、全ての辺を二度ずつ通り、
元の頂点に戻ってくる経路があることを示せ。ただし、正し各辺は違う向きに一度ずつ通る物とする。

正十二面体ではどうか?

ここでは図を書けないので理論で答えよ。
380132人目の素数さん:2008/01/01(火) 11:38:11
a を非負の実数とする。
i) Σ[n = 1 → ∞] a^(√n), ii) Σ[n = 2 → ∞] a^(log n), (iii) Σ[n = 2 → ∞ ]a^(√log n)
が収束する a の範囲をそれぞれ求めよ。
381132人目の素数さん:2008/01/01(火) 21:47:40
>>380
問題文列記が汚い
人に読ませる力がない
論文書いても読まずにポイ

382132人目の素数さん:2008/01/01(火) 21:59:46
では書き直し。

a を非負の実数とする。

(i) Σ[n = 1 → ∞] a^(√n),
(ii) Σ[n = 2 → ∞] a^(log n),
(iii) Σ[n = 2 → ∞] a^(√log n)

の三つの無限級数について、それぞれ収束する a の範囲を求めよ。
383132人目の素数さん:2008/01/02(水) 04:30:54
>>382
(ii)があると易し過ぎてツマラナイ。
(ii)がないと結果がツマラナイ。
ゆえにこの問題は
384132人目の素数さん:2008/01/02(水) 05:15:44
>>378
ということは、かなり簡単に導けるってことか…
実は、Aの作り方を

(i≠jのとき) Aの(i,j)成分=(iが書いてある黒マスとjが書いてある黒マスが隣接しているとき) 1 , (それ以外のとき) 0
(i=jのとき) Aの(i,i)成分=(iが書いてある黒マスの上下左右4箇所が全て黒マスのとき) 0 , (それ以外のとき) 1

に変えても

・A^nの成分が全て正ならば、塗りつぶしたn個の黒マスから成る図形は連結である。
・A^nの成分のうち0のものがあるならば、塗りつぶしたn個の黒マスから成る図形は連結でない。

が成り立つのだが( A^(n^2)は勘違いで、A^nで十分ですた )、これも簡単に導ける?
385132人目の素数さん:2008/01/02(水) 08:39:11
>>379
こうゆう消防でも問題自体はわかる問題は良問ノ予感
ただ「理論」というのが漠然として
386132人目の素数さん:2008/01/02(水) 09:44:31
>>385
「理論」は大げさだった。
「図で具体例を書くより、簡単な文章で」ぐらいに解して置いて下さい。
387132人目の素数さん:2008/01/02(水) 11:00:26
>>384
変形した方の問題は兎も角として、元の問題は、連結でなければ、
A を何乗しても 0 は消えないし、 A^k の要素が全て正ならば、更に A を掛けてもそうだから、
小さい n で示せば十分。グラフ理論の結果では、
「A^(n-1) の各要素が正である事が必要十分。」
が簡単に導けていたと思う。 A^(n^2) については、その系として、
「必要十分が知られている」と言う意味で書いた。

変形した方の問題に対応する直接のグラフ理論の結果は知らないが、
何かの結果から出そうな気はする。
388132人目の素数さん:2008/01/02(水) 11:33:43
>>367
便宜上2^(1/3)=pとおく
a=2なら
(1+p+p^2)^n=x[n]+p*y[n]+p^2*z[n]
として
x[n]^3+2y[n]^3+4^2*z[n]^3-6x[n]y[n]z[n]=1
が任意のnで成り立つよ.
389132人目の素数さん:2008/01/02(水) 12:30:38
>>386
>簡単な文章
経線5回&逆+緯線3回&逆(w

何通りあるかのほうが面白そう
「正多面体逆向二2筆書き」
390132人目の素数さん:2008/01/02(水) 12:33:04
多面体逆向二筆書き
391132人目の素数さん:2008/01/02(水) 12:48:01
>>389
うーん、簡単すぎたか。もう少し条件を付けたバージョンを考えておく。
392132人目の素数さん:2008/01/02(水) 13:53:06
>>389>>391
>>379では「頂点でUターンしない」という条件を忘れていた。
その条件を付けても可能となる。それでも簡単なら、改めて付帯条件を考える。
393132人目の素数さん:2008/01/02(水) 16:59:03
>>387
>「A^(n-1) の各要素が正である事が必要十分。」
>が簡単に導けていたと思う。
ああ!そうか、確かにn−1乗でOKだわ。
俺のやった方法だと、初めの方も、変形した方も、全く同じ方針で
解けるのだが、グラフ理論からの解法も気になるな。
394132人目の素数さん:2008/01/02(水) 21:10:09
>>388
なるほど。しかしこれでは 2^(1/3) の有理数近似は苦しいな。

x[n]^3+2y[n]^3+4^2*z[n]^3-6x[n]y[n]z[n]=1 は
x[n]^3+2y[n]^3+4z[n]^3-6x[n]y[n]z[n]=1 の誤記

だろうけど、これを2次のペル方程式の真似で

(x[n]+y[n]p+z[n]p^2)(A[n]+B[n]p+C[n]p^2)=1

と因数分解して 「x,y,z がデカイと A[n]+B[n]p+C[n]p^2 ≒ 0」を
利用しようとしても、pの2次方程式を解くことになってしまい、
「2次の無理数による、2^(1/3) の近似値」しか出てこない。
395132人目の素数さん:2008/01/02(水) 21:17:56
>>394
x[n]/y[n]→2^(1/3)
x[n]/z[n]→2^(2/3)
てな風に近似すんだよ
極限とってみ
396132人目の素数さん:2008/01/02(水) 22:26:34
>>395
あそーか。行列
[1,2,2]
[1,1,2]
[1,1,1]
の絶対値最大の固有値の固有ベクトル [2^(2/3),2^(1/3),1] の成分比だな。
なるほどー。
397132人目の素数さん:2008/01/03(木) 00:52:39
>>396
成る程、同様にして m^(1/n) の近似分数列も定まってきそうだな。
高次ペル方程式とは一寸違う方向かも知らんが。
398132人目の素数さん:2008/01/03(木) 07:05:13
行列 A
[1,2,2]
[1,1,2]
[1,1,1]
を、実射影平面 P^2 の射影変換と見ると、
(実の)不動点は [2^(2/3),2^(1/3),1] だけだから、
[1, 1, 1] からでなくとも、どの整数点 [p, q, r] から始めても
A を何回も施すと P^2 の点 [2^(2/3),2^(1/3),1] に収束しそうだな。
2^(2/3),2^(1/3) の同時近似(同一分母による近似)としては効率は良さそうだが。
単一近似としての効率はどうかな。
一般の初期値から出発した場合は、ペル方程式としては N(α) = 定数 の形になるのかな。

同じく、行列 B
[1, n, n, n]
[1, 1, n, n]
[1, 1, 1, n]
[1, 1, 1, 1]
を、実射影空間 P^3 の射影変換と見ると、(実の)不動点は [n^(3/4), n^(2/4), n^(1/4), 1] だけだから、
[1, 1, 1, 1] 等任意の整点を出発点として、 B を何回も施すと、n^(1/4) の近似分数列が得られるだろう。
この場合、ペル方程式は、 Q(n^(1/4)) の整数環の単数群の torsion free part を表すものとなるのだろうが、
その生成元は、 n = 2, 3 の場合でも結構複雑になるのかな。
399132人目の素数さん:2008/01/03(木) 11:54:58
>>398
>(実の)不動点は [2^(2/3),2^(1/3),1] だけだから、(中略)
>A を何回も施すと P^2 の点 [2^(2/3),2^(1/3),1] に収束しそうだな。

不動点がちょうど1つ、というだけでは収束は保証されないよ。
対応する固有値の絶対値が、虚数固有値の絶対値より大きいからOK。

> 同じく、行列 B(中略)を実射影空間 P^3 の射影変換と見ると、
>(実の)不動点は [n^(3/4), n^(2/4), n^(1/4), 1] だけだから、

[ - n^(3/4), n^(2/4), - n^(1/4), 1] も実不動点だよ。でも
[ n^(3/4), n^(2/4), n^(1/4), 1 ] は固有値(の絶対値)が最大なので
収束はこっち。
400132人目の素数さん:2008/01/03(木) 12:11:00
>>398
>単一近似としての効率はどうかな。

効率は悪いよ。平方根を今のと同じアルゴリズムでやった場合を a[n]、
ニュートン法でやった場合を b[n] とし、初項を b[0]=a[1]=1 にすれば
b[n]=a[2^n] になる。

立方根でも、ニュートン法のような2次収束する数列が、部分列になるか
どうかを調べるのも面白そうだ。
401132人目の素数さん:2008/01/03(木) 17:05:37
>>399>>400
tkx
大部感じが分かってきた。有難う。
402132人目の素数さん:2008/01/03(木) 18:37:11
リーグ戦の組合せ順は何通りあるか?
 例)4チーム(ABCD)の場合は6通り(↓の節の順=3!)
    □ABCD
    A□123  第1節がA−B&CーD
    B1□32  第2節がA−C&BーD
    C23□1  第3節がA−D&BーC
    D321□ 
一般解(nチームの場合)を求める問題です。
403132人目の素数さん:2008/01/03(木) 18:47:00
放置推奨
404132人目の素数さん:2008/01/03(木) 19:01:25
>>401
↓ここに高校生向きの問題として3次ペルがあるよ。2番な。
http://93.xmbs.jp/ch.php?ID=checkmath2&c_num=83037
405132人目の素数さん:2008/01/03(木) 19:07:43
>>400
 f(x) = x^2 -2/x = 0
を使えば3次の収束らしいお。 >>275
406Eukie_M_SHIRAISHI:2008/01/03(木) 19:27:32
407405:2008/01/03(木) 19:38:30
a_(n+1) -p = a_n -p -{(a_n)^2 - (p^3)/a_n}/{2a_n + (p^3)/(a_n)^2},
    = (a_n -p)^3 (a_n +p)/{2(a_n)^3 + p^3},
408400:2008/01/03(木) 21:33:05
>>405
本当だ、確かに3次収束だ。はえー
409132人目の素数さん:2008/01/03(木) 23:22:15
>>359が2007/12/25(火) 22:25:17
だから、このスレ年末年始で結構進んだな。
社会人は明日から初仕事だから、ちょっと勢いが鈍るかも。
410132人目の素数さん:2008/01/04(金) 04:55:41
>>394-401
同じように 整数 a,b,c,d に対して p=d^(1/3) (ただし 1,d,d^2 はQ上1次独立) として

x[n] + y[n]*p + z[n]*p^2 = ( a + b*p + c*p^2 )^n

で定まる数列 x[n],y[n],z[n]は、行列 A = aE + bD + cD^2 のn乗 A^n の第1列で与えられる。
ただし Dは
D =
[0,0,d]
[1,0,0]
[0,1,0]
であり Eは3次の単位行列。また Dの固有ベクトルは、ω=exp(i*2π/3) として
Dの固有値 p に対しては t[p^2, p, 1]
Dの固有値 ωp に対しては t[ω^2*p^2, ω*p, 1]
Dの固有値 ω^2p に対しては t[ω*p^2, ω^2*p, 1]

Aの固有値は上のそれぞれに対して
a+bp+cp^2, a+bωp+cω^2p^2, a+bω^2p+cωp^2
になる。従って a,b,c が正なら a+bp+cp^2 が絶対値が最大の固有値だが、たとえば
c=0 で aとbが異符号なら a+bωp+cω^2p^2 や a+bω^2p+cωp^2 の方が絶対値が大きくなる。
このため収束列を作りたいなら a,b,c の選択に制限がある。
411132人目の素数さん:2008/01/04(金) 06:13:30
>>成る程。
412132人目の素数さん:2008/01/04(金) 17:43:27
>>409
数学板は全般的にこの通りだな。
413132人目の素数さん:2008/01/05(土) 00:23:35
http://www6.atwiki.jp/omoshiro2ch/
に代数の問題が少ないので一つ投入しよう。
但し数検過去問の変形、と言ったら大ヒントになるので、
知っている人は別として、過去問は見ないやうに。

a, b, c, d, e, f の実係数 1 次同次式を成分とする 4 次正方行列 A で、
det A = (a^2 + b^2 + c^2 - d^2 - e^2 - f^2)^2
なる物が存在する。
414132人目の素数さん:2008/01/05(土) 10:20:46
>>413
電磁場テンソルF (4次元の二階交代テンソル) の行列表示が
det(F) = ( E_1*B_1 + E_2*B_2 + E_3*B_3 )^2
をみたすな。
E_1=a-d, B_1=a+d, E_2=b-e, B_2=b+e, E_3=c-f, B_3=c+f
とすれば題意をみたす行列の出来上がり。
415132人目の素数さん:2008/01/05(土) 10:51:12
>>382 の (ii)
簡単の為 a > 0 とする。
a = e^(log a)
a^(log a) = e^{(log a)*(log n)} = n^(log a)
よって log a < -1, i.e; a < 1/e で収束、
これが a の収束範囲。
416410:2008/01/05(土) 11:05:43
>>410
× (ただし 1,d,d^2 はQ上1次独立)
○ (ただし 1,p,p^2 はQ上1次独立)
417132人目の素数さん:2008/01/05(土) 14:50:06
>>414
意外と簡単だったか!
418132人目の素数さん:2008/01/05(土) 15:30:21
>>413
が意外と簡単だったので、行列ではなく多項式の代数問題。
n を奇数とし、 f (x, y), g (x, y) を高々 n - 1 次の実係数多項式とする。
この時、連立方程式

x^n = f (x, y)
y^n = g (x, y)

は、少なくとも一つの実数解を持つ。
419132人目の素数さん:2008/01/05(土) 15:50:46
>>418
そっちのほうが簡単な気が…

実係数、奇数次だから
(∀y∈R)(∃x∈R) x^n = f (x, y)
(∀x∈R)(∃y∈R) y^n = g (x, y)
よって(ry
420419:2008/01/05(土) 15:57:07
ごめん。できてなかった。
421132人目の素数さん:2008/01/05(土) 16:49:57
>>399
前半の話だが、虚数不動点は実の回転みたいなものだが、
整点でそんな点はないんじゃないか?
だから任意の整点から出発して収束するとは云えない・・・・?
いや待てよ、やっぱり云えないか。
複素力学系みたいな振る舞いをするのかな?
422132人目の素数さん:2008/01/05(土) 17:17:41
>>382
(1)
a ≧ 1 だと当然収束しないから、
0 ≦ a < 1 とする。
この時適当な定数 C により、
a^(√n) ≦ C/n^2
と評価されるから、その範囲で収束。
423132人目の素数さん:2008/01/05(土) 17:32:24
>>418
F:y∈R→x∈R, x^n = f(x,y)
G:x∈R→y∈R, y^n = g(x,y)
を満たす連続関数F,Gがあって、グラフx=F(y)とy=G(x)がxy平面で交点をもつ。
424132人目の素数さん:2008/01/05(土) 17:44:34
>>423
F, G は一価連続じゃないぞ。連続な物があるという保証はない。
425132人目の素数さん:2008/01/05(土) 18:07:25
遅ればせながら高校生スレに便乗

(1) 11223344の8つの数字を円形に並べるとき並べ方は何通りあるでしょう

(2) 111222333の9つの数字を円形に並べるとき並べ方は何通りあるでしょう


(1)(2)共に回転して一致する並べ方は1通りとする
てかあの問題と同じ仕様で
426132人目の素数さん:2008/01/05(土) 18:13:27
ん?
427132人目の素数さん:2008/01/05(土) 18:14:14
>>425
ポリアの定理で計算出来る。
428132人目の素数さん:2008/01/05(土) 18:52:00
>>379
が出題ミスだったので、もう一度きちんと書いておこう。

正二十面体の一つの頂点を出発して、全ての辺を二度ずつ通り、
元の頂点に戻ってくる経路があることを示せ。ただし、正し各辺は違う向きに一度ずつ通るものとする。
また、辺上を動いていて、頂点に到達した時、今通った辺にすぐ戻る事(Uターン)は禁止する。

正十二面体ではどうか?
429132人目の素数さん:2008/01/05(土) 21:32:15
D を xy-平面内の原点中心の「面積」 1 の閉円板、 f, g をその上の連続関数で、条件 :
任意の p, q ∈ D に対して [ f (p) ≦ f (q) ⇔ g (p) ≦ g (q)] を満たす物とする。
また、関数 g に円板 D の任意の回転を合成した関数を h とする。このとき

(∬_D f (x, y) dxdy)*(∬_D h (x, y) dxdy) ≦ ∬_D f (x, y)*g (x, y) dxdy

を示せ。
430132人目の素数さん:2008/01/05(土) 23:00:51
>429
 ただの 迫随序積 ≦ 箔ッ順序積 (積分版) だって…

http://science6.2ch.net/test/read.cgi/math/1179000000/206-207
不等式スレ3
431132人目の素数さん:2008/01/06(日) 00:07:59
X=cosX Xはいくつ?
432132人目の素数さん:2008/01/06(日) 00:20:41
>>431
で、問題は?
433132人目の素数さん:2008/01/06(日) 02:07:17
で、答えは?
434132人目の素数さん:2008/01/06(日) 02:44:35
>>431
X=0.7390851332151606416553120876738734040134117589…
死ぬほどクダラナイ
435132人目の素数さん:2008/01/06(日) 02:52:44
>>427
ポリア?
436132人目の素数さん:2008/01/06(日) 09:16:28
xyz-空間 R^3 で、x + y + z = a, -x + y + z = b, x - y + z = c, x + y - z = d
が、全て整数になる点は 0 ≦ x, y, z ≦ n の範囲で幾つあるか?
a, b, c, n が奇数の場合は 0 ≦ |x|, |y|, |z| ≦ n の範囲で幾つあるか?

a, b, c, d が整数なる平面族 x + y + z = a, -x + y + z = b, x - y + z = c, x + y - z = d
で、0 ≦ x, y, z ≦ n なる領域は幾つの領域に分割されるか?

a, b, c, d, n が奇数の時、
x + y + z ≦ n, -x + y + z ≦ n, x - y + z ≦ n ,x + y - z ≦ n
なる領域は、平面族 x + y + z = a, -x + y + z = b, x - y + z = c, x + y - z = d
で、幾つの領域に分割されるか?
437132人目の素数さん:2008/01/06(日) 13:13:22
438132人目の素数さん:2008/01/06(日) 15:15:01
>>436で与えられた各点を中心とする半径 r の球体を考える時、
それらが球面以外で共通点を持たない様な最大の r を求めよ
439132人目の素数さん:2008/01/06(日) 17:49:50
>>437
英語は嫌いだぁ
実際に解いてみてよ
440132人目の素数さん:2008/01/06(日) 18:01:45
英語が嫌いでは高等数学は出来ない。
441132人目の素数さん:2008/01/06(日) 19:06:39
>>428
シンプルな問題だったのが台無し

正多面体の辺を二筆書きする方法はそれぞれ何通りあるか?
ただし各辺は違う向きに一度ずつ通るものとする。

のほうが汎用性がある。
頂点発着と辺発着は同じとする条件は必要?蛇足?
442132人目の素数さん:2008/01/06(日) 19:19:11
>>425>>439
では(1) 11223344の8つの数字を円形に並べるとき並べ方は何通りあるでしょう
を解いてみよう。
先ず、回転を考えない並び方の総数は 8!/(2!)^4.
このうち、自明でない回転で一致する物は、
各 n に対し、 n と n が中心対称の位置にある物のみで、 4!.
従って答えは (8!/(2!)^4 - 4!)/8 + 4!/4.
443132人目の素数さん:2008/01/07(月) 07:46:34
各辺の長さがいずれも整数であり、その最大公約数が1である三角形で
1つの内角がθであるような三角形の集合を S[θ] と書くことにする。
S[θ] が空集合でなく、かつ有限個の要素からなるようなθは存在するか。
444132人目の素数さん:2008/01/07(月) 15:09:09
>>425
等間隔とかの条件がないから無限大
線対称は別の条件を入れないと誤解
445Eukie_M_SHIRAISHI:2008/01/07(月) 21:30:59
【賞品付きQuiz】(先着10名まで)


12個の硬貨ある。 そしてその中には贋金が一つ含まれている。

その偽(にせ)の硬貨は残りの本物の硬貨よりも質量が違うことが分かっている。

上皿天秤が与えられている。 その上皿天秤を3回だけ使って、

その偽(にせ)の硬貨を見つけ出し本物よりも重いか軽いかを判定する方法がある

どんな方法か? Web Page を作ってその方法を示せ。

E-mailの宛先は:− [email protected]


Nao kono Mondai ni wa bimyou ni tsugau(違う)fukusuu-ko no Seikai
ga aru !



Good Luck to YOU and to US ALL !
446132人目の素数さん:2008/01/08(火) 05:03:26
>>444
馬鹿無限大
447132人目の素数さん:2008/01/08(火) 15:43:48
>>443
存在しない。二次体の簡単な話題。
448132人目の素数さん:2008/01/10(木) 02:46:04
ここに、赤玉と青玉が入った袋がある。玉を無作為に一個取りだして、
玉の色を見て、それを袋に戻す。この操作を n 回行った時、
赤玉が r 回であるである確率はいくらか?

と言うのが問題であるが、ここで赤玉の出る確率は、
途中 k 回まで操作を行った時に赤玉が j 回であったなら、
次に赤玉が出る確率は (j + 1)/(k + 2) とする。
最初、即ち k = j = 0 の場合は 1/2 である。
449132人目の素数さん:2008/01/10(木) 03:27:41
>>443
>>447での解答が素っ気なかったので、もう少し詳しく述べる。

これは、三辺の長さが有理数で、相似で無い物が一個あれば無限個あるという問題と同値なので、
この形で考える。三辺の長さを X, Y, Z, cos∠C = a とすると余弦定理より、
Z^2 = X^2 + Y^2 - 2aXY. X^2 + Y^2 - 2aXY (a ∈ Q) は有理数体 Q または二次体 K で因数分解出来る。

(i) 有理数体 Q で X^2 + Y^2 - 2aXY = (X - bY)(X - cY) と因数分解された場合。。
相異なる素数 p, q, ...... が無限にあるから、 Z = pq, X - bY = p^2, X - cY =q^2, ...
とすれば、相似でない物が無限に得られる。

(ii) そうでない場合、二次体 K で X^2 + Y^2 - 2aXY = (X - bY)(X - cY).
K の整数環の元 p, q, ...... で (p), (q) が異なる素イデアルになる様な物が無限にある。
Z = pp~ (p~ は p の共軛元)、X - bY = p, X - cY = p~ とすれば得られる。
450132人目の素数さん:2008/01/10(木) 06:00:01
x^2+y^2-axy=1.
y=1+tx.
x=(a-2t)/(1-at+t^2).
y=(1-t^2)/(1-at+t^2).
451132人目の素数さん:2008/01/10(木) 06:29:01
>>450
当然有理曲線ではあるよ。
452132人目の素数さん:2008/01/10(木) 09:00:18
んー
453132人目の素数さん:2008/01/10(木) 16:43:25
n個の数θ_1, θ_2, ... , θ_n が
sinθ_1=θ_2, sinθ_2=θ_3, ... , sinθ_n=θ_1
を満たすとき、θ_nをnを用いて表せ。
454132人目の素数さん:2008/01/10(木) 17:17:56
0
455132人目の素数さん:2008/01/10(木) 22:16:08
ルービックキューブ(3X3X3、面中心色固定)をばらして無作為に組み直すと
完成出来なくなる場合がある。何通りあるか。
456132人目の素数さん:2008/01/11(金) 02:08:18
どこまでどうばらして良いのだ?
457132人目の素数さん:2008/01/11(金) 03:14:06
クォーク・レベルまで分解して、ハニーフラッシュ。
458132人目の素数さん:2008/01/11(金) 23:26:13
世の中、馬鹿が多い様だな。
459132人目の素数さん:2008/01/12(土) 12:05:10
ルービックキューブと言えば、

同形の立方体を27個使って、3×3×3に組み上げ、大きな立方体を作る。
はじめに赤いペンキで、その立方体の表面を塗る。
その後、一旦バラバラにして、まだ塗られていない面が表に出るように、
再び大きな立方体を作る。
次に、緑色で表面を塗る。
その後、同様に一旦組みなおして、まだ塗られていない面を表にして
大きな立方体を作る。
最後に、表面を黄色いペンキで塗る。

ここまでの作業で、ひとつひとつの立方体に着目したとき、
6面が2種類の色で塗られているものは何個あるか ( あるいは
その個数は定まるだろうか )。

って問題を思い出した。
なんか発展した問題ない?
460132人目の素数さん:2008/01/12(土) 15:58:34
ばらして組みなおしてもまたばらして組みなおせるから完成できなくならない。
461132人目の素数さん:2008/01/12(土) 16:22:07
>>455
直感的には
八つの角について位置の違いが2通り。向きの違いが3通り。
辺について位置の違いが2通り。向きが2通り。

2×3×2×2=24通り。


違うか…?
462132人目の素数さん:2008/01/12(土) 19:38:58
全ての部品が正解位置にある場合だけでも3^8*2^12-1通りはある。
463132人目の素数さん:2008/01/12(土) 21:58:17
9*AB*CDE=ABCDE
464132人目の素数さん:2008/01/12(土) 22:21:14
>>463
その種の問題はもう飽きた
465132人目の素数さん:2008/01/12(土) 22:27:23
どんな問題が「面白いか」は人さまざまだろうが…
このスレの他の問題のレベルを見てから投稿しても遅くはないだろう。
466455:2008/01/12(土) 22:52:59
予想通り茶化された(まじ予想では無視スルーだったから感謝)
コマネチ大の16パズルの発展系として考えたんだが
ルービックキューブを壊した事がない人には問題自体理解するのも無理か?
467132人目の素数さん:2008/01/12(土) 23:10:07
壊した事は無いが、「壊れた」事ならあるよ。
468132人目の素数さん:2008/01/12(土) 23:53:28
俺はメタメタにシールを貼りかえたことがある
4歳とか5歳とかそれくらいのときに
469132人目の素数さん:2008/01/13(日) 00:30:14
>>468
そりゃばったもの
470132人目の素数さん:2008/01/13(日) 00:31:23
もうかなり昔、中1の春にルービックキューブを買った。中の構造が気に
なったんだけど、でも壊すのは勿体無くて、しょうがないから、キューブと
キューブの間の僅かなスキマから中を覗いて、その構造を想像してた。
半年くらい考えて、中1の秋のころ、構造を解明した。友達の持っていた
キーホルダーサイズのルービックキューブを分解してもらい(結局、自分のは
分解せずw)実際の構造を見てみたら、バッチリ合ってた。
471132人目の素数さん:2008/01/13(日) 00:51:56
>>466
どんな状態からでも1段と半分までは揃うんだな。
あとは3軸周りの移動回数で場合分けすればいけそうだが面倒だな。
472132人目の素数さん:2008/01/13(日) 01:44:33
正方形のみで作る立方体の展開図の個数を数学的に解くとか、ってある?
473132人目の素数さん:2008/01/13(日) 02:33:09
>>472
無限個
474132人目の素数さん:2008/01/13(日) 02:49:47
正方形のみで作る立方体の意味が分からん
475132人目の素数さん:2008/01/13(日) 03:29:05
問題
無理数の無理数乗が有理数になる事はあるか?
476132人目の素数さん:2008/01/13(日) 03:43:34
e^(ln 2)
477132人目の素数さん:2008/01/13(日) 10:38:57
a^b、b^aがいずれも有理数となる無理数a、bは存在するか
478132人目の素数さん:2008/01/13(日) 11:07:31
>>466
ばらしたあとどう組みなおせるのかもわからん。
対象の位置にあるのは全部入れ替え可能なのか?
479132人目の素数さん:2008/01/13(日) 11:36:18
>>477
x^x = 3 なる実数 x が存在するから、
これが無理数なる事も容易に分かり、
a = b = x とする。
480475:2008/01/13(日) 12:31:31
一応用意しておいた解答を書いとくと

まず√2は無理数である
従って(√2)^(√2)が有理数ならば、主張が成り立つ
(√2)^(√2)が無理数ならば((√2)^(√2))^(√2)=2 なので
やはり主張が言える

というのがあった
481132人目の素数さん:2008/01/13(日) 12:43:52
>>480
(√2)^((√2)^(√2)) は、どうして有理数なの?
482132人目の素数さん:2008/01/13(日) 13:03:29
>>481
475は(√2)^((√2)^(√2))という数には触れてないぞ。((√2)^(√2))^(√2)でしょ。
483132人目の素数さん:2008/01/13(日) 13:44:15
あ、いや、わかった。>>477と勘違いした。
484132人目の素数さん:2008/01/13(日) 14:46:38
>>478
もちろん!
原型に直したら数学の問題にならない
機械組立工作の問題になる
485132人目の素数さん:2008/01/14(月) 13:30:20
1から6までの数字を1回ずつと+、−、×、÷、括弧のいずれかを使った式を考えます
それを例えば6×(1+25)÷3−4とします
この式を逆から読むと4−3÷(52+1)×6となります
この2式の差は48−3.66037……≒44.33963となります
この様に逆から見た値と元の式との差が、
0にはならず、できる限り0に近くなる様な式を見つけて下さい

[ルール]
(1)1から6までの数字を全て1回ずつ使う.これ以外の数字の使用は不可
(2)156の様に数字をくっつけてもよい
(3)+、−、×、÷、括弧はそれぞれ何回使ってもよいし、使わないものがあってもよい
それ以外の演算子の使用は不可
(4)4の斜め上に3を書いて4の3乗にする等は不可
486132人目の素数さん:2008/01/14(月) 14:19:18
>>485
ルールをつけたす必要があるぞ
1+2+3+4+5+6
6+5+4+3+2+1
差は0
487132人目の素数さん:2008/01/14(月) 16:17:30
>>486
「0にはならず」だよ
488132人目の素数さん:2008/01/14(月) 16:55:06
>>485
数学的な魅力に乏しい気がする。

条件を満たす最良の式を求めたりチェックしたりするには、総当りしかないだろうから。
489132人目の素数さん:2008/01/14(月) 17:37:33
ルールを厳密に定義したければ追加要
負の数を表す-は不可
()は逆使用

例 1+(-2x3)+4+5+6⇔6+5+4+)3+2-(+1
意味不明
490132人目の素数さん:2008/01/14(月) 19:01:38
156/234 -> 625/90000
491132人目の素数さん:2008/01/14(月) 20:47:44
>480
何が言いたいのかよくわからないけど
(√2)^(√2) は無理数だよ
492132人目の素数さん:2008/01/14(月) 20:51:19
>>491
元の問題良く読め。
(√2)^(√2) が有理数であっても無理数であっても、どちらを仮定しても、
元の問題が高校レベルで解けるという事だ。
493132人目の素数さん:2008/01/14(月) 20:52:04
>>491
高校生向きの解答なら>>480じゃないと無理だろ
高校生には(√2)^√2が無理数であることの証明はできない
494132人目の素数さん:2008/01/14(月) 20:56:32
ああ、なるほど よく読んでなかった
すまぬ

うまく考えたものね
495132人目の素数さん:2008/01/14(月) 21:11:48
nを2以上の自然数とするとき
√1+√2+…+√n が無理数であることを示せ
496132人目の素数さん:2008/01/15(火) 00:21:31
>>485
少しだけやってみた
5÷6+24÷31=1.6074・・・

13÷42+6÷5=1.5095・・・

上−下≒0.0979

ただこれが最小な自信も根拠も何も無い
問題というかチャレンジだねコレ
497132人目の素数さん:2008/01/15(火) 00:37:23
>>490が0.003くらいだな
498132人目の素数さん:2008/01/15(火) 01:20:30
156/432-234/651≒0.001664
プログラム書いてこのタイプの商の差を求めたらこれが最小だった
499132人目の素数さん:2008/01/15(火) 01:21:17
プログラム書くなら全部やればいいのにw
500132人目の素数さん:2008/01/15(火) 01:26:02
演算子や括弧まで網羅するのは俺には無理
3桁の商の差は網羅した
501132人目の素数さん:2008/01/15(火) 01:46:49
逆ポーランド記法使えば括弧使わなくて済むから
プログラミングが楽になるよ。

でも計算量がなぁ…
502132人目の素数さん:2008/01/15(火) 02:14:45
プログラムをちょっと変更して↓のタイプのを試した
6÷5132÷4-4÷2315÷6=0.00000431
503132人目の素数さん:2008/01/15(火) 02:18:11
0でない有理数というと結局は割り算しかないのだからこの辺で限界じゃまいか
504132人目の素数さん:2008/01/15(火) 02:39:56
1.05の4分の1乗はどうやって解くのですか?
505132人目の素数さん:2008/01/15(火) 02:40:27
>>504
「解く」とは?
506132人目の素数さん:2008/01/15(火) 02:44:45
どのように計算したら良いですか?
507132人目の素数さん:2008/01/15(火) 02:47:52
平方根の筆算ができるのなら、それを2回やるといいよ
508132人目の素数さん:2008/01/15(火) 03:02:15
ごめんなさい。
よくわかりません…
509132人目の素数さん:2008/01/15(火) 03:07:58
平方根の筆算は検索すればすぐ見つかる。
それができないなら、電卓を使うしかないな。

それから、その問題はこのスレじゃない方がいいだろうね。
510132人目の素数さん:2008/01/15(火) 16:52:45
23:MASUDA◆5cS5qOgH3M :2008/01/14(月) 21:46:37
以下の条件をみたす2008個の異なる正整数a[1],a[2],…,a[2008]が存在することを示せ.
条件:『1≦i<j≦2008をみたすすべての整数の組(i,j)において,(a[j]/a[i])-1がa[i]-1とa[j]-1の最大公約数になる』
511132人目の素数さん:2008/01/16(水) 23:01:32
>510
p≧2, b[k]≧2 として
 c[n] = Π[k=1,n] b[k],
 a[n] = p^c[n]
とおく。
 a[j]/a[i] -1 = p^(c[j]-c[i]) -1 = p^{c[i](c[j]/c[i] -1)} -1 = a[i]^(c[j]/c[i] -1) -1,
 c[j]/c[i] -1 ≧ 1 だから a[i] -1 の倍数。

 a[j] -1 = (a[i] -1)(a[j]/a[i]) + a[j]/a[i] -1,
より
 gcd(a[i] -1, a[j] -1) = gcd(a[i] -1, (a[j]/a[i]) -1) = (a[j]/a[i]) -1.

http://science6.2ch.net/test/read.cgi/math/1199706844/23-29
東大入試作問者スレ13
512132人目の素数さん:2008/01/17(木) 02:34:57
>>511
その答えに反例が挙がってるようだが
513132人目の素数さん:2008/01/17(木) 16:07:40
Nを任意の自然数とし、F(n)をフィボナッチ数列とする
F(n)がNで割り切れるような自然数n全体を決定せよ
514132人目の素数さん:2008/01/17(木) 17:16:18
>>513
http://www004.upp.so-net.ne.jp/s_honma/ohkawa/ohkawa.htm#118
より、フィボナッチ数 F(n) はmod. N 周期を持つが、
それすら分からない難問であるから、
もっと易しくしてちょ。
なお上記より、 N で割り切れるフィボナッチ数が必ず存在することは分かる。
515513:2008/01/17(木) 17:53:43
>>514
n|m ならば F(n)|F(m) であることに注意する
F(n) ≡ 0 (mod M) となる最小の n を n(M) とする
N = Π^{m}_{k=1} p^{a_k}_k と素因数分解されるとする
このとき Chinese Remainder Theorem より次が成り立つ
n(N) = LCM( n(p^{a_1)_{1}) , ... , n(p^{a_m)_{m}) )
従って求める自然数全体は n(N) の倍数全体となる
以上より、素数 p と自然数 a に対して n(p^a) を具体的に求めれば良い

n(p) を具体的に求めるのは大変なので、n(p^a) を n(p) を用いて表せ
516132人目の素数さん:2008/01/17(木) 20:39:05
>>515
>n(p) を具体的に求めるのは大変なので、n(p^a) を n(p) を用いて表せ
良く知られた予想はあるが、誰も証明した物はいない。
517132人目の素数さん:2008/01/17(木) 21:00:41
518132人目の素数さん:2008/01/17(木) 21:16:16
なんだよ転載かよ
519132人目の素数さん:2008/01/17(木) 21:43:10
>>441
蛇足
520132人目の素数さん:2008/01/17(木) 21:56:54
521513:2008/01/17(木) 22:58:17
>>516
用意していた解答にミスを見つけたorz
てか、有名な問題だったのね。知らんかった

お騒がせしました
522132人目の素数さん:2008/01/19(土) 23:02:13
平面ではなくて空間に世界地図があるとする。
各国は空間領域とする。各国に色を塗って塗り分けたい。
但し、点や線で接している国は同じ色にしても良いが、
面で接する国は違う色で塗り分けねばならない。最低何色必要か?・・・・

と言いたいが、これでは何色でも必要になってしまう。
それでは条件を付けて、国は全て有界凸閉多面体で、
国の数も有限とするとどうか?
523132人目の素数さん:2008/01/20(日) 00:14:08
小学校の時、先生が遊びで出した問題です。

マラソンランナーは42キロの距離をを、1時間で残りの距離の1/2進めるとします。
いつまで行ってもその残りの1/2が残りますが、数は無限大にあるので、
ランナーは永久にゴールできないのか?

確かゴールできるという答えだったと思うのですが、
理系の大学に入った今でもわかりませんw
ふと思い出したので、答えを教えてください。
524132人目の素数さん:2008/01/20(日) 00:24:59
高校の内容を使ってもいいなら↓

スタート地点をx、ゴール地点をy、ランナーをaとする。
このとき、aはyに限りなく近づく。
よって、a=y
つまり、aとyの位置が同じとなり、ランナーはゴールできる。
525132人目の素数さん:2008/01/20(日) 00:42:22
n時間後のランナーとゴールの距離をd[n]とすれば
d[n]=42*(1/2)^n>0
であるから、決してたどり着けない
526132人目の素数さん:2008/01/20(日) 01:00:39
足元がゴールラインすれすれになった頃に胸を突き出せばゴールイン扱いになる
そういう問題ではないのか
527132人目の素数さん:2008/01/20(日) 01:19:57
実数列A[n]で、以下の条件を満たすものが存在するか?
存在するなら具体例を、存在しないならその証明を示せ
1、lim[n→∞] A[n] = +∞
2、lim[n→∞] A[2^n]/A[n] が存在
528132人目の素数さん:2008/01/20(日) 02:08:05
>>527
数列{fn}を次のように定める。

f1=2 , f(n+1)=2^fn

数列{fn}は明らかに狭義単調増加である。g:[2,+∞)→Rを

g(x)=k ( fk≦x<f(k+1) )

として定め、数列A[n]をA[n]=g(n)で定義する。このA[n]は、
1と2をともに満たす。
529132人目の素数さん:2008/01/20(日) 02:22:56
>>528
別の問題になってしまうが、その fn をn(>0)について実解析的に拡張できないものだろうか。
何らかの意味で自然に。
530132人目の素数さん:2008/01/20(日) 07:02:39
f(x)=x^a,a=x^a とする.

(1)x≧1として,f(x)が収束するようなxの範囲を求めよ.
(2)x<1の場合を論ぜよ.
531132人目の素数さん:2008/01/20(日) 07:52:35
>>530
「収束する」という単語は、数列にしか使わない。
「数列{1/n}は0に収束する」という使い方しかしない。
1つの実数を持ってきて、それが何かに収束する、という言い回しはしない。
たとえば、「実数1は1に収束する」という言い回しはしない。

>x≧1として,f(x)が収束するようなxの範囲を求めよ.
f(x)は数列では無い。各xごとに、f(x)は実数を表しているから、「f(x)が収束するような」
という言い回しはしない。これに対し、たとえば、関数列{fn(x)}が与えられたとき、
「関数列{fn(x)}が収束するようなxの範囲を求めよ」という言い回しは正しい。
532132人目の素数さん:2008/01/20(日) 07:56:07
収束フォーーー
数列フォーーー
実数フォーーー
533132人目の素数さん:2008/01/20(日) 09:33:29
>>530
x^a = a なんだから、 f (x) = x^a = a は定数
534132人目の素数さん:2008/01/20(日) 09:38:30
10^30 を 1002 で割った数の 10 の位を求めよ。
535530:2008/01/20(日) 11:22:45
>>531
訂正どうも。だめだめでゴミン
>>533
確かに・・。
f(x)=x^a ←このaにa=x^aを代入を繰り返して
f(x)=x^(x^(x^(x^(x^・・・))))
ってさせたかったんだけど、これじゃ結果が一意にならない?(のかな?)
混乱してきたので問題訂正しますorz

【問】
x^(x^(x^(x^(x^・・・))))=α (α∈R)
となるようなxの値の範囲を求めよ.
536132人目の素数さん:2008/01/20(日) 11:27:48
>>535
f(1)=x
f(n)=x^(f(n-1))
lim [k->∞] f(k) = α

と理解してよいか?
537132人目の素数さん:2008/01/20(日) 12:01:11
>>536
なるほどそれなら>>533で指摘された不備が出ないですね。
問題無いと思います
538132人目の素数さん:2008/01/20(日) 12:06:07
>>537
-1<x<=1
539132人目の素数さん:2008/01/20(日) 12:47:19
540132人目の素数さん:2008/01/20(日) 13:09:29
巡回するとなると厄介だな
541132人目の素数さん:2008/01/20(日) 13:10:36
>>539
鼻血出た
542132人目の素数さん:2008/01/20(日) 13:20:36
1≦x≦exp(1/e)
と思ったら
exp(-e)≦x≦exp(1/e) までいけるのか
543132人目の素数さん:2008/01/20(日) 14:04:44
>>542
学部二年の時にそれ導いたわ
有界で単調な数列は収束する、が本質だった
複素数の範囲は難しくて諦めた
544132人目の素数さん:2008/01/20(日) 14:05:09
>>536
そいつを f と書いてしまうと、最初の出題の 「 f(x) 」 という書き方ができなくなってしまう。
g(x,1):=x
g(x,n):=x^( g(x,n-1) )
f(x):=lim [n→∞] g(x,n)
にしとこうぜ。

なおこのf は f^(-1) が簡単な式で書けるな。
545132人目の素数さん:2008/01/20(日) 14:07:20
>>543
> 複素数の範囲は難しくて諦めた

y=f(x) は
x=y^(1/y) をみたすから、これを用いて解析接続することになるね。
546529:2008/01/20(日) 14:12:26
オレは >>544 の g(x,n) を、
xについてではなく nについて解析的な拡張をしたい。
g(a,z+1)=g^( g(a,z) ) が成立し、 zが実数を動くときはzについて対数凸な、
そんな拡張はできませんか。
547132人目の素数さん:2008/01/20(日) 16:59:56
全部数字を入れるクロスワードです。入れる数字は全て平方数(何とかの2乗)です。同じ数は2度使っては駄目です。
 
■□■■□□
□□□■□■
□■□□□□
□□□□■□
■□■□□□
□□■■□■
548132人目の素数さん:2008/01/20(日) 17:04:08
>>547
それが埋まると、何か面白い数学的事実が証明できたりするのか?
549132人目の素数さん:2008/01/20(日) 17:44:07
>>547
平方数の「数字」は 0, 1, 4, 9 の四つだから重複を許さねば埋まる訳がない。
550132人目の素数さん:2008/01/20(日) 17:47:28
>>549
バカなの?
551132人目の素数さん:2008/01/20(日) 17:53:24
>>550
お前がな
552132人目の素数さん:2008/01/20(日) 17:58:17
〔527の変形〕
以下の条件を満たす実数列 A[n] の具体例を示せ。
1. lim[n→∞) A[n] = +∞
2. lim[n→∞) A[2^n]/A[n] = α,    (α>0)
553132人目の素数さん:2008/01/20(日) 18:02:33
>552
 A[n] = k*α^g(n),      (k>0, α≧1)
ただし、g(x) は >>528 に定義されているもの。
554546:2008/01/20(日) 18:11:12
× g(a,z+1)=g^( g(a,z) ) が成立し、
○ g(a,z+1)=a^( g(a,z) ) が成立し、
555132人目の素数さん:2008/01/20(日) 18:21:17
>>547
ほう、最近はこういうのも「クロスワード」というのか。
ところで君の言う「クロスワード」の定義を教えてくれないか?
556132人目の素数さん:2008/01/20(日) 18:48:30
>>552
α<1だと困るがな
557132人目の素数さん:2008/01/20(日) 18:58:05
558552:2008/01/20(日) 20:54:37
>556
 α≧1 だった … スマソ.
559132人目の素数さん:2008/01/20(日) 21:30:21
>>557
>>529の疑問には答えていない様だな。
これはアーベルの問題として昔から有名な問題だ。
幾つかの解も知られている。
560132人目の素数さん:2008/01/20(日) 23:26:53
>>559
kwsk
561132人目の素数さん:2008/01/21(月) 01:12:25
>>560
色々検索しても出てこなかったが、
サーティ編「現代の数学 III」(岩波)第一章に出ていた。
君が良く行く図書館にこれがあることを祈る。
562132人目の素数さん:2008/01/21(月) 01:17:23
ありがとう。 >>561
英語でいいから、もっといい検索キーはないもんかな。
abel
interpolatoin
power
563132人目の素数さん:2008/01/21(月) 13:55:30
俺が今考えてる問題
おまいらにも考えて欲しい

n,mを自然数とする.1からnまでの数字が書かれたカードがそれぞれm枚ずつ,
合計nm枚ある.これらをm枚ずつn組に無作為に分けるとき,それぞれの組から
うまく1枚ずつのカードを拾って1からnまでの数字を集めることができるか.
564132人目の素数さん:2008/01/21(月) 14:02:09
>>563
その問題の本質的な部分だけを取り出した問題を、
俺が過去ログに書いたことがある。解答もされた。
565132人目の素数さん:2008/01/21(月) 14:45:10
>>564
むうガイシュツだったか
すまん指摘ありがとう
566132人目の素数さん:2008/01/21(月) 15:11:04
>>560
やっと見つかった。
http://en.wikipedia.org/wiki/Functional_equation
に出てくる
f (h(x)) = f (x) + 1 で h (x) = e^x の場合。
解の具体例まではここには書いてない。
567132人目の素数さん:2008/01/21(月) 15:12:08
568132人目の素数さん:2008/01/21(月) 16:16:23
ありがとう!! >>566
analytic solution ( "abel functional equation" OR "abel equation")
で検索したら、面白そうなのがザクザク出てきた。
569132人目の素数さん:2008/01/21(月) 17:07:37
>>568
そんなに沢山出て来たんだったら、分かり易くて面白いの、一つ教えて!
570132人目の素数さん:2008/01/21(月) 21:22:30
方眼紙で,縦2マス,横nマスの長方形を考える.
この長方形の1つの角の点をS,Sからもっとも遠い点をGとする.
Sを出発してGに至る経路のうち,同じ点を2度通らないものの個数をa_nとする.
a_nを求めよ.
571132人目の素数さん:2008/01/24(木) 11:02:05
>>570
n×2の方眼紙内において
(0,0)を出発して(n,1), (n,0)に行き着く道順の総数をそれぞれb_n, c_nとおく
a_nのうち(n,0)→(n,1)の順路を含むものの数をa'_nとする
c_nのうち(n,2)→(n,1)の順路を含む物の数をc'_nとする
a_0 = a'_0 = b_0 = c_0 = 1, c'_0 = 0 となる
漸化式を図から考えて
a_n = a_(n-1) + b_(n-1) + c_(n-1) + a'_(n-1)
b_n = a_(n-1) + b_(n-1) + c_(n-1)
c_n = a_(n-1) + b_(n-1) + c_(n-1)  + c'_(n-1)
a'_n = c_(n-1) + a'_(n-1)
c'_n = a_(n-1) + c'_(n-1)

…もっときれいな解法あるんだろうな
572132人目の素数さん:2008/01/29(火) 23:21:46
(m+1)×(n+1) マスの升目がある。一番左側の列と一番下の行には既に碁石が置かれている。
(合計 n*m + 1 個)
残りの n×m の空白の升目に一つずつ碁石を置いて行って全部埋める方法は何通りあるか?
但し碁石を一つ置くには、そのすぐ左とすぐ下には既に碁石が置かれていなければならない。
(結構有名問題?)
573132人目の素数さん:2008/01/29(火) 23:40:43
株価2000円の会社があるとして、
ボラティリティ40%の場合、
株価が1000円を一回でも下回るのは、
5年間の場合、何%ありますか?
574132人目の素数さん:2008/01/29(火) 23:45:56
>>537
ボラレたくないからあっち行け
575132人目の素数さん:2008/02/02(土) 00:11:23
数時間かけて作ってみた。興味あればやってみてな。答えから先に作ったので矛盾はないと思うが・・・。

二次の係数が1、一次の係数と定数項が整数の二次方程式がある。
その解のうち小さい方は-1より小さく、大きい方は1より小さいという。
AおよびBの二人がこれを解こうとしたが、うち一人は誤って、一次の係数と定数項を入れかえて書いてしまった。
その結果、解の差はAのほうが小さくなった。

正しい二次方程式を示せ。また、それを解いたのはAとBのどちらか。
576132人目の素数さん:2008/02/02(土) 01:17:50
1,1,5,8を使って10って作れますか?
教えてくださいm(__)m
577132人目の素数さん:2008/02/02(土) 01:19:33
>>576
8/(1-1/5)
578132人目の素数さん:2008/02/02(土) 01:33:15
>>575
取り違えた方はもはや
>その解のうち小さい方は-1より小さく、大きい方は1より小さいという。
を満たさなくても良いんだな?
579132人目の素数さん:2008/02/02(土) 01:50:14
1.8*5+1
580132人目の素数さん:2008/02/02(土) 02:45:51
5*{√(8+1)-1}
581132人目の素数さん:2008/02/02(土) 15:01:41
>>578
おk。これは正しい二次方程式についての解の性質。

・・・っていうかすまん!よく見たら問題に重大な欠陥があった。
作り直してくるから>>575は無かったことにしてくれ!
582575の改訂版:2008/02/02(土) 16:17:18
二次の係数が1、一次の係数と定数項がいずれも絶対値が一桁の整数という二次方程式がある。
その異なる二解のうち小さい方は-2より大きく、大きい方は2より大きいという。
AおよびBの二人がこれを解こうとしたが、うち一人は誤って、一次の係数と定数項を入れかえて書いてしまった。
その結果、解の差はAのほうが小さくなった。
正しい二次方程式の候補をすべて示せ。また、それを解いたのはAとBのどちらか。

今度こそ間違いはないはず・・・問題作成って想像以上に気を遣うなあ。
583132人目の素数さん:2008/02/02(土) 17:09:51
二次方程式の解の大きい方、小さい方という言い方をする以上は
重解の場合(大きい方や小さい方というの解はない)や
解が非実数になる場合(そもそも大小関係がない)は
成り立たないものだと考えてよいのだな?
584132人目の素数さん:2008/02/02(土) 19:03:14
>>583
その通り。「〜その異なる二実数解〜」とでも書けばベターだったかな。
585132人目の素数さん:2008/02/02(土) 20:04:09
X_n = { 2^a + 2^b + 2^c | a, b, c は 1 ≦ a, b, c ≦ n なる自然数 } の濃度を求めよ。
586132人目の素数さん:2008/02/02(土) 21:43:20
>>585
20%くらいじゃないの?
587132人目の素数さん:2008/02/02(土) 21:45:31
集合の濃度ってそういう意味じゃないだろ…

a<b<cのときは、2進法表示の一意性により、全部異なる。
あとは、それ以外の場合でどれだけ重複があるかを調べる。
面倒くさいので他の人に任せた!(^o^)
588132人目の素数さん:2008/02/03(日) 04:04:37
>>586
4年前にタイムスリップしたのかとオモタじゃないか。
589132人目の素数さん:2008/02/04(月) 23:49:40
>>582
>をすべて示せ。
では面白い問題とは云えない。そんな問題なら誰でも作れる。
唯一つ解が有ってこそ面白い。
590132人目の素数さん:2008/02/05(火) 00:05:09
>>589
そんなことは百も承知さ。だから初めは>>575のようにしていた。
でもこれは欠陥問題だったから涙を呑んで改変したわけだ。

「間違えて○○してしまった」系の問題って面白いだろ?あとは矛盾無く問題にできるかどうか、だ。
だから君に期待する。「間違えて○○してしまった」系の問題を作っておくれよ・・・。
591132人目の素数さん:2008/02/05(火) 00:07:28
>>589
構造を調べたりするのも面白いと思う
まぁ>>582の設問は確かにつまらないが
592132人目の素数さん:2008/02/05(火) 05:26:11
100以上あるのを全て書くのは面倒
593132人目の素数さん:2008/02/05(火) 11:33:51
いったい何が100以上もあるのだろう?
594132人目の素数さん:2008/02/05(火) 22:08:05
実数 x_1, x_2, ........... , x_n が全て正なる為の必要十分条件は、
それらの基本対称式の値が全て正なる事である。
595132人目の素数さん:2008/02/05(火) 23:53:17
http://yutori.2ch.net/test/read.cgi/news4vip/1201948696/

問題文がちょっと悪いと思っても気のせいです。
596132人目の素数さん:2008/02/06(水) 00:12:05
それ落ちてない?
597132人目の素数さん:2008/02/06(水) 00:15:09
うん。未解決なんだ。困ってる。
598132人目の素数さん:2008/02/06(水) 01:19:30
糞スレ立ててまで
答え出したのじゃないのか?

11 :132人目の素数さん:2008/02/05(火) 23:04:52
・・・でも解いてくれます?多分オレ答え出したんだけど。
出題者いなくなっちゃったし。数学板の人なら、と思ったんだけど
先にオレの解き方言っちゃたら面白くないだろから言いづらいですけど

IQ160以上の超難問題解けるやついる?
http://science6.2ch.net/test/read.cgi/math/1202217368/11
599132人目の素数さん:2008/02/06(水) 01:20:55
600132人目の素数さん:2008/02/06(水) 03:54:15
>594
必要性は明らかなので十分性を示す。
基本対称式 S_1, S_2, ……, S_(n-1), S_n がすべて正とする。
 f(x) = (x+x_1)(x+x_2)(x+x_3)…(x+x_n) = x^n + S_1・x^(n-1) + S_2・x^(n-2) + …… + S_(n-1)x + S_n,
とおくと
 x≧0 ⇒ f(x) ≧ S_n >0,
根 -x_1, -x_2, ……, -x_n はすべて負。

数セミの「エレガントな解答をもとむ」に出てた…
601132人目の素数さん:2008/02/06(水) 10:30:25
具体的な問題でなくてすまん。
ここの住人なら何か情報を知っているんじゃないかと思って
あえてスレ違いを承知で尋ねたい。

平面上の位相幾何の問題なのだけど、
幾つかの点とそれを任意に結ぶ辺からなる図形が
辺が交差しないように変形(点の移動)できるか否か
についての研究などの情報をお持ちでないでしょうか?


・正方形の頂点(4点)を互いに全て結んだもの(辺は6本)はそのままでは対角線で交差しているが
 正三角形と重心の位置に点を移動させれば辺は交差しないように変形可能。

・正五角形の頂点を互いに結んだものは、交差しないように変形は不可能(?)
 しかしどこか一箇所を切ったものならば可能。

・一般にどういう条件なら交差しないように変形が可能なのか?
602132人目の素数さん:2008/02/06(水) 10:54:19
>>601
その研究についての情報は持っていないが、
>正五角形の頂点を互いに結んだものは、交差しないように変形は不可能(?)
これは実際に不可能。平面上の一般の位置(=どの3点も一直線上に無い)に
5つの点を取ると、その中のある4点は凸四角形を作る(ハッピーエンド問題)。
凸四角形があるならば、交差する2辺が存在するのは明らか。
以下、凸四角形が存在することの証明。

5つの点の凸包を考え、場合分けする。
凸包が五角形のとき:どの4点を選んでも、凸四角形になっている。
凸包が四角形のとき:凸包自身が凸四角形である。
凸包が三角形のとき:三角形の頂点をA,B,Cとする。残りの2点をD,Eとすれば、D,Eは△ABCの内部に
存在する。図を描けば分かるとおり、A,B,Cの3点から適当な2点X,Yを選べば、4点X,Y,D,Eが凸四角形になる。
603132人目の素数さん:2008/02/06(水) 11:07:04
>>594>>600
昔どこかのスレで見た
>>601
平面グラフ
604132人目の素数さん:2008/02/06(水) 12:19:45
>>601
>・一般にどういう条件なら交差しないように変形が可能なのか?

グラフが K5 あるいは K3,3 に位相同形な部分グラフを
含まなければ変形可能。
605132人目の素数さん:2008/02/06(水) 14:11:18
K5ってのは先に出た5角形のようだが
K3,3 ってどんな図形?
606132人目の素数さん:2008/02/06(水) 15:12:26
607132人目の素数さん:2008/02/06(水) 16:52:23
>>600
thx 確かにエレガントな解答だ。
http://www.iis.it-hiroshima.ac.jp/~ohkawa/math/math_problem_all.htm#115
に、微分を使った解答が載っていたのだが、そんな簡単に証明できるとは気がつかなかった。
608570:2008/02/06(水) 20:29:47
亀ですが
>>571
一解答例としてはその方針でおk
a'_nとc'_nについてはもう少し考察が必要
609132人目の素数さん:2008/02/07(木) 18:28:04
>>595の問題

>1 :以下、名無しにかわりましてVIPがお送りします。:2008/02/02(土) 19:38:16.48 ID:8Qf0u1wDO
>幽霊が次のような問題を出した
>800ml入る容器が8つあり1〜8まで番号がついていて 今それぞれ100mlずつ水が入っている
>何個かの容器の水には お前らには見えないし触れない魚の幽霊が 容器の番号と同じ匹数だけ入っている
>それ以外の容器には水しか入っていない
>何らかの理由で容器が空になった場合は すぐに100mlの水と最初に魚の幽霊が入っていた場合は魚の幽霊を 容器の番号と同じだけ入れる
>大きな水槽があるがこれに容器の水を移した時 移った魚の幽霊の数を教えてやる
>その際水槽に移す時使用した容器は没収する
>魚の幽霊の数はたまに適当な数を教えるがその後3回は正しい数を教えてやる
>最初にどの容器に魚の幽霊が入っていたか確実にわかるためには 何回容器を水槽に移せばいいか またどのようにすればよいか

>>610のようにすれば容器に移す回数は5回以内である事が分かる
4回以内だと無理かどうかはまた今度考える
610132人目の素数さん:2008/02/07(木) 18:28:14
番号nがついた容器に最初に入っていた水を水nと書くとする
ある容器に水1が100ml、水2が200ml、水4が400ml入っているような状態を(122444)と書くとする
まず
(1)(2)(4)(8)→(18)(24)(48)(8)→(18)(24)(48)(1244888)→(18)(1248)(124488)(1244888)
と容器内の水を移し変えれば水1,2,4,8のうちどれに幽霊が入ってるかを3回確かめられる
(3)(6)(5)(7)→(37)(65)(57)(7)→(37)(65)(57)(3655777)→(37)(3657)(365577)(3655777)
と移し変えれば水3,5,6,7についても同様に3回確かめられる

水1,2,4,8について確かめる動作を2回行い、同じ結果が得られれば
水3,5,6,7について確かめる動作を3回行う。
違う結果が得られた場合はもう一度水1,2,4,8について確かめ、
その後に水3,5,6,7について確かめる動作を2回行う。
これで水1-8のうちどれに幽霊が入っていたかが確実に分かる。
611132人目の素数さん:2008/02/07(木) 18:50:42
なにを言ってるんだかさっぱりわからん。
このあたりは間違いなく意図通りに正しく書かれているのか?

> ある容器に水1が100ml、水2が200ml、水4が400ml入っているような状態を(122444)と書くとする
まず
> (1)(2)(4)(8)→(18)(24)(48)(8)→(18)(24)(48)(1244888)→(18)(1248)(124488)(1244888)
612132人目の素数さん:2008/02/07(木) 19:11:06
>>611
まず最初に容器n(n=1〜8)に水nが100mlずつ入っている
この状態は(1)(2)(3)(4)(5)(6)(7)(8)と書ける
次に容器8の水を容器1に入れる
状態は(18)(2)(3)(4)(5)(6)(7)()となる
幽霊が容器8に水8を100ml補充してくれるから状態は(18)(2)(3)(4)(5)(6)(7)(8)となる
容器4の水を容器2に入れる。幽霊が容器4に水4を補充する。
容器8の水を容器4に入れる。幽霊が容器8に水8を補充する。
これで状態は(18)(24)(3)(48)(5)(6)(7)(8)となる。
容器1,2,4の水を容器8に入れると
状態は()()(3)()(5)(6)(7)(1244888)となる。

このように操作を繰り返して最終的に
状態を(18)(1248)(37)(124488)(3657)(365577)(3655777)(1244888)
としたかったんだが、勘違いしてて上手く出来ないっぽい…やり直す
あと>>595のスレ見ると4回のやり方もあるっぽいな
613132人目の素数さん:2008/02/07(木) 20:10:55
(1) (2) (3) (4) (5) (6) (7) (8)→(14) () (3) () (5) (6) (7) (28)
→(14) (2) (3) (4) (5) (6) (7) (28)→(14) () (3) (4) (5) (6) (7) (248)
→(14) (2) (3) (4) (5) (6) (7) (248)→(14) () (3) (24) (5) (6) (7) (248)
→(14) (2) (3) (24) (5) (6) (7) (248)→() (124) (3) (24) (5) (6) (7) (248)
→(1) (124) (3) (24) (5) (6) (7) (248)→(1248) (124) (3) (24) (5) (6) (7) ()
→(1248) (124) (3) (24) (5) (6) (7) (8)→(1248) (1248) (3) (24) (5) (6) (7) ()
→(1248) (1248) (3) (24) (5) (6) (7) (8)→(1248) (1248) (3) (248) (5) (6) (7) ()
→(1248) (1248) (3) (248) (5) (6) (7) (8)→() (1248) (3) (248) (5) (6) (7) (12488)
→(1) (1248) (3) (248) (5) (6) (7) (12488)→() (1248) (3) (1248) (5) (6) (7) (12488)
→(1) (1248) (3) (1248) (5) (6) (7) (12488)
(略)→(1) (1248) (3567) (1248) (5) (3567) (35677) (12488)
→(1) (1248) (3567) (1248) () (3567) (355677) (12488)
→(1) (1248) (3567) (1248) (5) (3567) (355677) (12488)
と水を移し変える
水1,2,4,8が入った容器の中身を水槽に入れると
幽霊は0〜15匹のどれかを言う→水1,2,4,8のどれに魚の幽霊がいるか分かる
水1,2,4が100ml、水8が200ml入った容器の中身を水槽に入れた場合、
水3,6が100ml、水5,7が200ml入った容器の中身を水槽に入れた場合も同様に分かる

ただ幽霊は嘘をつく事があるから2,3回同じことを確かめなきゃいけず
この場合だと5回かかる

>>611 この説明でも駄目か?
614132人目の素数さん:2008/02/08(金) 09:32:27
>>612

> ある容器に水1が100ml、水2が200ml、水4が400ml入っているような状態を(122444)と書くとする

オレにはこれは 「水1が100ml、水2が200ml、水4が300ml」 を意味しているように見えるんだが
444は400mlなのか? 300mlはどう書くんだ?

意図どおりに正しく書かれているのかを確認してくれ。
前提の大事なところにタイプミスがあったりしたら、意思の疎通ができない。
615132人目の素数さん:2008/02/08(金) 14:50:04
>>614
あぁ…そこでタイプミスしてたようでごめんよ
「水1が100ml、水2が200ml、水4が300ml」だ
616132人目の素数さん:2008/02/10(日) 03:18:01
△ABCがあり、三辺の長さは公差がdの等差数列をなしている。
最大辺の延長上に点Pを取ると、元の辺から増えた長さがdであった。
また、最小辺上に点Qを取ると、元の辺から切り取った長さがd/2であった。
ただしQが切り取った長さは、最大辺を含む点から測ったものとする。
さらに、直線PQの延長が△ABCの残った一辺と交わる点をRとする。
すると、直線PQRは△ABCの面積のうち半分を切り取ることとなった。
三辺の長さをdを用いて表せ。
617132人目の素数さん:2008/02/10(日) 04:22:45
>>616
AB < AC < BC として一般性を失わない。
AB = x とすると、 AB + AC > BC より x > d。
Q は AB 上にあり、 R は AC 上にある。
△ABC = 2△AQR より AB*AC = 2*AQ*AR。
メネラウスの定理より (PC/PB)*(QB/QA)*(RA/RC) = 1。
これらを連立して x について解けば良い。
618132人目の素数さん:2008/02/15(金) 22:05:18
ABCに正の整数(何桁でも可)を入れて等式を成立させて下さい
ただし同じ文字には同じ整数,違う文字には違う整数が入ります

(A+B−C)×{(1/A)+(1/B)−(1/C)}=564
619132人目の素数さん:2008/02/16(土) 18:17:31
正方形をどの2つも合同でない3つの相似な図形に分割せよ(フラクタル不可)
620132人目の素数さん:2008/02/16(土) 20:41:18
>>619
七問目-26
621132人目の素数さん:2008/02/18(月) 14:11:16
五段の階段があり、サイコロを振って出た目だけ上り、ちょうど一番上に止まったら終了する。
ただし、五段目までの段数よりサイコロの出た目が大きい時は、その分だけ下がるとする。
例えば、三段目で5が出たら、二段上った後三段下がるので、二段目になる。
サイコロの目が出る確率はすべて等しいとする。
一番下(0段目)から始めてN回サイコロを振った時、n段目にいる確率をA(N-n)とする。

問1.A(2-0),A(2-1),A(2-2),A(2-3),A(2-4),A(2-5)を求めよ。
問2.A(N-5) (N≧1) を、Nを用いてあらわせ。
622621:2008/02/18(月) 15:29:35
せっかく作ったんだから解いてよ!
623132人目の素数さん:2008/02/18(月) 16:19:49
まだ一日もたってないだろ&終了したらどうなる?&何故引き算?
624132人目の素数さん:2008/02/18(月) 16:21:36
A(i-3-1)
625621:2008/02/18(月) 16:24:53
>>622
引き算?

>一番下(0段目)から始めてN回サイコロを振った時、n段目にいる確率をA(N-n)とする。
N-nは引き算じゃなくて、添え字ww
626132人目の素数さん:2008/02/18(月) 16:56:36
>>625
だったらN_nって書け
627132人目の素数さん:2008/02/18(月) 17:08:06
漸化式書いて終わりのよくある問題にしか見えん
628132人目の素数さん:2008/02/18(月) 17:20:00
ttp://www.vippers.org/img/uho2820.jpg
□の次は何でこうなるの・・・?
わからん・・・やばい・・・
629sage:2008/02/18(月) 17:33:11
職場で
図の斜線が引いてある部分の面積を求めよ。但し、π=3 √3=1.7とする。

と言う問題が出たのですが無知な私にはさっぱり解りません。どなたかご教授お願いします


http://a.pic.to/nkx34
630132人目の素数さん:2008/02/18(月) 17:37:05
なんというスレ違い
631sage:2008/02/18(月) 17:59:28
お手数ですがお願いします。皆様が頼りなんです
632132人目の素数さん:2008/02/18(月) 18:03:59
取り下げて移動しないとここでは回答はいただけないかと
http://science6.2ch.net/test/read.cgi/math/1197828000/

>>629
◆指定されたページは存在しないか、携帯端末以外からのアクセスは許可されていません
633132人目の素数さん:2008/02/18(月) 18:04:52
>>627
解けないくせに…
634132人目の素数さん:2008/02/18(月) 18:37:19
>>633
難しくて全然分からないよ
635132人目の素数さん:2008/02/18(月) 18:52:11
>>621
スレ違い
636132人目の素数さん:2008/02/18(月) 21:07:26
30になった
637132人目の素数さん:2008/02/18(月) 23:22:22
三十路か。
誕生日おめ。
638132人目の素数さん:2008/02/19(火) 01:51:03
>>622名大入試問題のパクリやん…
639132人目の素数さん:2008/02/19(火) 04:19:38
五段だと他のどこの段にいたときも1/6の確率であがれるわけだから全然面白くない
640132人目の素数さん:2008/02/19(火) 13:45:23
4段目で6が出たら落ちて死亡して終了ですか?
641132人目の素数さん:2008/02/19(火) 13:50:00
あれ? ひょっとして5段の階段というのは高さは6種あるのかな?
階段の段数は高さの種類-1なんですかね?
0段の階段というのはいわゆる平地なんでしょうか?

じゃあ1段目ってのは地面と同じ高さ?
それともひとつ上?

1階って地面と同じ高さ、2階はその上
てことは1段目は地面と同じで、ひとつ登ると2段目?
いやいや、n階建のnは高さの種類と等しいから階段とは違うのかな?
642132人目の素数さん:2008/02/20(水) 00:01:15
>>618
 A = 122 44200 50100 02877 81163 51171 17995 21361 35134 91867 (48桁)
 B = 74807 19101 53025 27837 94583 60171 46464 94820 59055 28060 (50桁)
 C = 3460 69586 84255 04865 64589 22621 88752 08971 30654 24460 (49桁)

なお、因数分解すると
 A = bc * 19 * 29 * 46591 * 100363979,
 B = ca * 37 * 1806705194075701,
 C = ab * 3 * 11 * 73 * 1543 * 4663 * 169789,
 a = lcd(B,C) = 2^2 * 5 * 449 * 35869 * 516624461,
 b = lcd(C,A) = 97 * 3769 * 19329352219,
 c = lcd(A,B) = 7 * 79 * 239441 * 50787883,
 lcd(a,b,c) =1,
らしい。

〔参考文献〕
A.Bremner, R.K.Guy and R.J.Nowakowski;
"Which integers are representable as the product of the sum of three integers with the sum of their reciprocals ?"
 Math. Comput. Vol.61, No.203, (1993 July)
http://www.asahi-net.or.jp/~KC2H-MSM/mathland/math03/math0304.htm
643132人目の素数さん:2008/02/20(水) 00:17:27
〔問題〕
x,y,z に整数を入れて等式を成立させて下さいです。。。
1) x^3 + y^3 + z^3 = 564,
2) x^3 + y^3 +2z^3 = 564,
644132人目の素数さん:2008/02/20(水) 01:11:54
z=0は明らかなんだからただの二元三次連立方程式だな
645643:2008/02/20(水) 04:06:17
>644

1) と 2) は別々の問題でつ。 2) の方が易しいでつ。
646132人目の素数さん:2008/02/20(水) 04:50:19
問題番号見てなかったぜorz

>>645
2もzが奇数でx,yが偶数であることしかわからないんだぜ
647132人目の素数さん:2008/02/20(水) 04:52:49
ん!!?

よく考えたらxyzすべて奇数だよな・・・?
648643:2008/02/20(水) 05:31:00
>643 の 2) はすべて奇数で、2桁以下らしいよ。1桁もあるらしい。
649132人目の素数さん:2008/02/20(水) 06:47:58
この問題の面白さがよくわからんのだが

2)
-1^3 - 21^3 + 2*17^3
= 23^3 - 33^3 + 2*23^3
= 93^3 + 95^3 - 2*94^3
= 564
650132人目の素数さん:2008/02/20(水) 09:24:38
564 = 538724191073^ + (-1300749634)^3 + (-53872166335)^3
(Leonid Durman, 2007)

きっと ttp://www.asahi-net.or.jp/~kc2h-msm/mathland/math04/math0403.htm から
持ってきたんだと思うけど、出題者は面白さが分かってるんだよね?

俺は564に関するこの結果は、単に計算がんばったら出た、ってものだと
思ってたんだけど、なんか面白いところがあるなら教えてもらいたいなあ。
651643:2008/02/20(水) 22:39:22
>>649-650
御名答.
見た目は似たような「564」でも、難問もあれば易しいのもある、という事で……(当然だが)
因数分解しても、とくに面白くないし……
 564 = 2^2 * 3 * 47,
 53872419107 = 13 * 1193 * 3473623,
  1300749634 = 2 * 223 * 2916479,
 53872166335 = 5 * 13^2 * 47 * 59 * 83 * 277,


>>642
 B = c * a * 37 * 2829383 * 638550947,
 gcd(a,b,c) =1,
652132人目の素数さん:2008/02/20(水) 23:30:00
lcd=最大公約数?
653132人目の素数さん:2008/02/29(金) 02:19:14
いくつ解けますか?

ブラジル数学オリンピック 本選 2004 小学生対象
[1問目] 下記の条件を満たす全ての3桁の自然数nを全て見つけよ。
n は奇数である
n は平方数である
n の各位の数字の平方の和は平方数となる

[2問目] 辺の長さ1の4つの正三角形で、図のように、辺の長さ2の正三角形を作ることができる。

 △
△▽△

先の図のような三角形を作るために、3つの種類(白、黒、灰色)の辺の長さが1の正三角形をたくさん持っているとする。
作られた二つの三角形のうち、回転して同じになるもの同士は同じものとみなす。

 黒  |  白
白白灰 | 灰白黒
↑同じペア

 黒  |  黒
灰白白 | 白白灰
↑異なるペア

この提示された条件でいくつの異なる三角形を作ることができるか。
654132人目の素数さん:2008/02/29(金) 02:22:29
[3問目] 自然数が1より大きな二つの自然数の積で表されるとき、それを合成数という。
たとえば91は91=7*13とかけるので、合成数である。
2^(2^2004+2)+1 が合成数であることを示せ。

[4問目] ArnaldoとBernaldoが2*nのテーブルでゲームをする。駒は2*1のドミノで、まずArnaldoが最初にテーブルの2マスに収まるように縦または横にしてドミノをおく。
プレイヤーは常にテーブルの2マスに収まるように縦または横にしてドミノをおかなくてはいけない。既に置かれたドミノに重ねてはいけない。
テーブルにおけなくなった方の負けである。
どちらのプレイヤーが必勝であるか?
(a) n = 2004 の時は?
(b) n = 2005 の時は?

[5問目] 辺の数が13のタイルで平面の敷き詰めは可能か
もし可能ならば例を示し、不可能ならばそれを証明せよ。
655132人目の素数さん:2008/02/29(金) 03:51:19
>654
[3問目]
 2^(3m) + 2 = 4n + 2,
 2^(4n+2) = 4^(2n+1) ≡ (-1)^(2n+1) = -1 (mod 5)
∴ 5 の倍数…
656132人目の素数さん:2008/02/29(金) 04:50:26
>>654
[4問目]
nが偶数なら後手必勝。 先手と線対称(点対称でもいい)に置き続ける。
nが奇数なら先手必勝。 一手目は(n+1)/2マス目(中央)に縦に置く。

[5問目]
辺の数が13のタイルってのは13角形ならなんでもいいのかな?
だったら、正方形の対角線のあたりを中央点対象にギザギザに切ればいい。

それとも凸型に限定されるのか?
657132人目の素数さん:2008/02/29(金) 10:09:11
AKB48のCDを1枚買うと、ポスターが1枚もらえます。このポスターは全部で44種類あり
もらえるポスターはランダムです。
そして44種類全部集めると特典が付いてくるのですが、果たして44種類コンプリート
するには、平均何枚の購入が必要となるでしょう?

別に問題的には面白くないかw
ニュースとしては面白かったが・・・
658132人目の素数さん:2008/02/29(金) 10:24:55
>>657
クーポンコレクター問題。

問題としては面白いと思うが、あまりに有名だろう。
659132人目の素数さん:2008/02/29(金) 10:28:03
n種類だとおよそ平均n*log n くらいだね。(nが十分大きいとき)
660132人目の素数さん:2008/02/29(金) 13:37:02
>>657-658
ってか、すでにマルチされまくってるじゃんか。
661132人目の素数さん:2008/02/29(金) 19:16:33
3^n (n=1,2,3,‥)を十進表記したとき、
十の位は常に偶数であることを示せ。
662132人目の素数さん:2008/02/29(金) 19:33:19
3^1=3, 3^2=9,3^3=27, 3^4=81
3^(n+4)-3^n=3^n*80
663132人目の素数さん:2008/02/29(金) 19:52:23
ゴリ押しで解いてみた。
ttp://www.csync.net/service/file/view.cgi?id=1204281995

ちょうどn回目で全てのおまけが集まる確率を実際に計算し、そのあとは
期待値の定義に従ってゴリ押しで期待値を計算し、でも出て来る値は
MΣ[k=1〜M](1/k)の形をしていないので、この形になるように変形。

>>659
γをオイラー定数とするとき、nΣ[k=1〜n](1/k)=nlogn+γn+ε(n),|ε(n)|≦1 (n=1,2,…)
664132人目の素数さん:2008/02/29(金) 20:46:34
なんか添え字ミスがいっぱいあった。
ttp://www.csync.net/service/file/view.cgi?id=1204285553
665132人目の素数さん:2008/03/01(土) 06:38:29
>>655,656
正解.今度は中学生向けのを翻訳してみた.

[Q1] 幾何の問題なので省略

[Q2] 次の数字の列 1,2,3,4,0,9,6,9,4,8,7,...は次の方法で作られる.
5つめ以降の各要素はその前の4つの和の下一桁である.
a) 2,0,0,4という並びはこの数列に現れるだろうか
b) 1,2,3,4という並びは再びこの数列に現れるだろうか

[Q3] Esmeraldaは100個の石を持っている.
彼女はこの山を二の新しい山に分け、続いてこの二の山の石の数を掛け合せ
その積を紙に書きこむ.
そして、一つ以上の石がある山を選び次の操作をくりかえす.
一つの山を二つに分け、この二の山の石の数の積を紙に書きこむ.
紙に書かれたすべての積の和はどのような値になるだろうか?
666132人目の素数さん:2008/03/01(土) 06:40:51
前の書きこみ, "二の" -> "二つの" に訂正.

[Q4] ArnaldoとBernaldoの二人が交互に正の数を選ぶゲームをする.
それぞれのターンでは、前の人が選んだ数よりも大きく、かつ二倍未満の数を選ばないといけない.
このゲームでは2004を選べたプレイヤーが勝利する.
Arnaldoが先手で2から始めたとき、どちらのプレイヤーが必勝か.

[Q5] Dを直角三角形ABCの斜辺ABの中点とする.
O1とO2をそれぞれ三角形ADCとDBCの外接円の中心とする.
a) 角O1DO2が直角である事を示せ.
b) ABがO1O2を直径とする円に接する事を示せ.

[Q6] 0,1,2,...,9の各数字が10個ずつあり、これを10x10の升目に並べることを考える.
どの並べ方に対しても、ある列または行に少なくともn種類の数字が
含まれているというとき、nの最大値を求めよ.
667132人目の素数さん:2008/03/01(土) 10:05:24
>>665
[Q2] a)
この数列は 奇数、偶数、奇数、偶数、偶数 の並びを繰り返すので 4つ偶数が並ぶことはない。
b) 実際に書き下せば現れるんだが… うまい方法が見つからん。

「Q3]
Σ_[n=1→99]{n}
n個の山をかならずn-1と1の分けることにするとそうなる。
さらにn-2と2に分ける場合もn-1と1に分けたのと同じ結果になることを…
さらにn-kとkに分ける場合もn-1と1に分けたのと同じ結果になることを…
結局どう分けようとも…

[Q4]
nを選ぶためには、nが偶数ならn/2を、nが奇数なら(n-1)/2を選べられればいい。
2004を選ぶためには1002を選べればいい、そのためには501を、そのためには250を
そのためには125を、62を、31を、15を、7を、3を選べればいい。
後手必勝である。


668132人目の素数さん:2008/03/01(土) 12:18:37
Q2のb
ループすることと、列の4つの並びからその前が自動的に決まることから1,2,3,4を含むループだとわかる
669132人目の素数さん:2008/03/01(土) 15:22:01
a,b,cは異なる自然数とし、全て2以上であるとする。
(1+a+a^2+…+a^n)+(1+b+b^2+…+b^n)がcで割り切れ、さらに、
(1+a+a^2+…+a^(n+1))+(1+b+b^2+…+b^(n+1))がcで割ると
2余るような自然数nが存在するとき、そのようなnのうち最小のものを
rとおけば、rはnを割り切ることを示せ。
670132人目の素数さん:2008/03/01(土) 15:24:38
訂正。

2余るような自然数nが存在するとき、そのようなnのうち最小のものを
rとおけば、rはnを割り切ることを示せ。

2余るような自然数n=n_0が存在するとする。このようなnのうち最小の
ものを rとおけば、rはn_0を割り切ることを示せ。
671132人目の素数さん:2008/03/01(土) 15:47:36
>>666
[Q5] Dを直角三角形ABCの斜辺ABの中点とする.
O1とO2をそれぞれ三角形ADCとDBCの外接円の中心とする.

a) 角O1DO2が直角である事を示せ.
外接円は各辺の垂直二等分線の交点で表されるので,円O2はACの垂直二等分線上にある.∴DO2⊥AC 同様にBD⊥DO1 よってOD1⊥DO2 ∠OD1O2=90°(証明終了)

b) ABがO1O2を直径とする円に接する事を示せ.
[方針]例えばA(0,4a),B(4b.0)とおき,D,O1,O2の座標を求める.新たな円の中心をO3とする。このときABが円O3の接線になることを示す。又、接線と半径は垂直となるはずだから,<O3D>と<AB>との内積が0になることを示せばよい。

解答は計算が面倒なので...すいません。

[自作問題] 縦1,横√3の長方形をSとする。Sの対角線を中心に30°
回転させた長方形をS'とする。このときSとS'の重なった部分の面積を
求めよ.

 高校入試レベルの問題ですが、辺の長さの求め方などが解答する人によって分かれるという意味で面白いと思います。
672132人目の素数さん:2008/03/01(土) 19:52:49
>>668
ループすることはわかるんだが、その先がよくわからない。
もうすこし詳しく教えてもらえないだろうか?
673132人目の素数さん:2008/03/01(土) 19:55:51
>>671
> 角線を中心に30°回転

線を中心に回転させると(回転軸)重なる部分は直線(面積0)になると思うのだが
しかし高校入試に空間図形というのもめったに出ないだろうし、なにか出題ミスか?
6742ndVirgin ◆8dN/5Nqmfw :2008/03/01(土) 20:24:58
>>672
ループするのはわかるんだな、じゃあ2つの可能性がある
・途中からループに入る
・最初からループしている
途中からループに入るとしたら、その部分は逆回転するときに枝分かれに見えるだろ?
4つの並びからその前が決るとなら、そんなことは起こり得ない。
だから最初からループしてるてことだ

俺は>>668になるほど、と思っただけで気づいたわけじゃねえよ
675132人目の素数さん:2008/03/01(土) 20:27:56
>>674
いやすまん。
> 4つの並びからその前が決るとなら
ここがわからないんだ。
6762ndVirgin ◆8dN/5Nqmfw :2008/03/01(土) 20:30:56
a+b+c+d≡e (mod. 10)
なら
a≡e-b-c-d (mod. 10)

aは0〜9だから丁度1個ある
677132人目の素数さん:2008/03/01(土) 20:33:25
なるほどそうか。 サンキュ。
678132人目の素数さん:2008/03/01(土) 21:38:01
>>666
[Q6]の意味がいまひとつわからん。
679132人目の素数さん:2008/03/01(土) 22:38:32
並べ方Aに対し
i行にr_i(A)種類の数字
j列にc_j(A)種類の数字
があるとする
r(A):=max_i(r_i(A))
c(A):=max_j(c_j(A))
n(A):=max{r,c}
としたとき
min_A(n(A))を求めよ、ということだろう
680671:2008/03/01(土) 23:50:28
>>673さん。
 問題の部分。「対角線」ではなく。「(二本の)対角線の交点」となり
ます。書き間違えまえてしまってすみません。

[自作問題(訂正)] 縦1,横√3の長方形をSとする。このときSにおける二本
の対角線の交点を中心に30°回転させた長方形をS'とする。このときSとS'の
重なった部分の面積を求めよ.
681132人目の素数さん:2008/03/04(火) 10:48:57
□に1〜9の整数を1つずつ入れ成立する等式を全て求めて下さい
同じ数字2度使いは不可

□□□×□□=□□×□□
682132人目の素数さん:2008/03/04(火) 10:55:33
既出
683132人目の素数さん:2008/03/04(火) 11:09:16
134*29 = 58*67
138*27 = 54*69
146*29 = 58*73
158*23 = 46*79
158*32 = 64*79
174*23 = 58*69
174*32 = 58*96
186*27 = 54*93
259*18 = 63*74
532*14 = 76*98
584*12 = 73*96
684132人目の素数さん:2008/03/04(火) 11:34:32
>>600
アホな質問かも知れんけど、(x+x_i) の偶数個だけ負なんてないん?
6852ndVirgin ◆8dN/5Nqmfw :2008/03/04(火) 11:50:51
>>684
x≧0 ⇒ f(x) ≧ S_n >0から
f(x)=0の解はx≧0には存在しない
つまり、f(x)=0の解-x_1,-x_2,...,-x_nは(実数だから)すべて負
と考えてるんだと思う
686132人目の素数さん:2008/03/04(火) 19:49:41
正整数全体の集合をNとする。A={-1,0,1}に対して
M(A)={Σ[k=1,n](3^(k-1))a[k] | n∈N, a[k]∈A}
とする。このときN⊂M(A)であることを示せ。
687132人目の素数さん:2008/03/04(火) 19:53:24
>>686
ただの3進法じゃないの?(0,1,2じゃなくて-1,0,1に変えただけの)
688132人目の素数さん:2008/03/04(火) 20:09:07
三進法と同じです
条件を満たすAは他にあるか、を入れ忘れてました
689132人目の素数さん:2008/03/04(火) 20:12:18
Aを含む集合ならOKだし
690132人目の素数さん:2008/03/04(火) 20:16:41
ノントリビアルなのを考えるのが面白いと思い、出題しました
691132人目の素数さん:2008/03/04(火) 20:24:52
まあ出題者の意図を汲むのも解答者のたしなみとは思うが
もうちょっとシャンとしてくれや
692132人目の素数さん:2008/03/04(火) 20:35:26
シャンとしなくて申し訳ないです
問題文を改めてみました

正整数全体の集合をNとする。
相異なる三つ整数からなら集合Aに対して
M(A)={Σ[k=1,n](3^(k-1))a[k] | n∈N, a[k]∈A}
とする。このときN⊂M(A)となるAを全て求めよ。
693132人目の素数さん:2008/03/05(水) 03:54:27
この前数学教師から出されて解けなくてイライラしてる問題を。
既出だったらすまん

三角形がある。
今、△ABCはA=20°の二等辺三角形で
ACの間に、AD=BCの点Dをとる。

このとき、∠ADBを求めろ。
694132人目の素数さん:2008/03/05(水) 04:35:44
ラングレーでググれ
695132人目の素数さん:2008/03/05(水) 07:57:30
>>649
レスthx
でもこれラングレーとちがくね?
ラングレーってBとCの所に角度の決まった辺が付くし
求める角度の場所も違うような
一応ググったが・・・すまん
696132人目の素数さん:2008/03/05(水) 08:07:35
AとBとCそれぞれに0〜9迄の数字を1つずつ入れて成立する等式を全て求めて下さい
AとBは違う組み合わせにして下さい.同じ文字での同じ数字2度使いは不可

AAAAAA×AAAA=BBBBBB×BBBB=CCCCCCCCCC
697132人目の素数さん:2008/03/05(水) 09:53:08
15864*9327 = 18654*7932 = 147963528
37962*5148 = 51948*3762 = 195428376
26475*9381 = 62481*3975 = 248361975
54186*7923 = 87153*4926 = 429315678
698132人目の素数さん:2008/03/05(水) 10:24:05
197208*5346 = 723096*1458 = 1054273968
253098*4176 = 538704*1962 = 1056937248
192087*6435 = 960435*1287 = 1236079845
820794*1536 = 950784*1326 = 1260739584
435618*2970 = 653427*1980 = 1293785460
154638*9072 = 231957*6048 = 1402875936
286794*5103 = 479682*3051 = 1463509782
158640*9327 = 186540*7932 = 1479635280
537186*2940 = 926835*1704 = 1579326840
319875*6042 = 486210*3975 = 1932684750
379620*5148 = 519480*3762 = 1954283760
249651*7803 = 748953*2601 = 1948026753
815430*2679 = 945687*2310 = 2184536970
264750*9381 = 624810*3975 = 2483619750
286071*9354 = 781059*3426 = 2675908134
405612*7938 = 709821*4536 = 3219748056
549780*6231 = 837165*4092 = 3425679180
541860*7923 = 871530*4926 = 4293156780
453186*9702 = 906372*4851 = 4396810572
651840*7293 = 847392*5610 = 4753869120
704592*6831 = 830412*5796 = 4813067952
751398*6420 = 783240*6159 = 4823975160
564732*9180 = 914328*5670 = 5184239760
699132人目の素数さん:2008/03/05(水) 18:54:47
>>697-698 どうやって求めんの
700132人目の素数さん:2008/03/05(水) 20:33:02
?に0〜9迄の数字を1つずつ入れて成立する等式を全て求めて下さい
分や秒は59以下,時間は1以上.同じ数字2度使いは不可

??分??秒×?=?時間??分??秒
701132人目の素数さん:2008/03/05(水) 20:49:25
kingは死ぬ
702132人目の素数さん:2008/03/05(水) 20:55:12
明日ここにカキコしたあと心臓麻痺で死ぬ、とデスノートに書いておいた
703132人目の素数さん:2008/03/05(水) 21:24:23

つぎの やさいのうち なかまはずれは どれ ?

1. だいこん  2. にんじん 3. ごほう 4. じゃがいも

私立幼稚園入試問題
704132人目の素数さん:2008/03/05(水) 21:26:57
じゃがいも。
ほかのみっつはふといねっこそのものをたべる。
705132人目の素数さん:2008/03/05(水) 21:35:29

せいかい

3. ごぼう  です。

だいにじせかいたいせん のとき

ほりょとなった あめりかじん にだした りょうりに

ごほう を だしたら、 にほんじんは われわれに

きのねっこ を たべさした と くじょうがあり

そのご しゅうせん を むかえ ほりょしゅうようじょ の

しょちょうが じぃえいちきゅう から せんぱん の ようぎ を

かけられました。

とさ
7061stVirtue ◆.NHnubyYck :2008/03/05(水) 21:35:47
Reply:>>701 お前が先に死ね。
707132人目の素数さん:2008/03/05(水) 21:50:29

もんだい

1. しゃつ  2. ぱんつ 3. くつした 4. はんかち

なかまはずれは どれ ?

国立教育大付属幼稚園入試問題
708132人目の素数さん:2008/03/05(水) 21:54:24
ぱんつ

これはニオイかいでみたいから
709132人目の素数さん:2008/03/05(水) 21:55:18
面白くない問題は自粛願います
7101stVirtue ◆.NHnubyYck :2008/03/05(水) 22:02:18
Generalist の修行が足りない奴が仲間はずれを決めてはならぬ。
711132人目の素数さん:2008/03/05(水) 22:10:22
>>710
スレチ
712132人目の素数さん:2008/03/05(水) 22:14:14

せいかい

4. はんかち

ほかのものは どれも りったいてき であります。

また、ことばに つ が ついています。
713132人目の素数さん:2008/03/06(木) 00:35:04
>>700
50分42秒 * 9 = 7時間36分18秒
714132人目の素数さん:2008/03/06(木) 03:41:02
シャツとパンツ(小用穴付き)は同相。
ハンカチと靴下は同相。
715132人目の素数さん:2008/03/07(金) 07:11:22
>>705

ようぎを かけられたのは よくわかった が
なぜ それが なかまはずれの りゆうに なるんだ?
716132人目の素数さん:2008/03/07(金) 15:35:15
>>693 >>695

線分BCのC側の延長上に、BE=ACとなるような点Eをとり、
△BEF≡△ABCとなるような点Fを、BEから見てAと同じ側にとると
△ABEは正三角形,四角形ADEFは平行四辺形
△EDBはED=EBの二等辺三角形
717132人目の素数さん:2008/03/07(金) 17:27:21
>>716
> △ABEは正三角形,四角形ADEFは平行四辺形
? ?
718132人目の素数さん:2008/03/07(金) 17:33:43
>>716
> △ABEは正三角形

???
EはBCの延長上にあるのだから
∠ABE は80°じゃないか?

そもそも>>693の図形は一意じゃないのだが
二等辺三角形ってのは
AB と BC が等しいのか?
AB ACが等しいのか?
719132人目の素数さん:2008/03/07(金) 17:46:36
□に1〜25の整数を1個ずつ入れ縦横斜め(全12列)の合計を全て同じにして下さい
同じ数字2度使いは不可

 □ □ □ □ □

 □ □ □ □ □

 □ □ □ □ □

 □ □ □ □ □

 □ □ □ □ □
720132人目の素数さん:2008/03/07(金) 17:50:07
17 24 1 8 15
23 5 7 14 16
4 6 13 20 22
10 12 19 21 3
11 18 25 2 9
721132人目の素数さん:2008/03/07(金) 17:51:12
 17 24 1 8 15
 23 5 7 14 16
  4 6 13 20 22
 10 12 19 21 3
 11 18 25 2 9
722132人目の素数さん:2008/03/07(金) 18:06:57
>>717 >>718
タイプミス。「△ABFは正三角形」が正解
(一人ぐらい察してくれw)
723716,722:2008/03/07(金) 18:20:55
おっと一言忘れてた
タイプミス スマソ。
724132人目の素数さん:2008/03/07(金) 18:22:03
下の*に数字を1つずつ入れて式を成立させて下さい。
*****×*****=123456789

(ただし全通り求める事)
725132人目の素数さん:2008/03/07(金) 18:23:45
> 四角形ADEFは平行四辺形

については どうだ?
∠B = ∠E = 80°だと思うのだが AD//EFなのか?
726132人目の素数さん:2008/03/07(金) 18:40:30
>>724
10821×11409
11409×10821

>>725
DはAB上ではなくAC上にあるのだが
727132人目の素数さん:2008/03/07(金) 19:05:06
ここに全て異なる10個の自然数があります
その中のどの9個の和も素数になります
その10個の自然数は何でしょうか?
ただし合計が最も小さくなるものを答えて下さい
728132人目の素数さん:2008/03/07(金) 21:20:21
>>727
たぶんこれ
1,3,9,13,15,19,31,43,45,63
729132人目の素数さん:2008/03/07(金) 23:37:04
1,3,7,9,19,21,27,33,37,43
は試してみたか728
730132人目の素数さん:2008/03/08(土) 00:09:21
>>729
見落としてた...orz
手計算でやってたもので。残念。
731132人目の素数さん:2008/03/08(土) 13:50:39
ABCDの4人に10問の◯×試験をしたら下の様な回答が帰ってきました
Aは7点、Bは5点、Cは3点でした.これよりDは何点だったかを推測して下さい

 1234567890
A:xxOxOxxOxx 7点
B:xOOOxOxOOO 5点
C:xOOOxxxOxO 3点
D:xxOOOxxOOO ?点
732132人目の素数さん:2008/03/08(土) 15:48:43
問題
m, n を 5 ≦ m, n ≦ 10 なる自然数とする。
この時、 m^3 + n^2 の最小値はいくらか。
制限時間 1 秒。
733132人目の素数さん:2008/03/08(土) 15:59:13
すまん、問題読むだけで3秒くらい掛かった
734132人目の素数さん:2008/03/08(土) 16:00:50
では4秒で。
735132人目の素数さん:2008/03/08(土) 16:04:07
すまん、鉛筆探すので5秒ぐらいかかった。
736132人目の素数さん:2008/03/08(土) 16:06:18
何で面白くない問題ばかりなの?
737132人目の素数さん:2008/03/08(土) 17:39:34
>>736
しゃれが分からんのか?
732の答えは150ではない
738132人目の素数さん:2008/03/08(土) 17:42:26
そもそも150なんてどこから出てくるの?
739132人目の素数さん:2008/03/08(土) 17:44:27
5^3+5^2じゃね?
740132人目の素数さん:2008/03/08(土) 17:45:46
最小は5^3+1^2じゃないの?
741132人目の素数さん:2008/03/08(土) 17:54:29
>>740
正解が出るのに3年かかった
742132人目の素数さん:2008/03/08(土) 18:02:36
なんでn=1がOKなのかサッパリ分からん
743132人目の素数さん:2008/03/08(土) 18:05:00
>>742
オレもわからん
744132人目の素数さん:2008/03/08(土) 18:05:19
5 ≦ m
n ≦ 10
だろ?
n=1はOKでしょ
745132人目の素数さん:2008/03/08(土) 18:07:38
曖昧な記法で他人をたぶらかす愉快犯の類であろうか。
746132人目の素数さん:2008/03/08(土) 18:12:55
5 ≦ m, n ≦ 10

5 ≦ n ≦ 10
と思わせるってことか
逆に気がつかなかったw
747132人目の素数さん:2008/03/08(土) 18:21:00
n,m∈{5,6,7,8,9,10}のことを5 ≦ m, n ≦ 10 と表すのは
よくあることで、>>745が真理だな。
748132人目の素数さん:2008/03/08(土) 18:33:28
>>731
6点でいい?
749731:2008/03/08(土) 18:58:35
>>748
正解.解説は↓

245690の問題はAとBの答えが違うのでどちらかが正解していて合計6点
Aは7点、Bは5点だからこの部分でAは4点、Bは2点取っていると分かる
1378の問題は4人とも答えが同じだから、 この内3つが正解していて4人ともこの部分で3点取っていると分かる
よってCの245690は全部間違いだから正解は以下の4通りが考えられる

  1234567890
(1) xxxxOOxOOx
(2) xxOxOOxxOx
(3) xxOxOOOOOx
(4) OxOxOOxOOx

この4つのいずれで採点してもDは6点になる
750132人目の素数さん:2008/03/08(土) 22:27:58
どっかの中学入試で731みたいな問題なかったっけ
751132人目の素数さん:2008/03/08(土) 23:16:15
中学入試でそんな難しいのださないよ
752132人目の素数さん:2008/03/08(土) 23:30:45
似たような問題ってことだろ。
俺も日能研の電車広告で見た気がする。
753693:2008/03/09(日) 00:37:03
>>718
レス遅れてスマソ
問題を端折り過ぎたようだ

△ABCがあったとき
A=20°
B=C=80°の二等辺三角形だとし
AC上にBC=ADとなる点Dをとる

そのときの∠ADBを求めろ

でした><
面白くないかもだけど頼む!
754132人目の素数さん:2008/03/09(日) 00:47:10
x=cosx(0<x<π/2)の解は初等関数では絶対に表現できないと確信している。
755132人目の素数さん:2008/03/09(日) 01:04:35
756132人目の素数さん:2008/03/09(日) 15:40:12
>>754
定数は初等関数
よって初等関数で表現出来る
757132人目の素数さん:2008/03/09(日) 15:42:28
いや、たぶん無理数だろうから無理数をのぞいて初等関数で表現できないといいたかっただけ(ぁ
758132人目の素数さん:2008/03/09(日) 19:13:50
釣りにマジレス
759132人目の素数さん:2008/03/09(日) 19:24:51
>756は

x=cosxの(ただ1つの)解x=αに対して、f(x)=α (x∈R)という関数f(x)は定数関数であり、これは初等関数。
よって、x=cosxの解は初等関数で表現できる。

と言いたいのだろう。一方で>754の言いたいことも分かるが、まずは
「定数が初等的関数で表せる」
がどういうことなのか定義しなさい。
760132人目の素数さん:2008/03/09(日) 21:42:40
三つのベクトル↑a、↑b、↑cは異なるどの二つの内積も1になる。
このとき、max(|↑a| , |↑b| , |↑c|)とmin(|↑a| , |↑b| , |↑c|)の範囲を求めよ
761132人目の素数さん:2008/03/09(日) 22:46:10
3つのベクトルが互いに異なるなら
max(|↑a| , |↑b| , |↑c|)>1
min(|↑a| , |↑b| , |↑c|)>0
762132人目の素数さん:2008/03/10(月) 02:28:38
f(z) = 1 + z^2/3 + z^4/5 + z^6/7 + ... は f(2z/(z^2+1))=(z^2+1)f(z) を満たすことを示せ
763132人目の素数さん:2008/03/10(月) 03:45:29
|z|<1でf(z)=(1/(iz))arctan(iz)
764132人目の素数さん:2008/03/10(月) 20:19:55
情報学板にも同じ問題出したんだが華麗にスルーされたんでこっちにもマルチさせてくれ。
多分面白い問題だと思う。


グラフ同型判定問題について次のような判定方法を考える。

グラフG={V,E} V={v_1,v_2,...v_n}とおく。
各ノードv_iに対して次のような関数を用意する。
f(G,v_i,1)=1
f(G,v_i,k+1)=f(G,v_i,k)+Σ_{v_j|v_jはv_iの隣接点} f(G,v_j,k)

あるkに対してGの全てのノードに対するf(G,v_i,k)(0<=i<=n)を昇順にソートした列をg(G,k)と置く。

2つの与えられたグラフG,G'に対して、kをG,G'のサイズの多項式倍以下の自然数とする。
全てのkに対しg(G,k)とg(G',k)が同じならGとG'は同型であると判定する。
g(G,k)とg(G',k)が異なるkが存在するなら同型で無いと判定する。


問1
この判定方法で同型で無いと判定された場合は実際にGとG'は同型で無いことを示せ。

問2
この判定方法で、同型であると判定されるにもかかわらず実際は同型ではないグラフG,G'を具体的に一組挙げよ。
765132人目の素数さん:2008/03/10(月) 20:56:16
宿題は自分でやれよ
766764:2008/03/10(月) 23:17:39
>>765
でたな、宿題厨。
俺は大学なんぞもうとっくに卒業してるよ。
三十路のいい年こいたおっさんだよ。

ていうか、これ宿題レベルなのか?
結構難しいと思ってたんけど。
767132人目の素数さん:2008/03/10(月) 23:37:13
それなりに有名な同型性判定のヒューリスティクスの一つ
768132人目の素数さん:2008/03/11(火) 16:40:24
3-cycle2つと6-cycleでいい気がする
同じ頂点数、同じ次数の正則グラフを2つもってくれば十分なんじゃない?
769764:2008/03/11(火) 19:06:03
>>768
う、そんな穴があったか。
これじゃ宿題レベルといわれてもしょうがないか。
正直すまんかった。
770132人目の素数さん:2008/03/11(火) 20:47:50

今日も元気だ、煙草がうまい。

とは、真か偽か ?
771132人目の素数さん:2008/03/11(火) 21:30:59
客観性に欠ける主張のため命題とはいえません
よって真偽判断以前の問題です
772132人目の素数さん:2008/03/11(火) 23:46:45
文意の曖昧さをなくすために原文を
「煙草をうまく感じた。 ゆえに今日も元気だということだ。 元気でないならばタバコはうまくないからなあ… 」
という意味だと解釈し
「煙草がうまいならば元気(体機能の健康で正常な状態)である。」と言い替える。

この場合、煙草がうまくないならば命題は常に真である。

さて、さて、客観的にタバコはうまいのか?
煙草にはうまみの成分は含まれていない。
含まれていないものをうまいと感じるのは、
うまみを感じている器官が麻痺するなど正常な状態でなく
なにか勘違いをしているかいということだ。

勘違いをしているなら、うまいと感じたことは間違いであり
実際にはうまくないのだから命題は真。

∴命題は常に真であった
773132人目の素数さん:2008/03/12(水) 02:39:37
pを奇素数とする。正整数nに対し、
1^n + 2^n + ‥ + (p-1)^n をpで割った余りの値を決定せよ。
774132人目の素数さん:2008/03/12(水) 11:24:14
pの原始根を使えばmod pで等比数列の和
775132人目の素数さん:2008/03/12(水) 14:52:16
nがpの倍数のとき、p-1
それ以外のとき、0
776132人目の素数さん:2008/03/12(水) 16:15:59
>>775
nがp-1の倍数のとき、じゃね?
777132人目の素数さん:2008/03/12(水) 17:48:41
円をいくつかに切り分けて同じ面積の正方形に繋ぎ変える事はできるか
778132人目の素数さん:2008/03/12(水) 18:01:02
出来ない
779132人目の素数さん:2008/03/12(水) 20:51:54
切り方にルールが要るんじゃないだろうか
780132人目の素数さん:2008/03/12(水) 21:30:05
選択公理を使えば出来るんじゃなかったっけ。
781132人目の素数さん:2008/03/12(水) 22:51:22
int(y)をyを超えない最大の整数とする

x=8/33=0.2424242424....の時、
int(x*10)=2、int(x*100)=24、int(x*1000)=242…は何れも素数ではない

このようなxで0以上1以下の数の集合
A = {x∈[0,1] | int(x*10)、int(x*100)、int(x*1000)…が何れも素数でない}
の測度は幾つか?
782132人目の素数さん:2008/03/13(木) 00:17:40
2は素数だし
783781:2008/03/13(木) 00:57:46
0.42424242...に変えといてくれ…
784132人目の素数さん:2008/03/13(木) 01:00:52
これ答え有るの?
10進展開と絡んでて難しそうなんだけど
785132人目の素数さん:2008/03/13(木) 01:15:21
>>775-776
どっちも微妙に変じゃね?
786132人目の素数さん:2008/03/13(木) 03:07:13
>>785
ん? >>776は別に変じゃない気がするが。
787785:2008/03/13(木) 20:15:57
すまん、当方の勘違いだった。>>776でおk。
788132人目の素数さん:2008/03/14(金) 13:58:58
>>777-780
Tarski's circle-squaring problem ですね。
789132人目の素数さん:2008/03/14(金) 20:55:05
A,Bの2人はともに100円持っており、それを賭けて次のようなゲームをする。
まず2人は同時にそれぞれ賭け金を宣言する。
賭け金は手持ちの金額以内で、1円単位とする。
A,Bの賭け金がそれぞれa,b円のとき、
a/(a+b)の確率でA、b/(a+b)の確率でBの勝ちとして、
勝者は相手の賭け金をもらう。
これを繰り返しおこない、どちらかの持ち金が無くなった時点で終了とする。
このゲームでの最善の戦略は?
790132人目の素数さん:2008/03/14(金) 21:14:34
>>789
破産したら終了という条件がなければ、
1回の勝負の利益の期待値はどうやっても±0なので、
なるべく破産を回避する戦略が最善と思われる。
結局、両者とも1円ずつ賭けるようになるんじゃね?
791132人目の素数さん:2008/03/14(金) 22:18:40
常に1円かけるプレーヤーと常に全額かけるプレーヤーが戦ったらどっちが勝率いい?

792132人目の素数さん:2008/03/14(金) 22:24:32
>>791
A:常に1円、B:常に全額
Bが100連勝したらBの勝ち、それ以外ならAの勝ち
Bの勝つ確率
(100/101)*(101/102)*…*(199/200)=1/2
やっぱ五分五分か…
793132人目の素数さん:2008/03/15(土) 18:30:02
面白くないかもしれないけどこの問題いいですか?
サバイバルゲームを1地区、2地区のどちらかで行なうものとする
1地区は単位面積当たり一般人24人、殺し屋1人
2地区は単位面積当たり一般人98人、殺し屋2人
できるだけ殺し屋に遭遇しないためには1地区、2地区のどちらに行けばいいか?
ちなみに殺し屋に遭遇したら即殺されるのでなくこちらも相応の武器と術を持つ
解をよろしくお願いします
794794:2008/03/15(土) 18:33:07
794 年 平安京に遷都
795132人目の素数さん:2008/03/15(土) 18:49:26
単純に1地区は遭遇確率が1/25で2地区は2/100という答えじゃだめ?
796132人目の素数さん:2008/03/15(土) 19:03:01
>>793
マルチ
797132人目の素数さん:2008/03/15(土) 19:11:04
>>795
何十ものスレにマルチしてる>>793にはエサをやらない
798132人目の素数さん:2008/03/15(土) 23:38:29
>>769
しっかりしろよ、もっさん!
799132人目の素数さん:2008/03/16(日) 12:51:23
2人で次のようなゲームをおこなう。
まず1からnまでの数が書かれたカード計n枚を場に並べる。
2人が交互に場のカードを取っていく。
カードの取り方は、場にあるカードから1枚選び、
そのカードに書かれた数の約数のカードを全て取る。
最後のカードを取った方が勝ち。
双方が最善を尽くす場合、後手必勝となるnはあるか?
800132人目の素数さん:2008/03/16(日) 13:39:48
そのようなnはない
なぜなら、そのようなnがあったとすると先手か1だけを取った時にも後手がかつが、
後手が次の一手でつくりうるどんな状態も先手は一手目で作れたはずなので矛盾する
801132人目の素数さん:2008/03/16(日) 14:23:03
>>800
正解。
チョンプっていうゲームをアレンジして問題を作ってみた。
ちなみに最後のカードを取った方が負けとしても証明は同じ(n≧2のとき)。
先手必勝と分かっても具体的な手順は分からない不思議。
802132人目の素数さん:2008/03/16(日) 14:44:23
「1だけを取り除く」という行為は初めの1手目にしかできず、先手に与えられた特権。そして、
これは「1手目をパスする」という行為と同じ(どのカードを選んでも1が一緒に取り除かれるので)。
ゲーム理論のことは全然知らないが、2人で行う、偶然に左右されないゲームで、
・先手は、初めの1手目に限ってパスをしてもよい
・後手はパスできない
という条件があったら、>>800と同じ方法で先手必勝だと言えるのではなかろうか。
803132人目の素数さん:2008/03/16(日) 14:49:49
それはつまり、片方のプレーヤーに、
先手か後手かを選ぶ特権が与えられてるのと同じだね。
そのプレーヤーが必勝なのは当然。
804132人目の素数さん:2008/03/16(日) 22:29:28
ABCDEFGHIJに全て違う正の整数(何桁でも可)を入れて等式を成立させて下さい
同じ整数2度使いは不可

Aの9乗+Bの9乗+Cの9乗+Dの9乗=Eの9乗+Fの9乗+Gの9乗+Hの9乗+Iの9乗+Jの9乗
805132人目の素数さん:2008/03/17(月) 08:22:49
>>804
マルチ
806132人目の素数さん:2008/03/18(火) 16:39:41
座標平面の原点(0,0)に駒を1つ置き、
次のルールに従って駒の配置を変化させていくとする。

ルール:
平面上に置かれている駒(座標(x,y))を1つ選び、その駒を取り去り、
新たに2つの駒をそれぞれ(x+1,y)と(x,y+1)に置く。

このゲームの目的は、0≦x≦2, 0≦y≦1の範囲から駒を追い出すことである。
果たしてそれは可能だろうか?
807132人目の素数さん:2008/03/18(火) 16:40:52
>>806
ルール追加。
(x+1,y)または(x,y+1)のどちらかに既に駒が置かれている場合は、
(x,y)の駒を選ぶことはできない。
808132人目の素数さん:2008/03/18(火) 17:27:14
既出だったような気がするな。 うち帰ったら過去ログ見てみるわ。
809132人目の素数さん:2008/03/18(火) 18:13:20
既出。

最初の駒を重さ1の粘土だと思い、操作一回につき、粘土を半分に分けると
思うことにする。このとき、何回操作を繰り返しても平面上の粘土の重さの
和は1であり、もし、0≦x≦2, 0≦y≦1の範囲から駒を追い出せたとすると
重さの和が1より小さくなって矛盾。
810132人目の素数さん:2008/03/18(火) 19:44:12
じゃあ、(0,0),(1,0),(2,0),(1,0),(1,1)に駒が無いようにすることは可能か?
811132人目の素数さん:2008/03/18(火) 20:26:26
(0,0),(1,0),(2,0),(0,1),(1,1)?
812132人目の素数さん:2008/03/18(火) 20:59:06
>>810
(0,0),(1,0),(2,0),(1,0),(1,1)でも
(0,0),(1,0),(2,0),(0,1),(1,1)でも
可能だった。
813132人目の素数さん:2008/03/18(火) 21:00:55
あ。間違い発見> <
(0,0),(1,0),(2,0),(1,0),(1,1)は可能だけど、
(0,0),(1,0),(2,0),(0,1),(1,1)は まだ分からない。
814132人目の素数さん:2008/03/18(火) 21:08:49
(0,0),(1,0),(2,0),(0,1)(0,2)
から追い払って(1,1)に残すことは可能だから
不可能だとすれば重さだけで示すことはできないね
難問かも
815132人目の素数さん:2008/03/18(火) 21:30:17
(0,0),(1,0),(2,0),(0,1),(1,1)も不可能だと分かった。やっぱり重さで計算する。
816132人目の素数さん:2008/03/18(火) 21:37:46
どうやんの?
817132人目の素数さん:2008/03/18(火) 23:50:12
>>816
計算ミスってた(^o^)
可能だとしたら、駒のあるべき場所が段々絞り込めて来るが(例えば、
可能だとしたら、(0,2)には絶対に駒がなくてはいけない)、まだ
解けない。
818132人目の素数さん:2008/03/19(水) 02:51:31
>>812-813
>(0,0),(1,0),(2,0),(1,0),(1,1)は可能
(1,0)が2回重複してるからよく分からんが、
要は
>(0,0),(1,0),(2,0),(1,1)は可能
だと言いたいんだよね
819132人目の素数さん:2008/03/19(水) 04:38:26
>>818
YES.


(0,0),(1,0),(2,0),(0,1),(1,1)がもし可能ならば、(0,2)と(3,0)には
駒がなければならない(重さを精密に計算するとそうなる)。この結果を
使って、初めの(0,0)の状態から地道な場合分けで何回か操作をして
いくと、どうも

↑↑
■■→
■■■→

こういう配置がどこかに出てくる(矢印は進行方向)。そして、
この配置において左下の駒を消さなければならなくなる。左下というのは、

↑↑
■■→
☆■■→

☆の場所のこと。で、これが出来そうにない。かと言って、☆から駒を
消せたとして重さを計算しても、詰めが甘いのか矛盾が導けない( ^ω^)
820132人目の素数さん:2008/03/19(水) 22:54:57
>>804
(9,4,6) equaion
念のため J.Wroblewski による解を…

・18th solution to (9,k,10-k) equation: 2006/10/03.
 (A,B,C,D) = (2152, 979, 752, 514), (E,F,G,H,I,J) = (2101, 1793, 1021, 870, 750, 112),
http://euler.free.fr/ … Computing minimal equal sums of like powers.

・21st solution to (9,k,10-k) equation: 2007/02/28.
 (A,B,C,D) = (2137, 1044, 704, 178), (E,F,G,H,I,J) = (2107, 1631, 1464, 428, 340, 223),

・22nd solution to (9,k,10-k) equation: 2007/05/26.
 (A,B,C,D) = (2210, 1792, 1614, 1503), (E,F,G,H,I,J) = (2243, 1680, 1377, 819, 471, 49),
http://www.math.uni.wroc.pl/~jwr/eslp/tables.htm … Equal sums of powers - Ttables
821132人目の素数さん:2008/03/20(木) 14:30:29
なんか頭がパーンって。スッキリした。
http://www.nicovideo.jp/watch/sm2516171
822132人目の素数さん:2008/03/20(木) 21:27:21
>>804
-----------------------------------------------------------------------------------
        ( A, B, C, D), ( E, F, G, H, I, J),
-----------------------------------------------------------------------------------
 1st solution, ( 90, 64, 35, 35), ( 86, 80, 62, 43, 27, 16), *2度使い=35
 2nd solution, ( 108, 93, 66, 15), ( 104, 100, 79, 29, 20, 10),
 4th solution, ( 219, 155, 108, 13), ( 211, 174, 174, 165, 90, 11), *2度使い=174
 5th solution, ( 535, 272, 253, 75), ( 491, 487, 397, 374, 292, 174),
 6th solution, ( 613, 604, 467, 299), ( 648, 536, 360, 355, 272, 172),
13th solution, (1780,1410, 352, 183), (1724,1516,1374,1207,1060, 324),
15th solution, (1951,1538, 815, 209), (1960,1370,1241,1143,1103,1026),
16th solution, (1979,1303,1081, 381), (1835,1816,1441, 993, 603, 306),
18th solution, (2152, 979, 752, 514), (2101,1793,1021, 870, 750, 112), ↑
21st solution, (2137,1044, 704, 178), (2107,1631,1464, 428, 340, 223), ↑
22nd solution, (2210,1792,1614,1503), (2243,1680,1377, 819, 471, 49), ↑
------------------------------------------------------------------------------------
http://www.math.uni.wroc.pl/~jwr/eslp/955.txt による。
823132人目の素数さん:2008/03/21(金) 23:04:04
□に全て違う数字を入れて成立する組み合わせは全部で何通りあるか?

□□□+□□□+□□□=2003
824132人目の素数さん:2008/03/22(土) 00:51:55
〔類題〕
ABCDEFGHIJ に全て違う正の整数(何桁でも可)を入れて↓の3式を同時に成立させて下さい。
同じ整数2度使いは不可。

 A^9 + B^9 + C^9 + D^9 + E^9 = F^9 + G^9 + H^9 + I^9 + J^9,
 A^3 + B^3 + C^3 + D^3 + E^3 = F^3 + G^3 + H^3 + I^3 + J^3,
 A  + B  + C  + D  + E  = F  + G  + H  + I  + J,
825824:2008/03/22(土) 02:46:14
>824 の 参考
 A^3 + …… + E^3 =     F^3 + …… + J^3 = 545232080,
 A^2 + …… + E^2 = 985730, F^2 + …… + J^2 =   978260,
 A^1 + …… + E^1 =     F^1 + …… + J^1 =    1940,
 A^0 + …… + E^0 =     F^0 + …… + J^0 =     5,
でも、そんなの関係ねぇ!!(?)
826132人目の素数さん:2008/03/22(土) 15:33:22
 □□□□□
― □□□□
33333

□の中には1〜9までの数が必ず一つずつ当て嵌められる。
二通りの答えがあるらしいんだが、解法も合わせて教えてくれ
827132人目の素数さん:2008/03/22(土) 17:57:13
各アルファベットに全て違う正の整数(何桁でも可)を入れて等式を成立させて下さい
同じ整数2度使いは不可
(なるべく値が小さくなるように)

(1)Aの8乗+Bの8乗+Cの8乗+Dの8乗+Eの8乗+Fの8乗=Gの8乗+Hの8乗+Iの8乗+Jの8乗+Kの8乗+Lの8乗

(2)Aの6乗+Bの6乗+Cの6乗=Dの6乗+Eの6乗+Fの6乗

(3)Aの6乗+Bの6乗+Cの6乗+Dの6乗=Eの6乗+Fの6乗+Gの6乗+Hの6乗

(4)Aの5乗+Bの5乗+Cの5乗=Dの5乗+Eの5乗+Fの5乗

(5)Aの5乗+Bの5乗+Cの5乗+Dの5乗=Eの5乗

(6)Aの4乗+Bの4乗+Cの4乗=Dの4乗+Eの4乗+Fの4乗=Gの4乗+Hの4乗+Iの4乗

(7)Aの4乗×2+Bの4乗=Cの4乗+Dの4乗+Eの4乗

(8)Aの4乗+Bの4乗+Cの4乗=Dの4乗

(9)Aの4乗+Bの4乗+Cの4乗+Dの4乗=Eの4乗

(10)Aの4乗+Bの4乗=Cの4乗+Dの4乗

(11)AのK乗+BのK乗+CのK乗+DのK乗=EのK乗+FのK乗+GのK乗+HのK乗 (K=1,3,7)
828132人目の素数さん:2008/03/22(土) 19:33:15
>>726 レイトン乙。
最上位の桁としてありうるのは3 (千の位で繰り下がりがない) または4 (繰り下がりがある)
また、一般に、引き算の結果ある桁が3になるのは、
(a)自分の桁も下の桁も繰り下がり無しで3になる場合…上=下+3 (例:6-3)
(b)自分の桁だけで繰り下がりありで3になる場合…上=下-7 (例:11-8の1の位)
(c)下のj桁だけで繰り下がりがある場合…上=下+4 (例:61-28 の十の位)
(d)自分と下の桁両方で繰り下がりがある場合… 上=下-6 ( 112-79の十の位)

これらのうち(a)に注目する。この組み合わせは、6-3、7-4、8-5、9-6。これらが
最大限使える組み合わせからとりあえず潰していくことにする。
3が最上位に入ったとすると、上記組み合わせで使えるのは
7-4,8-5,9-6、残ったのは1と2で、これは(a)〜(d)のどれにも当てはまらない。

4が最上位だとすると、上記組み合わせで使えるのは8-5と6-3or9-6。
残るのは1,2,7, (3or9) で、2と9が(b)のパターンを作れる。ここで繰り下がりが
生じるから、残った1と7が(d)のパターンを作れ、全て使い切れる。

従って、(1)2-9=3が3が百の位、(1)1-(繰り下がりの1)-7=3が千の位、
万の位が4になるようにすればすべて当てはまって、
41286-7953 または 41268-7935。
一つあるいは二つ見つければいいならこれで完了。
829132人目の素数さん:2008/03/22(土) 20:08:51
aのパターンでなぜか4-1と5-2を見落としていたことに気づいて愕然orz

でもまあこんな感じ。下の桁に繰り下がりなしが続き、あるところから
繰り下がりが発生すると仮定して(根拠はないが、とりあえず対象を
絞るために)、その中で探すというのが大方針。

もちろん、全ての解を探すためにはこれでは不十分だけど。

830132人目の素数さん:2008/03/22(土) 20:30:02
ありがとんくす!
全ての解を書き出す方法はちょっと自分でも考えてみます!
831132人目の素数さん:2008/03/22(土) 21:23:18
問題

@春夏A秋B

さて、数字が入る○は、どれでしょう。

{都内有名私立小学校入試問題}


って、うそ
832132人目の素数さん:2008/03/22(土) 22:25:00

ヒント

明日が千秋楽です〜
833132人目の素数さん:2008/03/22(土) 22:29:49
閣下、もう少しマシなヒントいえ問題をお願いします
834132人目の素数さん:2008/03/22(土) 23:12:53
>>827
(1) (8;6,6)
 (A,B,C,D,E,F) = (23,15,10,8,6,3) (G,H,I,J,K,L) = (22,20,12,9,9,5) (Moessner & Golden 1994) *2度使い=9, 不可。

(2) (6;3,3)
 (A,B,C) = (23,15,10), (D,E,F) = (22,19,3)  (Subba-Rao 1934)
 http://www.math.uni.wroc.pl/~jwr/eslp/633.txt

(3) (6;4,4)
 (A,B,C,D) = (9,9,2,2) (E,F,G,H) = (10,6,5,3)  (Moessner 1947)  *2度使い=9,2, 不可。しかし
  (6;4,4,4)
 (A,B,C,D) = (111,49,34,1) (E,F,G,H) = (110,69,43,7) (I,J,K,L) = (109,77,25,18) (Lander et al.1967)

(4) (5;3,3)
 (A,B,C) = (67,28,24) (D,E,F) = (62,54,3)
 (A,B,C) = (66,44,18) (D,E,F) = (64,51,13)
 (A,B,C) = (74,43,21) (D,E,F) = (68,62,8) ほか多数

(5) (5;1,4)
 (A,B,C,D) = (133,110,84,27) E = 144  (Lander, Parkin and Selfridge 1966)
 (A,B,C,D) = (85282,28969,3183,55) E = 85359  (J.Frye 2004)
835132人目の素数さん:2008/03/22(土) 23:15:04
>>827

(7) (4;3,3) with (A,A,B) = (C,D,E)
 A = 2, B = 13 (C,D,E) = (12,9,6)  (Ramanujan)

(8) (4;1,3)
 (A,B,C) = ( 414560, 217519, 95800) D =  422481  (R.Frye  1988)
 (A,B,C) = (18796760,15365639,2682440) D = 20615673  (N.Elkies 1987) ← Euler予想の反例
 http://euler.free.fr/database.txt

(9) (4;1,4)
 (A,B,C,D) = (272,315,120,30) E = 353
 http://www.math.uni.wroc.pl/~jwr/eslp/414.txt

(10) (4;2,2)
 (A,B) = (158,59) (C,D) = (134,133)  (L.Euler, 1772)
 (A,B) = (239, 7) (C,D) = (227,157)
 http://www.math.uni.wroc.pl/~jwr/422/index.htm

(参考)
 http://mathworld.wolfram.com/DiophantineEquation*thPowers.html (*=4,5,6,8)
 http://euler.free.fr/database.txt
836132人目の素数さん:2008/03/22(土) 23:30:45
>>824 ↓の中にあるらしい…

http://www.math.uni.wroc.pl/~jwr/eslp/955.txt
837132人目の素数さん:2008/03/23(日) 05:17:26
>>826
 41268-7935 と 41286-7953
らしい…
838132人目の素数さん:2008/03/23(日) 05:35:21
>>823

2003 を9で割ると5余る。したがって、0〜9 から「4」を除いた9つの数字を使う。
∴ 1728とおり。
839132人目の素数さん:2008/03/23(日) 15:36:22
(10^30)/1002を小数で表したとき10の位の数字を求めよ
(数検過去問の変形)
840132人目の素数さん:2008/03/23(日) 17:01:59
>>839
まさか元の分母は2002だった、って言うんじゃないよね。
841132人目の素数さん:2008/03/23(日) 17:11:16
>>831
正解は、B

初春夏名秋九
場場場古場州
所所所屋所場
   場 所
   所
842132人目の素数さん:2008/03/23(日) 17:26:19
>>839
元の分母も1002だが求める物が1の位だった。
843132人目の素数さん:2008/03/23(日) 17:49:27
10^6=1002*998+4 より、
10^24=1002*x+4^4 = 1002*x+256 (24/6=4より)
10^30=1002*x*10^6 + 256*10^6
=1002*x*10^6 +256*998*1002 + 1024

従って、求める商の下6桁は 256*998+1 (←1024/1002)

で、十位が8で一位が9?
844132人目の素数さん:2008/03/23(日) 18:56:52
>>843
当たり
845132人目の素数さん:2008/03/24(月) 03:05:03
>839,842

 (N^10) /(N+2) = N^9 -2N^8 +4N^7 -8N^6 +16N^5 -32N^4 +64N^3 -128N^2 +256N -512 + 1024/(N+2),

ここで N=10^3 とおくと、
 (10^30)/1002 = 10^27 -2・10^24 +4・10^21 -8・10^18 +16・10^15 -32・10^12 +64・10^9 -128・10^6 +256・10^3 -511 + (22/1002)
        = 998 003 992 015 968 063 872 255 489 + (22/1024).
846132人目の素数さん:2008/03/24(月) 19:31:27
>>827 (11)
 http://euler.free.fr/ にあるらしい…

 (A,B,C,D) = (33704,32317,14977, 9803) (E,F,G,H) = (33450,32630,14664,10057)
 (A,B,C,D) = (76925,52473,50279,15187) (E,F,G,H) = (74895,64419,27857,27693)
 (A,B,C,D) = (58711,42312,38285, 9544) (E,F,G,H) = (56170,51782,24427,16473)
847132人目の素数さん:2008/03/25(火) 08:23:02
(1)正方形を全て大きさが異なる直角二等辺三角形に分割して下さい
但し切り分け回数はなるべく少なくして下さい

(2)87×14の長方形を全て大きさが異なる直角二等辺三角形に分割して下さい
但し切り分け回数はなるべく少なくして下さい
848132人目の素数さん:2008/03/25(火) 10:21:41
>>847
とりあえず(1)だけ。絶対無理と思ったが、なんか出来たw
(これが最小回数かどうかはわからん)
A(0,10),B(0,0),C(10,0),D(10,10)
の正方形ABCD
P(4,0),Q(4,6),R(0,2),S(1,3),T(2,2)
△DAC:斜辺=10
△PCQ:斜辺=6
△QAR:斜辺=4√2
△SPQ:斜辺=3√2
△TBP:斜辺=2√2
△RBT:斜辺=2
△STR:斜辺=√2
の7個
849132人目の素数さん:2008/03/25(火) 11:53:50
>>847
(2)の答えは、ぐぐったら見つかった。すげーな
850132人目の素数さん:2008/03/25(火) 12:41:02
>>849
詳しく
851132人目の素数さん:2008/03/25(火) 12:52:03
>>850
正確には、ぐぐった、ではなく、Yahooで検索した、だな。googleではなぜか見つからなかった。
キーワードは「直角二等辺三角形 87×14」
パズル系の問題をコレクションしてるページが2番目ぐらいに見つかるはず。
852132人目の素数さん:2008/03/25(火) 16:24:05
「長方形を全て大きさが異なる直角二等辺三角形に分割」でググったら一発
853132人目の素数さん:2008/03/27(木) 23:13:24
∠□

↑こんな感じの正方形と直角二等辺三角形がくっついた図形を合同な形に二分割する事はできるか
(ただしフラクタル不可)
854132人目の素数さん:2008/03/27(木) 23:17:46
カバディをやりながら円周率を言い続けた男がギネスブック申請へ
http://namidame.2ch.net/test/read.cgi/poverty/1206543035/
855132人目の素数さん:2008/03/27(木) 23:18:30
>(ただしフラクタル不可)
ちっ 釘をさされたか
856132人目の素数さん:2008/03/28(金) 01:48:57
>>855
フラクタルなら可なの?
857132人目の素数さん:2008/03/28(金) 03:48:31
>>856
いや、この手の問題で、境界がフラクタルになるのを許すなら、
別解が存在するってのがよくあるので、ひねくれてまずそっちから考えようと
思っただけ。今回はそれも無理っぽいな。
858132人目の素数さん:2008/03/29(土) 07:58:23
□━□━□
┃ ┃
□━□━□

一つまたはいくつかの隣り合った数の合計が1〜31まで全て表せる様に□に1〜10の整数を入れて下さい
同じ数字2度使いはNG
(組み合わせを全て求めて下さい)
859132人目の素数さん:2008/03/29(土) 12:44:25
>>858
1,3,2,7,8,10 のみ
入れ方は12通り
860132人目の素数さん:2008/03/29(土) 21:57:18
1gの硬貨の山
2gの硬貨の山
3gの硬貨の山
4gの硬貨の山
5gの硬貨の山
の5つの山abcdeがあります

(1)1kg迄測れる秤を1回だけ使い各山が何gの硬貨の山かを当てて下さい

(2)何kgでも測れる天秤を使い各山が何gの硬貨の山かを当てて下さい(ただし最短回数で)
861132人目の素数さん:2008/03/29(土) 23:18:28
>>860
(1)に関しては、
「1g単位で重さを量れる秤を1回だけ使い、各山が何gの硬貨の山かを確実に特定するために、
最小限必要なその秤の最大測定グラム数を答えよ」
の方が面白いかも。
ちなみに、742gではない。
862132人目の素数さん:2008/03/29(土) 23:23:50
どうやると742gになるの?
863132人目の素数さん:2008/03/29(土) 23:58:07
山aから125枚
山bから25枚
山cから5枚
山dから1枚
を量った時の、ワーストケース。
864132人目の素数さん:2008/03/31(月) 17:18:06
?に任意の整数を、□に1〜8までの数字を1つずつ入れ等式を成立させて下さい
(全通り求めて下さい)

(1) ???÷?=□□.□□□□□□……
(2) ?????÷?=□□□□□.□□□
865132人目の素数さん:2008/03/31(月) 17:28:11
>>853
ここにフラクタルの分割らしきものが載っているが
http://www.mathpuzzle.com/Ltrom.html
866132人目の素数さん:2008/03/31(月) 18:06:59
>>865
それ相似であって合同ではないと思うんだが
867132人目の素数さん:2008/03/31(月) 19:46:39
>>853が実は「合同」ではなく「相似」と言いたかったのであれば、がっかりだ
868132人目の素数さん:2008/03/31(月) 20:07:23
しかし言い間違えたとしたら、相似に分割は問題として簡単すぎないか?
869132人目の素数さん:2008/04/03(木) 10:21:38
面白問題の宝庫スレ
http://c.2ch.net/test/-/jsaloon/1202402941/i
870132人目の素数さん:2008/04/08(火) 08:44:53
□に1〜9迄の数字を1つずつ入れて成立する等式を全て求めて下さい

(□÷□□)+(□□÷□□)=□÷□
871132人目の素数さん:2008/04/08(火) 15:27:17
できれば、どこが面白いのか説明してほしい。
872132人目の素数さん:2008/04/08(火) 17:26:25
1/28 + 45/36 = 9/7    1/78 + 34/52 = 6/9
2/18 + 36/54 = 7/9    2/18 + 59/36 = 7/4
2/38 + 91/76 = 5/4    3/18 + 27/54 = 6/9
3/18 + 72/54 = 9/6    3/27 + 45/81 = 6/9
4/12 + 39/78 = 5/6    4/72 + 18/36 = 5/9
4/78 + 56/91 = 2/3    5/18 + 23/46 = 7/9
5/18 + 32/64 = 7/9    5/27 + 39/81 = 4/6
5/36 + 29/18 = 7/4    5/38 + 64/19 = 7/2
6/27 + 34/51 = 8/9    6/48 + 15/72 = 3/9
6/48 + 31/72 = 5/9    6/54 + 19/72 = 3/8
7/14 + 95/38 = 6/2    7/16 + 58/32 = 9/4
7/21 + 65/39 = 8/4    7/38 + 25/19 = 6/4
8/12 + 57/36 = 9/4    8/17 + 69/34 = 5/2
8/19 + 25/76 = 3/4    8/64 + 15/72 = 3/9
8/64 + 31/72 = 5/9    8/96 + 35/21 = 7/4
9/12 + 35/84 = 7/6    9/12 + 38/76 = 5/4
9/18 + 27/36 = 5/4    9/21 + 45/63 = 8/7
9/36 + 15/24 = 7/8    9/42 + 18/36 = 5/7
9/48 + 37/16 = 5/2    9/48 + 53/16 = 7/2
9/54 + 13/78 = 2/6    9/54 + 27/81 = 3/6
9/72 + 14/56 = 3/8
873132人目の素数さん:2008/04/08(火) 22:20:08

勉強する時間がなかったので、俺はこの試験に落ちた。

が、真とすれば、はたして彼は

勉強する時間が十分にあれば、試験に合格したか。

ただし、そのときの試験問題は 

彼が不合格したときの問題ではない。

そして、彼の性格は、なまけものである。
874132人目の素数さん:2008/04/09(水) 05:02:12
>>873
> 勉強する時間がなかったので、俺はこの試験に落ちた。

これを
「勉強する時間がなかった」⇒(ならば)⇒「試験に落ちた」
と解釈する。

一方
> 勉強する時間が十分にあれば、試験に合格した

「勉強する時間が十分にあった」⇒(ならば)⇒「試験に合格した」
である。
これは先の命題の裏である。

一般に、命題とその裏の真偽は一致しない。
875132人目の素数さん:2008/04/09(水) 07:42:09
真偽が一致するとは限らず、どうなるか分からないのに、
「勉強する時間が十分にあれば、試験に合格したに違いない」
と、裏の命題を「真」だと断定してしまうような行為を
皮肉っているだけではないのか?
876132人目の素数さん:2008/04/09(水) 08:22:52
だとしたら、余計な条件(最後の3行)を付けずにそのままでいいものを

余計なものをつけた分わかってないように見えてしまう。
877132人目の素数さん:2008/04/09(水) 10:59:58
>勉強する時間がなかったので、俺はこの試験に落ちた。
と言った場合,「勉強する時間がなかった」は真だから
>勉強する時間が十分にあれば、試験に合格した
は真じゃね.
878132人目の素数さん:2008/04/09(水) 12:52:03
「p⇒q」 がp=0のとき常に真とするのが気持ち悪い。
そもそも「p⇒q」はpが偽のときはどうなるか何も言ってないのだから
命題(常に真偽がはっきるするもの)とはいえないんじゃないの?
879132人目の素数さん:2008/04/09(水) 13:14:47
>>874
>> 勉強する時間がなかったので、俺はこの試験に落ちた。

>これを
>「勉強する時間がなかった」⇒(ならば)⇒「試験に落ちた」
>と解釈する。
それは、明らかに誤読だろう。
既に試験に落ちた状態で述べているのだから、この文は
「俺はこの試験に落ちた」という事実と、
「試験に落ちた原因は勉強する時間がなかったことだ」という2つのことを述べていると解釈される。
これだけでは、勉強を開始するより前の段階に視点を移して、
「勉強する時間がなければ、確実に試験に落ちる」と判断されると考えているのか、
「勉強する時間が十分にあれば、確実に試験に受かる」と判断されると考えているのかは
明らかでないし、どちらでもないという解釈の方が自然だが、
ニュアンスとしては前者よりも後者。

まあ、「彼の性格は、なまけものである」という文を
「試験に対して勉強する時間を十分に確保するということはありえない」と解釈するならば、
勉強を開始するより前の段階から見た
「勉強する時間が十分にあれば、試験に合格する」
という命題は、前提部分が恒偽だから真ということはできるかもしれない。

いずれにせよ、既に起きてしまったことについて、「そうでなかったらどうなるか」と考えることを
命題論理の範疇で捉えること自体無理がある希ガス
880132人目の素数さん:2008/04/09(水) 13:24:16
日本語としては、落ちた原因は勉強する時間がなかったこと“だけ”であると言っていると解釈できるけど、
数学的に考えると、落ちた原因の一つだということしかわからず、他にも原因があるのかないのかはわからないことになるんじゃ?
881132人目の素数さん:2008/04/09(水) 14:29:02
>>878
命題とは、「常に真偽がはっきるするもの」では無い。これは厳密な定義ではない。
ttp://ja.wikipedia.org/wiki/%E8%A8%BC%E6%98%8E ( [より厳密な定義] を参照のこと )
882132人目の素数さん:2008/04/09(水) 17:14:38
>>879
他の板ならともかく、ここは数学板なんだから
874が追加した公理を否定するなら
矛盾を導出するか、
もっと弱い公理で同じ結果が得られることを示すか、
またはせめて他の公理のほうが
おもしろい定理が導き出せることを示すべき。
883132人目の素数さん:2008/04/09(水) 17:47:04
線形時相論理
884132人目の素数さん:2008/04/09(水) 18:07:56
まあつまり
面白い問題だれかプリーズ
ということだ。
885132人目の素数さん:2008/04/09(水) 21:11:14
俺は貧乏だ。何故
886132人目の素数さん:2008/04/09(水) 21:23:23
数学なんてやってるからだろ
887132人目の素数さん:2008/04/10(木) 00:33:48
公理にワロタ
8881stVirtue ◆.NHnubyYck :2008/04/10(木) 12:39:13
Reply:>>886 お前はなぜ話をすりかえるのか。思考盗聴で個人の生活に介入する奴が工作しているからだ。
889132人目の素数さん:2008/04/10(木) 12:47:54
1が211個続いた数を素因数分解して下さい
890132人目の素数さん:2008/04/10(木) 12:54:37
面白くはない
891132人目の素数さん:2008/04/10(木) 15:15:17
>>889
こうですか?わかりません><
692624557324389620662782322677336711138108482588281739734375570506492391931849524636731866879
×1604204037181898492842452177634233120825494895604445254059369227570068074354992595031636365651567169241873842145514809
892132人目の素数さん:2008/04/10(木) 17:01:33
>>891
こうですか?わかりません><
3×31×37×41×43×71×101×239×271×1061×1933×4649
×123551×2906161×5051749×10838689×30703738801
×625437743071×102598800232111471
×57802050308786191965409441
×245391150214875249502685807421982149521
×75277048352808729626679875852901448034540173569840168229
893132人目の素数さん:2008/04/10(木) 17:55:17
3で割れるか?
894132人目の素数さん:2008/04/10(木) 18:41:07
>>892
それは
111111111111111111111111111111111111111111111111111111111111111111111111111
111111111111111111111111111111011111111111111111111111111111111111111111111
1111111111111111111111111111111111111111111111111111111111111
895132人目の素数さん:2008/04/11(金) 02:48:15
転載
一辺の長さが1の正三角形ABCの内部(境界を含む)に
三角形XYZ(退化した線分、点も許す)があるとき
3辺の長さx,y,z( 0≦x≦y≦zとする) が満たすべき三角不等式以外の条件を求めよ。

z=1 のときは x^2-y^2+y ≦1 ってのは出るけど。。
896132人目の素数さん:2008/04/11(金) 12:48:36
>>895
アイディアだけだけど、
XYZを動かしていくと、必ず
・3点が異なる辺上にある
・2点が同一の辺上にあり、もう1点が別の辺上にある
・1点が正三角形の頂点と重なり、もう1点がその頂点を含む辺上にある
のいずれかにできると思う。
897132人目の素数さん:2008/04/11(金) 19:53:04
どの2点間の距離も有理数である直方体の大きさは?
898132人目の素数さん:2008/04/12(土) 17:51:09
正方形をいくつかに分割して正七角形につなぎ変えて下さい
899132人目の素数さん:2008/04/12(土) 22:22:36
>>898
任意の直線のみで囲まれた図形は、いくつかに分割して重ならないように並べることで
正方形にすることができることを証明せよ。
900132人目の素数さん:2008/04/12(土) 22:36:26
>>898
分割してつなぎかえるだけなら可能だが
それをコンパスと定規で作図せよとか言われたら厄介だなw
901132人目の素数さん:2008/04/12(土) 22:46:19
>>900
ガウスさんでも無理w
902132人目の素数さん:2008/04/12(土) 22:54:30
>>897

645 名前:132人目の素数さん :2005/11/22(火) 23:03:25
どの2つの頂点間の距離も整数になっている直方体は存在するか?
存在するならどんな直方体か?

646 名前:132人目の素数さん :2005/11/22(火) 23:12:57
>645
それまだ未解決だと思う。(最近の発展は知らんけど)

647 名前:132人目の素数さん :2005/11/22(火) 23:38:23
>645
うん、未解決。半年前くらいの数セミにもあった。
903132人目の素数さん:2008/04/13(日) 08:13:15
6×6のマス目の内12個に色をつけて縦横対角線(全14列)で色がついたマスが2個なる組み合わせは全部で何通りあるか

□□□□□□
□□□□□□
□□□□□□
□□□□□□
□□□□□□
□□□□□□
904132人目の素数さん:2008/04/13(日) 08:23:33
>>902
こんなのが未解決なのか!!!!!!!!
905132人目の素数さん:2008/04/13(日) 09:22:42
そうなんだ
906132人目の素数さん:2008/04/13(日) 11:05:43
a^4+16
を因数分解せよ
907132人目の素数さん:2008/04/13(日) 11:43:50
>>906
バカマルチ
9081stVirtue ◆.NHnubyYck :2008/04/13(日) 13:21:26
思考盗聴で個人の生活に介入する奴は早く地球から去ったほうがよい。
909132人目の素数さん:2008/04/13(日) 13:52:52
>>907
はわわ〜
910132人目の素数さん:2008/04/13(日) 20:12:24
>906
 x^4 = -16
の4乗根をとると
 x = 2exp((2n-1)π/4) ≡ c_n, n=1〜4
よって
 (a-c_1)(a-c_2)(a-c_3)(a-c_4)
911132人目の素数さん:2008/04/13(日) 21:31:40
>>908
元祖マルチ
912132人目の素数さん:2008/04/14(月) 12:58:43
AB=3,AC=4,BC=5の直角三角形があります
BC上でBから3の点Pを中心として△ABCを時計と反対周りに90゚回転した三角形を△DEFとします
△ABCと△DEFの重なりの部分PQSRの面積はいくらになるでしょうか?
913132人目の素数さん:2008/04/14(月) 13:33:26
普通に1.44
914132人目の素数さん:2008/04/14(月) 17:52:18
中学生向けだな
915132人目の素数さん:2008/04/14(月) 23:16:04
>>2005/11/22(火)
>>2008/04/12(土)
916132人目の素数さん:2008/04/15(火) 00:37:12
>>915
2年半やそこらで未解決問題が解決するとでも?
917132人目の素数さん:2008/04/16(水) 14:51:57
1×1の正方形2004個でできた167×12の長方形がある
これをその正方形の辺にそって切り分けて三つの正方形につなぎ変えたい
切り分ける回数をできるだけ少なくするにはどう切り分ければ良いか?
918132人目の素数さん:2008/04/16(水) 15:22:24
lim(x→0)e^x-1/x=1
を証明してください(>_<)
9191stVirtue ◆.NHnubyYck :2008/04/16(水) 15:48:49
Reply:>>918 コンパクト閉集合の補集合全体を基本近傍系と考えるときの無限遠点。
920132人目の素数さん:2008/04/16(水) 21:53:43
a^5+b^5の因数分解の公式を教えてください
921132人目の素数さん:2008/04/17(木) 04:29:24
(a+b)で割れ。 それ以上はできん。
922132人目の素数さん:2008/04/17(木) 14:39:49
a^5+b^5 = (a+b)(a^2-((1-√5)/2)ab+b^2)(a^2-((1+√5)/2)ab+b^2)
923132人目の素数さん:2008/04/17(木) 17:09:37
>923
ありがとうございます!
924132人目の素数さん:2008/04/17(木) 17:14:34
>922でした。すいません
ありがとうございます!
925132人目の素数さん:2008/04/17(木) 23:06:15
>>921
(a+b)に誤れ! に見えた…
926132人目の素数さん:2008/04/19(土) 18:13:37
1kgの木球と10kgの鉄球(大きさは両方同じ)を、真空中の煙突の中で同じ高さから同時に落とした
どちらが先に地面に落下するか?(理由も)
(1)木球
(2)全く同じ
(3)鉄球
927132人目の素数さん:2008/04/19(土) 18:33:50
>>926
状況設定に依存する.
物理モデル(運動方程式)の設定が必要.
928132人目の素数さん:2008/04/19(土) 20:56:39
煙突の下のほうが低くなっていた場合、つまる
929132人目の素数さん:2008/04/19(土) 21:17:30
a1>0,a2>0,…,an>0,
(a1+a2)/2-√(a1a2)≧
(a1+a2+…an)/n-(a1a2…an)^(1/n)
930132人目の素数さん:2008/04/19(土) 22:36:37
真空に存在する以上、煙突が煙突としてのアイデンティティを失っている
931132人目の素数さん:2008/04/19(土) 22:44:56
>>929
a_1 = a_2 = 1, a_3 = 2
932132人目の素数さん:2008/04/20(日) 00:31:08
>>929
a>0,b1>0,…,bn>0,
(a+b1)-2√(ab1)≦(a+b1+b2+…+bn)-(n+1)(ab1b2…bn)^{1/(n+1)}
933132人目の素数さん:2008/04/20(日) 03:09:01
関数f:R^3 → R^2を考える。
任意のA,B,C∈R^3に対し、
|A-C|<|B-C| ならば |f(A)-f(C)|<|f(B)-f(C)|
が成り立つ。

このような、fは存在するか。
934132人目の素数さん:2008/04/20(日) 09:04:37
>>933
まず、fの存在を仮定すると、
fは単射ということはすぐ言える。
R^3上に点Pをとり、あるr(>0)についてPからの距離がrの点の集合をA_rとする。
以下、R^3上の集合Sに対して、{f(x)|x∈S}で表されるR^2上の集合のことをf(S)と書くものとすると、
f(A_r)の任意の要素とf(P)との距離の集合は、明らかに有界であり、
連続公理により上限U_rと下限L_rを持つ。
ここで、U_r=L_rと仮定すると
f(A_r)の要素は全てf(P)を中心とする半径U_rの円周上にある。
A_rの1つの要素をQとし、あるx(r/2≦x≦r)について、
A_rの要素のうち、Qとの距離がxであるものの集合をB_(r,x)とすると、
f(B_(r,x))の任意の要素とf(Q)との距離の集合の上限と下限が一致したら
f(B_(r,x))の要素が高々2個しかないことになり、矛盾。
よって、いかなるxについても,
f(B_(r,x))の任意の要素とf(Q)との距離の集合の上限と下限は一致しない。
・・・・・
ここから矛盾を導き、U_r=L_rの仮定を否定し、
そこからさらに矛盾を導き、fの存在を否定する。
・・・・・
簡単そうだと思ったのだが、ここで詰まってしまった...。
例えば、ある区間で定義された実関数で、あらゆる点で連続でなく、
なおかつ、単調増加であるものは存在しない、というのは、
どう証明すればいいんだっけ?
935132人目の素数さん:2008/04/21(月) 14:50:27
次の覆面算(足し算)を解いて下さい

INTO
ONTO
CANON
INTACT
AMMONIA
OMISSION
DIACRITIC
STATISTICS
ASSOCIATION
ANTIMACASSAR
CONTORTIONIST
NONDISCRIMINATION
CONTRADISTINCTION
━━━━━━━━━
MISADMINISTRATION
936132人目の素数さん:2008/04/22(火) 00:40:34
-------------INTO
-------------ONTO
------------CANON
-----------INTACT
----------AMMONIA
---------OMISSION
--------DIACRITIC
-------STATISTICS
------ASSOCIATION
-----ANTIMACASSAR
----CONTORTIONIST
NONDISCRIMINATION
CONTRADISTINCTION
━━━━━━━━━
MISADMINISTRATION

>>935
"-"は空白扱いとして、これをMSゴシックで表示した感じの覆面算を解けばいいのか?
2chの仕様のせいで935は無残な覆面算になってしまってるが
937132人目の素数さん:2008/04/22(火) 18:43:10
座標平面において、2点間の距離が1であるような2つの格子点を「隣接格子点」と呼び、
隣接格子点を結ぶ線分のみからなる閉曲線(ただし自分自身と交わらないもの)を
「格子閉曲線」と呼ぶことにする。
(1) 0≦x≦m, 0≦y≦2 の領域に格子閉曲線はいくつあるか。
(2) 0≦x≦m, 0≦y≦n の領域に格子閉曲線はいくつあるか。

適当に思いついただけなので答えは知らない。
938132人目の素数さん:2008/04/22(火) 20:38:35

「どこの誰だか知らないけれど、誰もがみんな知っている

月光仮面の おじさんは〜 ♪ 」

と、歌っているが、なぜ、月光仮面は「おじさん」

と、わかったのだろう ?

覆面とサングラスをしているのに ?
939132人目の素数さん:2008/04/22(火) 21:02:03

1. 加齢臭が漂っていたから。

2. バイクに乗るとき「よっこらしょ」と言っていたから。

3. 下腹が出ていたから。

4. ヘルメットの後ろに血液型と名前と年齢を書いていたから。


さて、正解は、何番 ?
940132人目の素数さん:2008/04/22(火) 22:55:03
              /
               〃  _, - ' ¨´  ̄ ̄ ``  、._
                ll /_,.             `ヽ、
           '⌒Y´.::           ヽ ヽ  ヽ \
             / .:/     .:/ ..:.:ji ヽ::. \::.\::. \ ヽ
              ,' .:/..:/.:/.:/.:/:.. .:/;ハヽ::.. ヽ:.ヽ::.ヽ::. ヽハ
               ! .:::i:...:l .:l .:l .:l .:l l jl lヽ ヽ ,ィ' 丁ヽヽヽi:l.::!
           |l :l:l:..::l .:l .:l .:l .:l ,.ィ,リ丁 \ヽ ,r=、!i::l.川:l.:|
           |l :l:l:..::l .:l .:l .:l,.イli l| _ノj    !l |i;l l |j ノソリソ
           |l :l:l:..::l .:l .:l .:l::リ jィ〒ミヽ   リ ゞ' いハ
           |l :l:l:..::l .:l .:l .:| 〈.イ;;:リ.i  ノ 、   |:.l川
           ハ;ハ:..:ヽ:ヽヽ::ヽ. ヾー '     ′ ハリノ
            乂ノヽ:.\:.ヽ、._ヽ__    -‐'  /Vl:|          頑張って
              弋_,ヽ. ヽ. \ヽ._     _/ }jリ        もうすぐ1000だよ      
                 _乂ノレヘ弋ト、ヽ>ーァ' ´ ̄ ̄`ヽ.
               __,.イレ′   ヽ //           ヽ
              /  !l       //             i
            /    !l      //           |
 
941132人目の素数さん:2008/04/23(水) 03:24:14
>>938
子供でも小母さんでもないから。
942132人目の素数さん:2008/04/23(水) 21:11:42
おじいさん かもしれないよっ
943132人目の素数さん:2008/04/24(木) 04:27:12
小父さん(おじさん) とは 大人の男性の意でもあるので
老齢者であっても小父さんと呼んで問題ない。
944132人目の素数さん:2008/04/24(木) 08:05:57
いや、きっとお兄さんだよ。
945132人目の素数さん:2008/04/24(木) 13:57:29
お兄さんもまた、子供でなければ小父さんとよんで問題ない。

ところで、お兄さんとは相対的なものである。
946132人目の素数さん:2008/04/25(金) 20:10:13
一匹のサルをつれた5人の男が難破して無人島にたどり着きました
5人は食料のココナッツを集めその日は寝ました
5人が寝静まった頃,1人が置きだして自分の取り分を隠そうと考えました
ココナッツを5等分したら1個余ったので,それをサルに与えて1/5を隠しました
やがて2人目も起きだして残りのココナッツを5等分したらまた1個余ったので,それをサルに与えて1/5を隠しました
さらに3人目も起きだして残りのココナッツを5等分したらまた1個余ったので,それをサルに与えて1/5を隠しました
さらに4人目も起きだして残りのココナッツを5等分したらまた1個余ったので,それをサルに与えて1/5を隠しました
さらに5人目も起きだして残りのココナッツを5等分したらまた1個余ったので,それをサルに与えて1/5を隠しました
朝になって5人は知らん顔でココナッツの前に集まり,それを5等分しました
また1個余ったので,それをサルに与えました
最初にあったココナッツは最低でも何個あったのでしょうか?
947132人目の素数さん:2008/04/25(金) 20:28:57
15621個
948132人目の素数さん:2008/04/25(金) 20:36:47
朝の1人の割当をn個とすると、
(((((5n+1)*(5/4)+1)*(5/4)+1)*(5/4)+1)*(5/4)+1)*(5/4)+1が最初の個数
これを計算すると(15625n+11529)/1024
15625≡11529≡265 mod1024
なので、元の個数を整数とするnの最小値は1023
949132人目の素数さん:2008/04/26(土) 02:15:14
>>934
>例えば、ある区間で定義された実関数で、あらゆる点で連続でなく、
>なおかつ、単調増加であるものは存在しない、というのは、
>どう証明すればいいんだっけ?

区間内で定義された単調関数は不連続点を高々可算個しか持たない事を示せばよい
950132人目の素数さん:2008/04/26(土) 04:14:55
>>949
それって、簡単なんですか?
951132人目の素数さん:2008/04/26(土) 10:26:16
>>950
簡単.
各不連続点 x に半開区間 (f(x)-ε(x), f(x)] を対応させる.
こうして作った半開区間は,相異なる不連続点に対して互いに素.
ところで任意の半開集合は有理数を含むので、不連続点→有理数の
単射が作れる.よって不連続点は有理数濃度=可算以下.
952132人目の素数さん:2008/04/26(土) 10:57:17
早稲田大学で出題された問題

5回に1回の割合で帽子を忘れるくせのあるK君が,
正月にA,B,Cの3軒を順に年始回りをして家に帰った時,帽子を忘れてきた事に気づきました
2軒目の家Bに忘れてきた確率はいくらでしょうか?
953132人目の素数さん:2008/04/26(土) 11:10:53
ただの条件付確率の計算のどこが面白いの?
954132人目の素数さん:2008/04/26(土) 11:13:35
20/61
955132人目の素数さん:2008/04/26(土) 14:09:51
>>951
なるほど、「任意の半開集合は有理数を含む」がキモなのですね。
どうもです。
956132人目の素数さん:2008/04/26(土) 14:57:01
?に入る記号は何か

×▲○┃▲○×┃▲×▲
▲○▲┃▲×▲┃○×○
××○┃×○○┃×○▲
━━━╋━━━╋━━━
×○▲┃×○▲┃×○▲
○××┃×▲○┃×○×
▲○▲┃○×▲┃▲▲○
━━━╋━━━╋━━━
▲××┃××○┃???
▲○○┃○×▲┃???
○×▲┃▲▲○┃???
957132人目の素数さん:2008/04/26(土) 15:04:55
>>956
パズルはパズル板でやれ
958132人目の素数さん:2008/04/26(土) 20:39:36
>>952
「どこで忘れたか」ということと「忘れたことに気付く」ということが
独立の事象である保証はないので、答えられません。
959132人目の素数さん:2008/04/27(日) 01:34:48
最初からかぶり忘れていた可能性も考慮しないとな。
960132人目の素数さん:2008/04/27(日) 02:43:28
与えられた点を通り、与えられた三角形の面積を2等分する直線を作図せよ。
961132人目の素数さん:2008/04/27(日) 02:53:06
角の三等分の問題に帰着できそうなできなさそうな。
962132人目の素数さん:2008/04/27(日) 10:15:14
963132人目の素数さん:2008/04/27(日) 15:05:20
>あらゆる点で連続でなく
>各不連続点 x に半開区間 (f(x)-ε(x), f(x)] を対応させる.
プギャーw
964132人目の素数さん:2008/04/27(日) 15:07:20
循環論法にすらなっていない...
965132人目の素数さん:2008/04/27(日) 17:50:04
>>962-964
もっと詳しくお願いします
966132人目の素数さん:2008/04/27(日) 18:18:12
不連続点の定義と関数の単調性とを正確に使うことで、
各不連続点に相異なる有理数を対応させることができる。
967132人目の素数さん:2008/04/28(月) 00:54:59
ある平面の有限グラフGを考える。このグラフには次のことが分かっている。
以下の問いに答えよ。

(1) 辺の数、節点の数が分かっている。
 このときGは一意に定まるとは限らないことを示せ。

(2) (1)に加え、任意の非負整数nに対しn本の辺が接続されている節点の個数v(n)が分かっている。
 このときGは一意に定まると言えるか。
968132人目の素数さん:2008/04/28(月) 00:56:49
>>967に加えて
(3) (1)に加え、グラフGには閉路がないことが分かっている。
 このときGは一意に定まるといえるか。
969132人目の素数さん:2008/04/28(月) 00:58:31
f(x-(1/n))とf(x+(1/n))は、fの単調性からそれぞれ有界な単調数列であるが
fの不連続性から、ともにf(x)に収束することは無い。
したがってI(x)=(limf(x-(1/n)),x)∪(x,limf(x+(1/n))≠φであるが
fの単調性より、これは異なるxにおいて互いに素。
970132人目の素数さん:2008/04/28(月) 02:10:12
>>967-968訂正
ある平面の有限な連結グラフGを考える。このグラフには次のことが分かっている。
以下の問いに答えよ。

(1) 辺の数、節点の数が分かっている。
 このときGは一意に定まるとは限らないことを示せ。

(2) (1)に加え、任意の非負整数nに対しn本の辺が接続されている節点の個数v(n)が分かっている。
 このときGは一意に定まると言えるか。

(3) (1)に加え、グラフGには閉路がないことが分かっている。
 このときGは一意に定まるといえるか。


---

連結性を忘れてた
971132人目の素数さん:2008/04/28(月) 07:50:25
用語に詳しくないのだが
YとZは閉路のない3辺4点って
ことでいいのかな?
972132人目の素数さん:2008/04/28(月) 07:51:41
>>970
「平面のグラフ」というのは平面グラフということ?
973132人目の素数さん:2008/04/29(火) 03:29:33
>>970
(1)、(2)、(3)いずれもGは一意に定まらない。
下の2つのグラフが、例の一つ。

○―○―○―○―○
        |
        ○―○

○―○―○―○―○
    |
    ○―○
974132人目の素数さん:2008/04/29(火) 04:05:55
位相幾何はよくわからん。
グラフの問題って973の2例は同じと見なす流儀ってなかったっけ?
なんか別の名が付いてたかな?
975132人目の素数さん:2008/04/29(火) 12:10:34
グラフ論でこの2例を同一視することは,まずないと思う.
976132人目の素数さん:2008/04/29(火) 14:05:13
かなり特殊な例になってしまうが
一筆書きとかの枝分かれにのみ着目したやつなら同一視だな。
977132人目の素数さん:2008/04/29(火) 17:31:01
それはどっちかつーと、とぽろじー的な見方だよね?
978132人目の素数さん:2008/04/29(火) 18:42:54
うん。トポロジーではわりとよくどちらもY型と同一視するかもしんない。
でも一筆書きっていえばトポロジーよりはグラフ理論よりのような気がする。

まあ、どこかで厳密に線が引けるものではなさそうだけど。
979132人目の素数さん:2008/04/29(火) 19:35:25
Yとの同一視はグラフの代数構造(辺-頂点の接続関係)を壊すので
普通のグラフ理論の枠組みでは、あんまりやらない。
先の2例の同一視は、グラフ理論的にやるなら
「次数2の頂点を縮約して作られるグラフが一致する」みたいにするけど
強く明記する必要があるタイプの同一視だと思う。

一方、グラフをユークリッド空間等に埋め込んでどうか、という分野:
位相幾何学的グラフ理論もあるのだけれど、そっちだとYとの同一視は
それなりに自然にやられる。
980132人目の素数さん:2008/04/29(火) 21:46:45
□に1〜9の整数を1つずつ入れ各列の合計を全て同じにして下さい(全5列)
(全通り求めて下さい)



│\
□※□
│※│\
□─□─□
│※│/
□※□
│/
981132人目の素数さん:2008/04/29(火) 21:54:53
>>980
その問題のどこが面白いのか教えてくれ
982132人目の素数さん:2008/04/29(火) 22:36:01
>>981
面白いの定義を教えてくれ
983132人目の素数さん:2008/04/29(火) 22:37:37
そうだよなあ。
誰も見向きもしないような分野を研究してる人だっているしな。
984132人目の素数さん:2008/04/29(火) 23:28:49
>>980
右端の1個を除いた5個の縦列と3個の縦列が等しいためには
右端の1個が奇数である必要あり。等しい和Sは、45からこの
右端の奇数nを引いた差を2で割った数。

この場合、nと他2つの9以下の自然数で和をSにする組み合わせが3通り必要。
n=1でS=22、S-n=21で、2つの1桁の自然数で和を21にはできない。
n=3でS=21、S-n=20で、同上。
n=5でS=20、S-n=15、このとき7+8、6+9の2組しか作れない。
n=7でS=19、S-n=12、このとき3+9、4+8の2組しか作れない。
n=9でS=18、S-n=9、このとき1+8、2+7、3+6、4+5の4通りがありうる。

3個の縦列が9を含まず和が18になる組み合わせは、873、864、765 の3組。
\・―・/の3列の区別、5個の縦列の2番目・4番目の区別は本質的な
差ではないとすると(これを差と見るとつまらなく組み合わせ数が
増えるだけ)、縦列が上記3組のいずれか、右端が9、残りは差から
埋めてできあがり、nとSによる検討で示したようにこれら以外はない。

985132人目の素数さん:2008/04/30(水) 09:27:54
>>982
「面白い」みたいな各人の感覚によるものに定義を求めてどうするのよ

ここは面白い問題スレなんだから、出題者は面白いと思って出してるはずでしょ
それがどこなのかを聞くのは別段おかしいことじゃないと思うんだけど

>>983
他の人が面白いと思って無くても、やってる本人は面白いと思ってるはず
まあつまらないけどやってる研究ってのもあるけど、そういうのは不幸だよね
986132人目の素数さん:2008/04/30(水) 20:57:26
保 守
987132人目の素数さん:2008/05/01(木) 07:35:03
□に0〜9までを1つずつ入れ各辺3つの□の合計(全6列)がすべて同じになる組み合わせは全部で何通りあるか


□※□─□─□
│\│/
□※□
│/│\
□※□─□─□
988132人目の素数さん:2008/05/01(木) 17:14:55
類似問題
下の35個の□に1〜35の数字を1個ずつ入れ縦横それぞれの合計が全て同じになる組み合わせは全部で何通りあるか?

□□□□□
□□□□□
□□□□□
□□□□□
□□□□□
□□□□□
□□□□□
989132人目の素数さん:2008/05/02(金) 03:11:55
コンピュータで総当りすればいいような問題はやめにしようぜ。
990132人目の素数さん:2008/05/02(金) 04:23:37
だな。虫食い算スレでも何でも立ててそこでやってくれ。
991132人目の素数さん:2008/05/02(金) 09:00:00
三百一日。
992132人目の素数さん:2008/05/02(金) 21:14:32
次スレを
993132人目の素数さん:2008/05/02(金) 21:55:18
面白い問題おしえて〜な 十四問目
http://science6.2ch.net/test/read.cgi/math/1209732803/
994132人目の素数さん:2008/05/02(金) 21:56:10
このスレを見ている人はこんなスレも見ています。(ver 0.20)
【毎年恒例】北朝鮮が世界的な食品価格高騰で再び飢饉になるみたいです [ニュース速報]
桃太郎伝説について語る第弐拾壱集 [レトロゲーム]
▼○◆○ 高橋尚子の走る道 16 ○◆○▼ [陸上競技]
995132人目の素数さん:2008/05/02(金) 22:25:18
止めを刺そう

King氏ね
996132人目の素数さん:2008/05/02(金) 22:25:39
King氏ね
997132人目の素数さん:2008/05/02(金) 22:26:00
King氏ね
998132人目の素数さん:2008/05/02(金) 22:26:23
King氏ね
999132人目の素数さん:2008/05/02(金) 22:30:52
King氏ね
1000132人目の素数さん:2008/05/02(金) 22:30:58
やぁ〜い、馬鹿King、レスしてみやがれ

思考盗聴〜って書きたいんだろ馬鹿King!

出来ねぇ〜だろ、くやしいか!馬鹿
10011001
このスレッドは1000を超えました。
もう書けないので、新しいスレッドを立ててくださいです。。。